Ensayo Usach EUNACOM

INDICE Página 07 PROLOGO CAPITULO 1 “ENSAYO” PREGUNTAS DE ENSAYO HOJA DE RESPUESTAS 001 A 090 HOJA DE RESPUESTAS 091 A

Views 174 Downloads 74 File size 5MB

Report DMCA / Copyright

DOWNLOAD FILE

Recommend stories

Citation preview

INDICE

Página 07

PROLOGO CAPITULO 1 “ENSAYO” PREGUNTAS DE ENSAYO HOJA DE RESPUESTAS 001 A 090 HOJA DE RESPUESTAS 091 A 180

11 29 31

CAPITULO 2 “FUNDAMENTO DE LAS RESPUESTAS” INDICE TEMATICO DE PREGUNTAS POR ESPECIALIDAD SECCION 1 MEDICINA INTERNA: CARDIOLOGIA ENFERMEDADES RESPIRATORIAS GASTROENTEROLOGIA ENDOCRINOLOGIA NUTRICION DIABETES NEFROLOGIA NEUROLOGIA HEMATOONCOLOGIA INFECCIOSO GERIATRIA REUMATOLOGIA SECCION 2 OBSTETRICIA Y GINECOLOGIA SECCION 3 PEDIATRIA SECCION 4 CIRUGIA: CIRUGIA Y ANESTESIA TRAUMATOLOGIA UROLOGIA SECCION 5 PSIQUIATRIA SECCION 6 ESPECIALIDADES SECCION 7 SALUD PUBLICA

35 37 39 51 61 69 77 83 91 97 105 111 117 133 153 153 165 171 177 189 207

5

INDICE TEMATICO Nº 1 2 3 4 5 6 7 8 9 10 11 12 13 14 15 16 17 18 19 20 21 22 23 24 25 26 27 28 29 30 31 32 33 34 35 36 37 38 39 40 41 42 43 44 45 46 47 48 49 50 51 52 53 54 55 56 57 58 59 60

Tema Pediatría Salud Pública Endocrinología Cirugia General Cardiología Pediatría Reumatología Cardiología Cardiología Psiquiatría Nefrología Salud Pública Cirugia General Salud Pública Reumatología Cardiología Oftalmología Cardiología Pediatría Obstetricia y Ginecología Obstetricia y Ginecología Reumatología Endocrinología Otorrinolaringología Urología Respiratorio Respiratorio Cirugia General Pediatría Geriatría Dermatología Obstetricia y Ginecología Obstetricia y Ginecología Oftalmología Hemato-Oncología Salud Pública Psiquiatría Obstetricia y Ginecología Psiquiatría Obstetricia y Ginecología Obstetricia y Ginecología Gastroenterología Psiquiatría Geriatría Obstetricia y Ginecología Cardiología Salud Pública Cardiología Gastroenterología Cirugia General Gastroenterología Psiquiatría Cirugia General Psiquiatría Psiquiatría Dermatología Pediatría Gastroenterología Infeccioso Psiquiatría

Nº 61 62 63 64 65 66 67 68 69 70 71 72 73 74 75 76 77 78 79 80 81 82 83 84 85 86 87 88 89 90 91 92 93 94 95 96 97 98 99 100 101 102 103 104 105 106 107 108 109 110 111 112 113 114 115 116 117 118 119 120

Tema Salud Pública Obstetricia y Ginecología Obstetricia y Ginecología Geriatría Pediatría Pediatría Pediatría Traumatología Traumatología Pediatría Obstetricia y Ginecología Neurología Urología Hemato-Oncología Gastroenterología Pediatría Respiratorio Endocrinología Cardiología Obstetricia y Ginecología Obstetricia y Ginecología Salud Pública Cirugia General Urología Respiratorio Obstetricia y Ginecología Obstetricia y Ginecología Respiratorio Pediatría Cirugia General Gastroenterología Cardiología Endocrinología Gastroenterología Cardiología Endocrinología Pediatría Obstetricia y Ginecología Obstetricia y Ginecología Obstetricia y Ginecología Cirugia General Gastroenterología Neurología Respiratorio Infeccioso Salud Pública Otorrinolaringología Obstetricia y Ginecología Hemato-Oncología Neurología Pediatría Traumatología Nefrología Nefrología Respiratorio Obstetricia y Ginecología Endocrinología Endocrinología Obstetricia y Ginecología Oftalmología

35

Nº 121 122 123 124 125 126 127 128 129 130 131 132 133 134 135 136 137 138 139 140 141 142 143 144 145 146 147 148 149 150 151 152 153 154 155 156 157 158 159 160 161 162 163 164 165 166 167 168 169 170 171 172 173 174 175 176 177 178 179 180

Tema Urología Respiratorio Neurología Respiratorio Psiquiatría Oftalmología Otorrinolaringología Obstetricia y Ginecología Dermatología Obstetricia y Ginecología Pediatría Pediatría Obstetricia y Ginecología Cirugia General Reumatología Pediatría Obstetricia y Ginecología Dermatología Traumatología Geriatría Pediatría Obstetricia y Ginecología Neurología Gastroenterología Endocrinología Pediatría Pediatría Pediatría Pediatría Hemato-Oncología Obstetricia y Ginecología Endocrinología Pediatría Respiratorio Nefrología Pediatría Obstetricia y Ginecología Nefrología Otorrinolaringología Endocrinología Salud Pública Hemato-Oncología Pediatría Cirugia General Infeccioso Psiquiatría Psiquiatría Psiquiatría Psiquiatría Obstetricia y Ginecología Pediatría Psiquiatría Traumatología Urología Gastroenterología Pediatría Pediatría Pediatría Pediatría Infeccioso

40

41

MEDICINA INTERNA - CARDIOLOGIA Perfil: Embolia Pulmonar; diagnóstico sospecha, tratamiento inicial y derivación.

vasculares pulmonares (RVP) >320 dinas.s.cm; 3) presión media de la arteria pulmonar (PMAP) >30mmhg; 4) obstrucción >50% del lecho vascular pulmonar, quirúrgicamente accesible y en clase funcional IV de acuerdo con los criterios de la NYHA. Filtro de Vena Cava Inferior. Sus dos principales indicaciones son : a) Hemorragia activa que impide anticoagulación, que es el caso clínico y b) Trombosis venosa recurrente a pesar de anticoagulación intensiva. Si no tuviese un ACV reciente, los trombolíticos podrían estar indicados ya que han demostrado su utilidad para revertir rápidamente la insuficiencia del corazón derecho. La tromboendoarterectomía pulmonar está indicada en pacientes con hipertensión pulmonar debida a un TEP anterior que no es el caso clínico que se presenta. Por lo tanto cuando no puede realizarse la anticoagulación suele ser necesaria la inserción de un filtro en la vena cava inferior, respuesta correcta D. Administració n de trombolíticos. En los pacientes con TEP es una buena alternativa dentro de su manejo, actuando hasta 14 horas después de ocurrido. El tratamiento consiste en el empleo de 100 mg de tPA (activador del plasminógeno tisular) en aplicación Intravenosa, continua, durante dos horas. Pero dentro de las contraindicaciones de esta terapia se encuentran la enfermedad intracraneal aguda, cirugías recientes o los traumatismos. Alternativa incorrecta E.

5.- Hombre de 73 años insuficiente renal crónico que hace una semana presentó un accidente vascular cerebral. Desde hace 18 horas con dolor, aumento de volumen e impotencia funcional en la extremidad inferior. Trae ecografía Doppler compatible con trombosis venosa profunda. Ingresa con malestar general, disnea súbita y signos de insuficiencia cardiaca derecha. ¿Cuál de los siguientes es el tratamiento de elección ? A.- Administrar heparina intravenosa no fraccionada. B.- Administrar anticoagulantes orales. C.- Indicar tromboendoarterectomía pulmonar. D.- Derivar para inserción de filtro en la vena cava inferior. E.- Administrar trombolíticos. Respuesta correcta D Estamos ante una trombosis venosa profunda complicada con un tromboembolismo pulmonar (TEP). La clave para contestar esta pregunta es el antecedente de un ACV hace solamente una semana. El uso de Heparina esta contraindicada en el caso de ACV reciente, ya que una de las complicaciones principales es la hemorragia potencialmente fatales o intracraneales. Alternativa incorrecta A. Con la administración de anticoagulantes orales, al igual que con la heparina la principal complicación es la hemorragia fatal, pudiendo utilizarse crioprecipitados de plasma fresco ante alguna complicación para corregir la hemostasia. Alternativa incorrecta B. La indicación de tromboendoarterectomía pulmonar es para los pacientes que presentan Hipertensión pulmonar debido a un Tromboembolia Pulmonar anterior , disnea al ejercicio minimo o de reposo. Sus indicaciones son: sin evidencia de mejoría después de 6 meses de tratamiento anticoagulante; 2) resistencias

Bibliografía 1. Harrison. 15° edición, Pág. 1770.

42

2. Harrison 16ª edición, Pág. 1729.

Perfil: Pericarditis específico.

-

ECG;

Angina Inestable se define si posee una de las 3 características anteriores: 1) surge durante el reposo (o a ejercicio mínimo) durando más de 10 minutos , 2) es intensa y de comienzo reciente (no diagnóstico

8 .- Hombre de 45 años que presenta un ECG con elevación del segmento ST en todas las derivaciones salvo en AVR , sin cambios significativos en los complejos QRS. Varios días después otro ECG muestra segmento ST isoeléctrico y alteraciones de la onda T. En controles posteriores de electrocardiograma la onda T se aplana y se hace negativa ¿Cuál de los siguientes es el diagnóstico más probable?

más de 4 a 6 semanas anteriores), 3) su perfil es de intensificación constante. Al electrocardiograma se puede observar depresión o elevación del segmento ST asociado con inversión de la onda T, no siendo concordantes como signo de isquemia. Alternativa incorrecta B El infarto no Q presenta las siguientes características electrocardiográficas: elevación de ST en 2 o más derivaciones contiguas y signos de isquemia en onda T . Asociado con Marcadores cardiacos positivos como es el caso de CK-MB y troponinas. alternativa incorrecta C. Los signos Electrocardiográficos de una insuficiencia cardiaca son poco específicos, pero cabe señalar que el crecimiento del ventrículo derecho, visualizado en el ECG como Rs en V1 y rS en V5 o V6 suelen asociarse a insuficiencia cardiaca. Como también el de crecimiento ventricular izquierdo (con S en V1 + R en V5 o V6 >o= 35 mm y/o alteraciones de la repolarización), también con crecimiento biventricular, podríamos hablar de insuficiencia cardiaca. Alternativa incorrecta D. En un alto porcentaje de casos de pericarditis aguda cuando presentan frotes pericárdicos, se observan alteraciones de la repolarización. En el ECG al inicio se observa un supradesnivel de ST de concavidad superior, excepto en AVR y/o V1 donde puede haber una depresión reciproca de ST. Luego hay un aplanamiento de la onda T y posteriormente una inversión de esta. En la pericarditis se puede también observar cierto grado de reducción de los complejos QRS en relación al voltaje, frente a pacientes con grandes derrames pericárdicos. Todos estos cambios son

A.- Angina de esfuerzo B.- Angina inestable C.- Infarto no Q D.- Insuficiencia cardiaca E.- Pericarditis Respuesta correcta E En la Angina de Prinzmetal se observa una elevación del ST en dos o más derivaciones contiguas de forma transitoria, causado por espasmo focal en una arteria

epicárdica coronaria. Alternativa incorrecta A.

43

reversibles al ECG desapareciendo tras unos meses. Alternativa correcta E.

depresión del segmento ST o elevación del mismo, asociado con inversión de la onda T, no siendo concordantes como signo de isquemia. (Alternativa incorrecta A y D). La Angina variante de Prinzmetal da un dolor isquémico, similar al de la angina inestable. Salvo por su falta de relación con el esfuerzo y por la tendencia a presentarse durante el reposo nocturno. El mecanismo responsable es un espasmo localizado en un segmento la rama epicárdica de la arteria coronaria. El ECG suele ser normal en especial en ausencia de obstrucciones coronarias fijas, y durante las crisis aparece la elevación característica del segmento ST. Está indicada la coronariografía para descartar presencia de lesiones arterioscleróticas y mediante éste examen se observan los espasmos coronarios transitorios, signos diagnósticos de la Angina de Prinzmetal. El tratamiento de elección son los antagonistas del calcio (Alternativa correcta B). Angina Postinfarto, puede presentar en cualquiera de las formas de angina, ya sea estable o inestable, sin embargo no es concordante con la clínica que se presente de forma cíclica a la misma hora. (Alternativa incorrecta E).

Bibliografía 1.Farreras, 14ª edición 2.Harrison, 16ª edición.

Perfil : Angina Inestable; diagnóstico específico. 9.- Mujer de 47 años fumadora de 40 paquetes año, hipercolesterolémica, presenta durante el sueño un dolor precordial opresivo que se repite cíclicamente a la misma hora de la noche. IMC : 34. ECG de reposo y esfuerzo normales. ¿ Cuál de los siguientes es el diagnóstico mas probable ? A.- Angina inestable. B.- Angina de Prinzmetal. C.- Angina post infarto. D.- Angina de reposo. E.- Infarto de miocardio. Respuesta correcta B Angina inestable o de reposo se define si posee una de las 3 caracteristicas anteriores: 1) surge durante el reposo (o ejercicio mínimo) durando más de 10 minutos , 2) es intensa y de comienzo reciente (no más de 4 a 6 semanas anteriores), 3)

Bibliografía 1.Farreras, 14ª edición, Pág. 634. 2.Harrison, 16ª edición, Pág. 1601

Perfil: Dislipidemias SiCl 16.- Hombre de 53 años, fumador de 10 paquetes / año, sedentario, hipertenso grado 1. IMC: 22.. Exámenes: Colesterol LDL : 135 mg/dl HDL: 35 mg/dl Triglicéridos : 190 mg/dl ¿ Cuál de los siguientes es el tratamiento de elección para su dislipidemia? A.- Atorvastatina B.- Gemfibrozilo C.- Rozubastatina D.- Ejercicio aeróbico E.- Dieta hipograsa Respuesta correcta E.

su perfil es intensificación constante . Al electrocardiograma se puede observa una

44

De acuerdo a los datos entregados en el

factores de riesgo y LDL >130 y < 160. Su tratamiento es indicar Dieta Hipograsa. Bibliografía 1. Harrison, 15ª edición, Pág. 1625. Cuadro 242-2. 2. Harrison, 16ª edición, Pág. 1581 Cuadro 225-1, 2526-2527. 3.Detection, Evaluation, and Treatment of High Blood Cholesterol in Adults (Adult Treatment Panel lll). May, 2001. www.nhlbi/nih.gov/guidelines/cholesterol/atp3xsu m.pdf

caso clínico, éste paciente presenta 3 factores de riesgo que modifican la concentración ideal de LDL, pero sin antecedentes de Cardiopatía Coronaria. Los principales factores de riesgo a consideran en cualquier paciente con LDL elevados son: -Tabaquismo -Hipertensión Arterial (mayor o igual a 140/90) -HDL< 40 mg/dl -Historia familiar de Cardiopatía Coronaria. -Edad (hombres > 45, mujeres > 55 años). -Estilos de vida como: obesidad, sedentarismo, régimen alimentario aterógeno. -Factores de riesgo Emergentes como: lipoproteína A, Homocisteína, Factores protrombóticos, pro inflamatorios, intolerancia a carbohidratos en ayuno, aterogénesis subclínica. El tratamiento farmacológico esta indicado en pacientes sin cardiopatía coronaria, menos de 2 factores de riesgos y con niveles de Colesterol LDL >190; también un paciente sin cardiopatía coronaria, 2 factores de riesgo y LDL >160. Por último paciente con cardiopatía coronaria o Diabetes Mellitus, con LDL >130. En cada uno de estos casos esta indicado iniciar tratamiento farmacológico. Alternativa incorrecta A, B y C . El ejercicio con actividad física moderada al menos 30 minutos diarios está indicado como prevención de las dislipidemias y como elemento coadyudante dentro de los cambios de estilos de vida, pero no como elemento único de tratamiento. Es útil, en ese caso, una prueba de esfuerzo previo para ver las capacidad funcional del paciente antes de su indicación (alternativa incorrecta D). El tratamiento a elección se determina de acuerdo a la tabla adjunta, según las pautas actuales. Si el paciente se ubica en la población sin antecedentes de Cardiopatía coronaria , 2 o más

Perfil: Electrocardiografía; interpretación de examen, bloqueos. 18.- Paciente de 45 años que presenta ECG con complejo QRS de 0.14 segundos, QS en V1 con T positiva y RR´ en V6 con T negativa. ¿ Cuál de los siguientes es el trastorno de conducción es más probable ? A.- Bloqueo completo de rama izquierda. B.- Hemibloqueo izquierdo anterior. C.- Bloqueo completo de rama derecha. D.- Hemibloqueo inferoposterior. E.- Bloqueo bifascicular. Respuesta correcta A El trastorno de la conducción o bloqueo de las ramas del haz de His puede producirse en la rama derecha, en el tronco de la rama izquierda, en la división superoanterior o en la división inferoposterior. Además de los bloqueos aislados de un solo fascículo, pueden existir bloqueos de dos (bloqueo bifascicular) o de tres fascículos (trifascicular). La zona bloqueada, cualquiera que sea, se despolariza con retraso. En los casos de bloqueo completo tanto de la rama derecha como de la rama izquierda se ha de tomar en cuenta que el diagnóstico se establece fundamentalmente con las derivaciones V1 y V6. Bloqueo Completo de Rama Izquierda, los criterios diagnósticos son: 1) QRS mayor o igual a 0.12 seg. 2) V1: QS o rS con r muy pequeña y T positiva. 3) V1 y V6: R exclusiva con

45

pico de la R después de 0,08 seg. y casi siempre negativa (alternativa correcta A). Hemibloqueo izquierdo anterior: los criterios son: 1) Complejo QRS menor de 0,12 seg. 2) QRS hiperdesviado a la izquierda (sobre todo entre -45° y -75°) . Alternativa incorrecta B Bloqueo completo rama derecha: 1) QRS mayor o igual a 0,12 seg. con empastamientos medio finales. 2) V1: rsR' con cúspide de la R´ empastada y T negativa. 3) V6: qRs con empastamiento de la S y T positiva (alternativa incorrecta C). Hemibloqueo inferoposterior: 1) Complejo QRS menor de 0,12 seg. 2) ÁQRS desviado a la derecha (entre +90° y +140°); para algunos autores mayor o igual a 110° (Alternativa incorrecta D). Bloqueo Bifascicular: Se produce por bloqueo de dos fascículos (Alternativa incorrecta E).

Legión ella sp. La ecocardiografía proporciona un dato de gran valor diagnostico al permitir demostrar la existencia de vegetaciones. Estas se observan en alrededor del 60% de los casos de técnica trasntorácica habitual y en mas del 90% de los casos de técnica transesofágica. Es sobre todo útil en caso de hemocultivos negativos y en la endocarditis sobre prótesis valvulares. Las alteraciones electrocardiográficas son las propias de la patología de base. Con la radiografía de tórax ocurre lo mismo ya que la patología de base definirá la silueta cardiaca que se observe.

Bibliografía 1. Harrison, 15° edición, Pág. 1492. 2. Harison, 16ª edición, Pág. 1458. 3. Farreras, 14° edición, Pág. 508.

48.- Hombre de 71 años, con antecedentes de disnea de esfuerzo, angina de pecho, síncope y dislipidemia tipo I. Examen físico : PA 120/60 mm Hg., FC 72/min. regular, débil. Auscultación cardíaca : soplo mesosistólico de eyección irradiado a las carótidas. ¿ Cuál de los siguientes es el diagnóstico más probable ?

Bibliografía 1. Harrison, 16ª edición, pag.1502.

Perfil : Estenosis Aórtica ; diagnóstico específico.

Perfil : Endocarditis Diagnostico 46 ¿Qué examen se efectúa para confirmar endocarditis infecciosa en válvula nativa?

A.- Endocarditis reumática B.- Insuficiencia mitral C.- Prolapso de la válvula mitral D.- Estenosis aórtica calcificada E.- Miocardiopatía hipertrófica

A.- Ecocardiograma B.- Hemocultivo C.- Electrocardiograma D.-Radiografía de tórax E.- Hemograma

Respuesta correcta D La Cardiopatía Reumática, en mayor frecuencia es causa de insuficiencia Mitral en un tercio de los casos. También puede ser causado por Miocardiopatía hipertrófica. La clínica de insuficiencia mitral es fatiga, disnea de esfuerzo y ortopnea. Con el tiempo, estos pacientes sufren insuficiencia cardiaca derecha con congestión hepática dolorosa, edema maleolar, dilatación venosa del cuello, ascitis e insuficiencia tricuspídea. En relación al soplo este es de tipo Sistólico al menos grado III/VI, pero habitualmente es Holosistólico. Alternativa incorrecta A, B, C, E.

Respuesta correcta B Junto al cuadro clínico , el hemocultivo constituye la base del diagnóstico. En la endocarditis la bacteremia es continua y de poca magnitud. Con dos hemocultivos se aísla el agente etiológico en más del 90% de los casos. La sangre se debe cultivar en medio aerobio y anaerobio y conservar varias semanas para detectar la presencia de bacterias de crecimiento lento.( microorganismos grupo HACEK) estreptococos nutricionales deficientes, Brucela sp.., Corynebacterium sp.,

46

La sintomatología nos orienta a un cuadro de estenosis aórtica, ésta se presenta clásicamente con la triada clínica de: disnea, angina de pecho y síncope de esfuerzo. La sintomatología no se presenta hasta que el orificio valvular se estrecha a unos 0.5 cm2/m2, .La reducción progresiva del área valvular determina que en general la sintomatología no se manifieste hasta los 60 u 80 años de edad. Otros síntomas son: fatiga, debilidad, cianosis periférica Signos de insuficiencia cardiaca derecha, signos de insuficiencia cardiaca izquierda: ortopnea, disnea paroxística nocturna, edema pulmonar, los que se presentan más tardíamente. Los signos son: Pulso débil y lento (parvus et tardus). La presión suele ser normal o con alguna baja en la Sistólica, Pulso yugular con onda “a” acentuada; choque del ápex desviado a izquierda por hipertrofia del ventrículo izquierdo. En la auscultación se puede encontrar un clic de eyección, o soplo sistólico de eyección irradiado al cuello y desdoblamiento paradójico del segundo ruido (S2). Otros datos que nos ayudan al diagnóstico es que el 80% de los afectados son de sexo masculino. El soplo sistólico también nos podría hacer pensar en prolapso de la válvula mitral y miocardiopatía hipertrófica, pero como ya hemos explicado que el cuadro clínico no nos dirige a éstas. En resumen la alternativa correcta es D.

B.- Neurogénico C.- Cardiogénico D.- Distributivo E.- Séptico Respuesta correcta B En el Shock hipovolémico la Presión venosa central y de capilar pulmonar de enclavamiento disminuye, el gasto cardiaco y la saturación disminuyen, en cambio la resistencia vascular sistémica aumenta. La clínica es de: taquicardia, taquipnea, ansiedad, palidez, sudoración, extremidades frías, confusión e incluso pérdida del conocimiento. En tanto la presión arterial desciende y se vuelve inestable incluso con maniobra decúbito dorsal (alternativa incorrecta A). El Shock Neurogénico, surge después de interferencia grave en el equilibrio de los factores vasodilatadores y vasoconstrictores en arteriolas y vénulas. Producido por lesión del sistema nervioso central, secundario a traumas o inducido por fármacos: anestésicos, bloqueantes adrenérgicos o ganglionares y/o sobredosis de barbitúricos o fenotiazinas. En donde frente a un estado de normovolemia con aumento considerable de la capacidad de arteriolas y vénulas (aumento considerable del continente con preservación del contenido) se produce una disminución del retorno venoso y con ello la caída del gasto cardíaco. A menudo las extremidades están calientes, a diferencia de la frialdad por vasoconstricción del shock hipovolémico o cardiogénico (Alternativa correcta B). El Shock Cardiogénico se caracteriza por un bajo gasto cardiaco, por lo que disminuye la presión de perfusión periférica, y aumenta la resistencia vascular sistémica. Estos cambios generan inestabilidad hemodinámica, vasoconstricción periférica y congestión venosa, entre otros efectos. Estos cambios se traducen en una serie de signos clínicos , semejantes a los de un shock hipovolémico, salvo que la presión capilar pulmonar de enclavamiento está aumentada a diferencia del hipovolémico (alternativa incorrecta C). El Shock Distributivo se debe esencialmente a una disminución de resistencia vascular sistémica con la consecuente baja de la

Bibliografía 1. Harrison, 16ª edición, pag.1545-1546

Perfil: Skock; diagnóstico específico, tratamiento incial y derivación. 79.- Hombre de 70 años que después de accidente automovilístico, ingresa en estado de shock. Examen fisico FC 90 /min., regular. PA 70/50 mmHg, FR 18/min. y extremidades inferiores calientes Exámenes de laboratorio Hcto 38% Hb 12 mg/dl. ¿Cuál de los siguientes tipos de shock es el más probable? A.- Hipovolémico

47

presión venosa central y bajo gasto cardiaco en un inicio. Sus causas son múltiples pudiendo ser de origen séptico, anafiláctico, por drogras vasodilatadoras, o por causas endocrinas como tirotoxicosis, mixedema o síndrome de addison (alternativa incorrecta D). En el Shock Séptico hay gasto cardiaco elevado, la resistencia vascular sistémica está disminuida, mientras que la presión capilar pulmonar esta elevada en su fase hiperdinámica. La clínca es de taquicardia, y piernas calientes. En la fase hipodinámica se produce vasoconstricción y el gasto disminuye. El paciente está hipotenso, taquipneico, febril, sudoroso, embotado, con oligoanuria y piernas frías, entre otros (alternativa incorrecta E).

con un máximo de 100 mg por 4 veces al día (Alternativa correcta es A). La segunda elección es un glucocorticoide como la Prednisona (Alternativa incorrecta C) . Una vez que el paciente remite los síntomas y se encuentra afebril se reducen gradualmente las dosis de AINES. Bibliografía 1. Farreras, 14 edición, pag 692; 2. Harrison, 15 edición, pag 1608; 3. Harrison 16 edición, pag 1568.

Perfil : HTA esencial; tratamiento completo, diagnóstico específico.

Bibliografía Harrison, 15ª Edición, Pag. 268 - 270 Harrison, 16ª Edición, Pag. 1772

95.- Hombre 45 años consulta por hallazgo de valores de presión arterial elevados. Hace cuatro semanas era de 165/90 mm Hg, actualmente con 162/90, sin presencia antecedentes mórbidos. El ECG y la creatinina son normales. Además de indicar un diurético. ¿Cuál de los siguientes fármacos debe indicar? A.- Inhibidores de enzima convertidora de angiotensina. B.- Betabloqueador C.- Bloqueador de canales de calcio. D.- Antagonista de Angiotensina II E.- Bloqueador alfa adrenérgico

Perfil : Pericarditis Aguda; diagnóstico específico; tratamiento inicial y derivación. 92 .- Mujer de 22 años, dueña de casa, sin antecedentes mórbidos, presenta cuadro de ocho días de evolución de fiebre y dolor medioesternal intenso, que se modifica con la respiración y tiene tope inspiratorio. ¿ Cuál de los siguientes es el fármaco de elección ? A.- Indometacina B.- Ibuprofeno C.- Prednisona D.- Diclofenaco sódico E.- Paracetamol

Respuesta correcta A En relación a la Hipertensión Arterial, los diuréticos como las tiazidas deben ser los primeros en utilizarse para la mayoría de pacientes con HTA sin indicaciones específicas "compelling indications" (insuficiencia renal crónica,

Respuesta correcta: A Este es un típico caso clínico de Pericarditis Viral. En relación a su manejo, no hay tratamiento específico conocido. Por tanto se manejan los síntomas y se indica reposo en cama. En cuanto al uso de fármacos es de primera elección un AINE. Algunos estudios señalan al ácido acetilsalicílico como de primera línea y otros a la indometacina,

48

insuficiencia cardiaca, postinfarto de miocardio, riesgo elevado de enfermedad coronaria, diabetes mellitus, entre otras). El paciente de la pregunta está en el "stage 2" según el algoritmo de HTA, y la terapia más adecuada es un tiacídico más uno de los siguientes fármacos: primera elección IECA; luego Betabloqueadores, Antagonista del Calcio o bien un Antagonista de la Angiotensina. Alternativa correcta A. Bibliografía 1.Harrison, 15ª edición, pag. 1626-1636. 2 Harrison, 16ª edición, pag 1631. 2.Prevention, Detection, Evaluation, and Treatment of high Blood Pressure. The Seventh Report of the Joint National Committee. May 2003. Figure 1. (JNC7).

49

50

SECCION UNO TEMA DOS ENFERMEDADES RESPIRATORIAS

REVISADO POR DR GUSTAVO MONTALDO SRTA STELLA INTI BARBAGELATA

51

52

53

54

MEDICINA INTERNA – ENFERMEDADES RESPIRATORIAS Paciente que tiene neumonía con criterios de gravedad (bilateral e insuficiencia respiratoria), hay que cubrir empíricamente neumococo pero además podemos pensar que la forma de presentación en este caso es lo suficientemente grave como para considerarla como factor de riesgo de patógenos atípicos y tendríamos que cubrir también Legionella con un macrólido por lo que la respuesta que más se adecua a estos criterios es la alternativa E. La Alternativa A, no es una asociación óptima porque su espectro de acción es similar. Ambas alternativas B y D cubren el neumococo, sin embargo el ciprofloxacino es más eficaz en bacterias Gram negativas y es menos eficaz para Micoplasma y Clamidillas. La opción C, a pesar de cubrir empíricamente neumococo su espectro de acción no abarca microorganismos atípicos (Legionella).

Perfil: Tuberculosis Pulmonar SiCl 26. Mujer portadora de lupus eritematoso sistémico en tratamiento con corticoides hace 10 años. Desde hace tres semanas con compromiso del estado general, fiebre hasta 39º C, disnea y tos con expectoración purulenta . ¿Cuál de los siguientes es el agente etiológico más probable de este cuadro? a) Citomegalovirus b) Haemophilus Influenzae c) Pneumocistys jiroveci d) Criptococcus neoformans e) Bacilo de Koch Alternativa correcta: E Dentro de los patógenos oportunistas: el Haemophilus Influenzae no se asocia a una inmunodepresión específica (opción B incorrecta), el Pneumocistys jiroveci, Criptococcus neoformas y el Citomegalovirus se asocian a pacen con VIH en etapa SIDA (opción A, C y D incorrecta). El uso crónico de corticoides se asocia a Tuberculosis (opción E correcta)

Bibliografía 1. Harrison 16 ed. Pág 1696 cuadro 239 – 9 2. Tratamiento de la Neumonía Adquirida en la Comunidad, Rev. Chilena Enfermedades Respiratorias, abril 2005. vol 21 n 2 pág 117 - 131

Bibliografía 1.- Harrison, Principios de Medicina Interna 15a Ed. cuadro 255 -2 , pág 1730

Perfil: Tuberculosis Pulmonar SiCl 77.- Mujer de 45 años refiere un cuadro de mes y medio de dolor en costado derecho, fiebre de 38° y tos irritativa. Examen Físico pulmonar: disminución del murmullo vesicular en base derecha. Radiografía de tórax: velamiento de la base derecha hasta más o menos la mitad del campo pulmonar. Líquido pleural amarillo neutrófilos 20%, linfocitos 51% Macrófagos 1%, bacilos de Koch negativo Relacion proteínas pleura/proteínas suero 0,6. ¿Cuál de los siguientes es el diagnóstico más probable?

Perfil :Neumonía Adquirida en la Comunidad SiCl 27 Paciente de 64 años, fumador 20 paquetes/año, desde hace 20 años consulta por cuadro de 48 horas de evolución de fiebre y tos con expectoración mucopurulenta Rx T: condensación alveolar en lóbulo inferior derecho y un pequeño infiltrado en el lóbulo inferior izquierdo pH: 7.39 Pa O2: 54 mm Hg y Pa Co2: 29 mm Hg ¿Cúal de las siguientes asociaciones antibióticas es la más adecuada para tratar al paciente?

A.- Hidatidosis pulmonar. B.- Tuberculosis. C.- Embolia pulmonar. D.- Neumonía con derrame pleural. E.- Cáncer de pleura.

A.- Claritromicina + Doxiciclina B.- Ciprofloxacino + Penicilina C.- Amoxicilina + Ácido Clavulánico D.- Ciprofloxacino + Ampicilina E.- Ceftriaxona + claritromicina

Alternativa Correcta: B Los derrames pleurales se clasifican en exudados o trasudado según los criterios de Light. Luego

Alternativa Correcta: E

55

solo se emplean en algunos casos de reagudización. El tratamiento de elección del asma persistente moderada sería la opción B, corticoides inhalados y beta2 de larga duración.

con una relación de 0,6 en la relación PP/PS consideramos al derrame de este paciente como exudado (opciones D y E falsas). La clínica del paciente descarta la posibilidad de hidatidosis pulmonar, ya que esta no se presenta con tos ni con expectoración (opción A incorrecta). La embolia pulmonar es de inicio súbito y se caracteriza por la presencia de eritrocitos en el líquido del derrame pleural. (Opción C incorrecta) No olvides que las características principales del derrame pleural tuberculoso son el aumento de linfocitos, la ausencia de células mesoteliales y eosinófilos, y que, clásicamente, se presenta en pacientes jóvenes. No siendo necesaria para hacer el diagnóstico el test para bacilos de Koch en el líquido pleural. (Opción B correcta)

Bibliografía 1. Guía de Bolsillo de Tratamiento del Asma. Última Revisión 2006. Basado en Programa Gina. Pág 14

Perfil Cáncer Bronquial Primario 88 .- Paciente de 60 años, fumador de 30 paquetes /año por 20 años, en examen preventivo anual se le solicita radiografía de Tx la cual informa un nódulo pulmonar de 3 cms de diámetro. Tac de Tórax: muestra la misma lesión cuyos bordes son indefinidos. No tiene Radiografía de tórax previa ¿Cuál de las siguientes conductas es la indicada?

Bibliografía 1.- Aparato Respiratorio. Fisiología y Clínica. Escuela de Medicina Universidad Católica. E Cruz Mena. R Moreno Bolton Capítulo 50. Pág. 4

Alternativas: A.- Indicar nuevo TAC en 3 meses B.- Indicar nueva radiografía de tórax en 3 meses C.- Iniciar tratamiento para tuberculosis D.- Solicitar Resonancia Magnética E.- Solicitar Biopsia de la lesión.

Perfil Asma Bronquial SiCl 85. Mujer de 22 años presenta respiración sibilante y tos nocturna hace tres meses. En los últimos dos días los síntomas se han acentuado y no la dejan dormir bien. Examen físico: FC. 86 l/min regular. Examen pulmonar : discreta disminución del murmullo vesicular y algunas sibilancias en ambos hemitórax. ¿Cuál de los siguientes es el mejor tratamiento?

Alternativa Correcta: E Una vez que los estudios de detección sugieren la existencia de un cáncer pulmonar es necesario hacer un estudio histológico, en este caso el tamaño y los límites indefinidos son criterios de malignidad que deben descartarse con biopsia. (Alternativa E correcta) El retraso del diagnóstico de certeza pude empeorar la sobrevida del paciente (Opción A y B incorrecta). No hay signos en la TAC de tórax ni en la radiografía de tórax que sean sugerentes de Tuberculosis. (Opción C incorrecta) La resonancia magnética no agrega elementos que modifiquen la conducta de biopsia.

A.- Indicar broncodilatadores B2 estimulantes de corta acción. B.- Indicar broncodilatadores de larga acción asociado a corticoides inhalatorios C.Indicar antibióticos asociados a broncodilatadores anticolinérgicos. D.- Prescribir N- acetil cisteina asociado a antibióticos E.- Prescribir prednisona por vía oral

Bibliografía 1. Harrison, Principios de Medicina 16 ed Pág 570

Alternativa Correcta: A El asma se clasifica clínicamente en intermitente y persistente; y que este último se subclasifica en leve, moderado y grave. Para el tratamiento del asma bronquial en fase estable se deben conocer los escalones terapéuticos secuenciales: beta 2 de corta duración a demanda (repuesta opción A falsa), corticoides inhalados, beta2 de larga duración pautados, y corticoides orales(opción E falsa). Las respuestas C y D son falsas puesto que

Perfil Asma Bronquial SiCl 104.- Paciente de 35 años que consulta por disnea. Gases Arteriales pH: 7.48, Pa O2: 59 mmHg, PaCO2: 26 mmm Hg y HCO3: 26 mEq/L Administrándole O2 al 31% la PaO2 asciende a 75

56

Paciente drogadicto que presenta lesiones múltiples cavitadas en el pulmón sugiere endocarditis tricuspídea con embolia séptica, por tanto, vía hematógena. El compromiso bilateral se observa en aproximadamente Un 60% de los casos. Las neumonías hematógenas, en cambio, tienen un patrón de nódulos múltiples debido al crecimiento centrífugo de los focos metastásicos. Este aspecto radiográfico, en el contexto de una sepsis, es altamente sugerente del origen estafilocócico de la neumonía. Debido a que el paciente presenta factores de riesgo, obliga al clínico a sospechar una etiología más grave que la neumonía adquirida en la comunidad (opción A incorrecta). Los infiltrados centrales en la radiografía de tórax no son los característicos de la TBC pulmonar(opción B incorrecta). Se carece del antecedente de aspiración, por lo cual se descartaría una neumonía por anaerobios (opción C incorrecta) La edad del paciente y las características de los infiltrados pulmonares a la radiografía de tórax me permiten descartar cáncer pulmonar en primera instancia (opción E incorrecta).

mmHg. ¿Cuál de los siguientes es el diagnóstico más probable? A.B.C.D.E.-

Intoxicación por monóxido de carbono Enfermedad neuromuscular Crisis Asmática Atelectasia pulmonar Síndrome de Distress Respiratorio del Adulto

Alternativa Correcta: C El cuadro clínico corresponde al diagnóstico de Asma Bronquial con Hipoxemia. Para poder desplazar el monóxido de carbono fijado a hemoglobina se debe utilizar oxígeno hiperbárico por lo que la alternativa A es incorrecta. La PaCO2 está descendida, por lo tanto probablemente está hiperventilando, lo que descartamos opción B (enfermedad neuromuscular, el paciente hipoventilaría y la Pa CO2 se encontraría elevada) Mejora al administrarle oxígeno por lo que descartamos las causas de shunt (atelectasia y SDRA, opciones D y E descartadas) por lo tanto la opción correcta es la C; se trata de una crisis asmática por lo que el paciente está hiperventilando lo que explica la situación de alcalosis respiratoria que presenta

Bibliografía 1.- Aparato Respiratorio. Fisiología y Clínica. Escuela de Medicina Universidad Católica. E Cruz Mena. R Moreno Bolton Capítulo 30. Pág. 22

Bibliografía Harrison 16 ed. Pág. 1670 – 1671

Perfil: Neumonías Adquiridas en la Comunidad (tipos 1, 2 y 3 de la Sociedad Chilena de Enfermedades Respiratorias).

Perfil: enfermedad Pulmonar Obstructiva Crónica Avanzada - Sicl 122.- Paciente de 68 años, con antecedentes de Enfermedad Pulmonar Obstructiva Crónica refiere incremento de la disnea y del volumen de expectoración purulenta desde hace 2 días. Radiografía de tórax sin infiltrados . Gases Arteriales pH 7,28 PCO2: 53 mmHg y Pa O2: 48 mmHg. ¿Cúal de los siguientes es el tratamiento inmediato a administrar?

115. - Hombre de 30 años adicto a drogas por vía parenteral y marihuana. Desde hace tres días con calofríos, fiebre, dolor torácico y tos con expectoración verdosa con tinte sanguinolento. Radiografía de tórax: infiltrados pulmonares con cavitación central en alguno de ellos. ¿Cuál de los siguientes es el diagnóstico más probable? : A.- Neumonía de la comunidad. B.- Tuberculosis pulmonar. C.- Neumonía por anaerobios. D.- Neumonía hematógena. E.- Cáncer pulmonar.

A.- Oxígeno alto flujo B.- Salbutamol en aerosol C.- Ampicilina oral D.- Prednisona Oral E.- Bicarbonato sódico

Alternativa Correcta: D Alternativa Correcta: B

57

de acción corta (beta-agonistas y anticolinérgicos). Se debe administrar antibiótico porque la exacerbación es infecciosa (aumento de la disnea, aumento del volumen y la purulencia del esputo). Y ventilación mecánica no invasiva (VNMI)(por tratarse de una acidosis respiratoria aguda). La respuesta A es falsa puesto que las mascarrillas nasales no están indicadas en fase aguda. La respuesta B es falsa, porque la sedación inhibiría al centro respiratorio. La respuesta C es falsa porque la intubación no se realiza en este estadio de la enfermedad. La Kinesiterapia respiratoria es el tratamiento a seguir en el tiempo, no es la conducta inmediata, alternativa E incorrecta

La oxigenoterapia sería obligada en este caso, pero no de alto flujo porque disminuiría la hipoxemia que es el principal estímulo del centro respiratorio para iniciar la respiración. (Opción A incorrecta) Salbutamol es el fármaco indicado en guías globales e internacionales para el tratamientote las exacerbaciones ya que produce una broncodilatación directa al actuar sobre el músculo liso bronquial. (opción B correcta) La principal causa de descompensación del paciente EPOC es infección pulmonar bacteriana, sin embargo el tratamiento de elección seria amoxicilina – ácido clavulánico y no ampicilina. (opción C incorrecta) La prednisona oral debe usarse en un curso entre 10 y 14 días, sin embargo no es el tratamiento de inmediato (opción D incorrecta) Estamos ante una acidosis respiratoria, la cual nunca debe tratarse con bicarbonato sino que con Salbutamol, oxígeno y terapia antibiótica. (opción E incorrecta)

Bibliografía 1. Guía EPOC 2007. Sociedad Española de Neumología y Cirugía toráxico. Pág 33

Perfil Embolia Pulmonar Masiva Si Cl- Urg Bibliografía 1. Harrison. Principios de Medicina Interna 16 ed. Pág 1717

154.- Mujer de 70 años hipertensa con insuficiencia renal crónica, acude por disnea súbita con signos de trombosis venosa en miembro inferior derecho. FC 115/min , regular .Gases Arteriales PaO2 55 mmHg, PaCO2 24 mmHg. Creatininemia 3,5 mg/dl .Plaquetas y exámenes de coagulación dentro de límites normales. ¿Cuál de las siguientes es la conducta más adecuada?

Perfil Enfermedad Bronquial Obstructiva Crónica descompensada Sicl, Urg 124. Paciente de 78 años , con antecedente de EPOC grave, (FEV1, 25%), ex fumador, con tres hospitalizaciones en el último año por exacerbaciones de su cuadro. Ingresa por disnea severa y febrícula. No se ausculta murmullo vesicular y se observa expectoración verdosa y abundante. GA (FIO2, 21%) : PaO2 : 36 mmHg; PaCO2 : 63 mmHg; pH : 7,28; HCO3, 31; BE: -6; Rx T : sin infiltrados. Se le administró antibióticos y broncodilatadores (beta2-agonista y anticolinérgico) de acción corta, nebulizados. ¿Cuál de las siguientes es la conducta inmediata a indicar?

A.- Solicitar broncoscopía e iniciar heparina sódica. B.- Solicitar tomografía axial con contraste e indicar heparina de bajo peso molecular C.Solicitar gammagrafía pulmonar de ventilación/ perfusión y administrar heparina sódica D.- Solicitar ecografía doppler de miembros inferiores e indicar heparina de bajo peso molecular E.- Solicitar arteriografía pulmonar y administrar heparina sódica.

A.- Oxigeno por mascarilla. B.- Sedación. C.- Ventilación mecánica. D.- Ventilación mecánica no invasiva. E.- Kinesiterapia respiratoria

Alternativa Correcta: C El cuadro clínico es característico de una tromboembolia de pulmón. En este caso se debe saber que la insuficiencia renal contraindica los contrastes intravenosos (no se puede realizar TAC ni arteriografía), y contraindica el tratamiento con heparina de bajo peso molecular.

Alternativa Correcta: D El paciente presenta una exacerbación de EPOC por lo que se deben administrar broncodilatadores

58

Por lo tanto la única opción que queda correcta es pautar un bolo de heparina sódica, y confirmar el diagnóstico con gammagrafía de ventilación/perfusión Bibliografía 1.- Aparato Respiratorio. Fisiología y Clínica. Escuela de Medicina Universidad Católica. E Cruz Mena. R Moreno Bolton Capítulo 45. Pág. 13 -14

59

60

SECCION UNO TEMA TRES GASTROENTEROLOGIA

REVISADO POR DR. GUSTAVO MONTALDO SR .MANUEL LEAL

61

62

63

MEDICINA INTERNA - GASTROENTEROLOGIA

Perfil: Hepatitis crónica-SiCl; diagnostico de sospecha, tratamiento inicial y derivación. Perfil: Endoscopía digestiva alta y baja-ExIm; diagnóstico de sospecha

42.- Mujer de 19 años desde hace seis meses con episodios de ictericia, fiebre, malestar general, artralgias y elevación importante de transaminasas. No usa fármacos. Exámenes de laboratorio: hipergammaglobulinemia, marcadores de la hepatitis A, B, C y anticuerpos antinucleares negativos. Biopsia hepática: necrosis "en puentes". ¿Cuál de los siguientes exámenes será el mejor para establecer el diagnóstico?

49.- Hombre de 45 años, bebedor de 60 grs. de alcohol diarios, desde hace 6 meses presenta disfagia para sólidos, odinofagia y sialorrea. Exámenes de Laboratorio compatibles con anemia ferropénica. ¿Cual de los siguientes exámenes es el más indicado?

A.-Determinación de factor reumatoide. B.-Anticuerpos antimitocondriales. C.-Anticuerpos anti antígenos microsomales de hígado y riñón. D.- Anticuerpos anti virus de la hepatitis D. E.-Anticuerpos anti virus de la hepatitis E.

A.- Endoscopía alta B.- Resonancia magnética de abdomen C.- Ecotomografía abdominal D.- TAC Abdominal E.- Radiografía simple de abdomen Respuesta correcta A La endoscopía digestiva alta es el Gold Standar en la pesquisa de cualquier y toda causa de disfagia (benigna o maligna), también lesiones que radiquen en el esófago o que se localicen en el estómago. Ese objetivo, se cumple de manera óptima, con la endoscopía alta. El esófago de Barret consiste en la sustitución de

Respuesta correcta C Primero corresponde el análisis clínico del síndrome. La elevación importante de transaminasas es sinónimo de Hepatitis. La evolución en seis meses, se debe catalogar como Sub aguda o Crónica. Se trata de una mujer joven, que no usa fármacos ni ingiere alcohol, lo que descarta en principio, el daño hepático por drogas. No refiere dolor, por lo que se hace poco probable una patología bilio pancreática (obstructiva) Las etiologías virales, A, B, C se descartan por la serología. Sin la coexistencia de hepatitis B, no puede haber infección por virus D. La hepatitis E, es poco probable, por su baja frecuencia en Chile, y por su evolución aguda. Por otra parte, la presencia de artralgias e hipergamaglobulinemia, orientan a una etiología autoinmune. A ello se unen las características epidemiológicas de la paciente que es de sexo femenino y tiene una edad compatible. También debe tomarse en cuenta el aspecto descrito en la histopatología de la biopsia hepática. En consecuencia, es la determinación de anticuerpos anti antígenos microsomales de hígado y riñón, el examen que puede confirmar el diagnóstico de hepatitis autoinmune.

epitelio escamoso del esófago por epitelio cilíndrico. Este revestimiento cilíndrico puede complicarse con la aparición de úlceras o estenosis que característicamente afectan a la parte superior del tercio inferior del esófago.

Bibliografía: Manual CTO 3ª Ed., Digestivo y C. General. Farreras 14ª Ed., págs. 392-395.

Bibliografía:

64

Manual CTO 3a id., Digestivo y C. General 4 Harrison 13° Ed., pág. 1589

Respuesta correcta D En el caso que se nos plantea, el paciente tiene várices esofágicas de gran tamaño con manchas rojas. Estos son indicadores de que existe hipertensión venosa esofágica, con riesgo de rotura y hemorragia de las várices. La profilaxis de

Perfil: Enfermedad inflamatoria crónica intestinalSiCl: diagnóstico de sospecha, manejo inicial y derivación. 51.- Mujer de 24 años, sin antecedentes mórbidos, que presenta diarrea con moco, sangre, y dolor abdominal. De los siguientes: ¿cuál es el examen más adecuado para establecer el diagnóstico? A.- Enema baritado. B.- Ecotomografía abdominal. C.- Colonoscopía. D.- TAC de abdomen y pelvis E.- Leucocitos fecales

hemorragia digestiva alta (HDA), puede ser primaria o secundaria dependiendo de si el paciente ha sangrado previamente o no. En la profilaxis primaria sólo se indican betabloqueadores, para disminuir la presión portal. Si no se toleran o están contraindicados, se realiza ligadura endoscópica.

Respuesta correcta C En un síndrome disentérico en una mujer joven sin antecedentes mórbidos, se deben considerar dos etiologías posibles: infecciosa (en cuyo caso los exámenes a solicitar son coprocultivo y examen parasitológico seriado), o una enfermedad inflamatoria intestinal, para la cual el examen indicado en su diagnóstico, es la colonoscopía.

Bibliografía: Manual CTO de Digestivo y Cirugía General, 4a Edición, pág. 66.

Bibliografía: Manual CTO de Digestivo y Cirugía General, 5a Edición, pág. 35.

Perfil: Cirrosis hepática-SiCl; diagnóstico de sospecha, manejo inicial y derivación, seguimiento y control.

Perfil: Hepatitis agudas B, C, por otros virus, por drogas y tóxicas-SiCl: diagnóstico de sospecha, manejo inicial y derivación, seguimiento y control.

58.- Hombre de 45 años con cirrosis hepática alcohólica sin historia de hemorragia por várices esofágicas. Exámenes: ecografía abdominal sin lesiones focales y endoscopía con várices esofágicas de gran tamaño con manchas rojas. Radiografía de tórax y electrocardiograma normales. ¿Cuál de las siguientes es la conducta más adecuada para prevenir la hemorragia por várices esofágicas?

75.- Hombre de 25 años adicto a drogas endovenosas, que presenta astenia e ictericia. Se le diagnostica hepatitis. Exámenes de laboratorio: Transaminasas más de 20 veces el valor normal. Serología para virus: anticuerpos anti virus C:(+). Ig M anti core del virus B: (-), Antígeno HBs (+); ADN del virus B (-). Ig M antivirus D (+), Ig G anti citomegalovirus: (+), Ig M anti virus A: (-). ¿Cuál de los siguientes diagnósticos es el más probable?

A.- Indicar escleroterapia endoscópica de las várices. B.- Administrar citroflavonoides. C.- Indicar ligadura endoscópica de várices. D.- Administrar betabloqueadores. E.- Derivar para cirugía.

A.- Hepatitis aguda A en un paciente con hepatitis C previa. B.- Hepatitis aguda B en un paciente con hepatitis C previa C.- Hepatitis aguda C en un portador de virus B.

65

E.- Solicitar colangiografía isotópica.

D.- Hepatitis aguda CMV en un paciente portador de virus C y B. E.- Hepatitis aguda por infección D en un paciente portador de virus B, con hepatitis C.

Respuesta correcta C Ante un paciente colecistectomizado que comienza con dolor cólico, en el flanco derecho, debemos pensar como primera posibilidad en coledocolitiasis. El caso clínico nos aporta una ecografía que así lo confirma. Con estos antecedentes, lo indicado es realizar una colangiografía endoscópica retrógrada (ERCP) con fines tanto de confirmación diagnóstica, como terapéutico (extracción de los cálculos).

Respuesta correcta E El paciente presenta una Hepatitis aguda (reflejada en el aumento de transaminasas más de 20 veces los valores normales). El análisis de la serología exige prestar atención a la naturaleza de la Inmunoglobulina pesquisada: IgM o IgG. Ello orienta respecto del tiempo de evolución de la infección: IgM, aguda; IgG, pretérita). La serología de este paciente, revela ausencia de infección actual por virus A y B: IgM anti virus A (-); IgM anti core virus B (-) y anti DNA virus B (-), respectivamente. La presencia de antígeno de superficie del virus B (Ag H.B.s) es indicador de portación crónica por el virus, pero no de infección crónica activa. La presencia de IgG anti virus CMV, es expresión serológica de infección pretérita (pasada) por ése virus, no de infección reciente o en curso. La presencia de IgM anti D, confirma una infección reciente por virus D, en un paciente portador de antígeno de superficie de la Hepatitis B ( condición necesaria para la eventual infección por virus D, como en este caso). Finalmente, esta infección, se produce en un paciente con infección por virus C (la que no podemos determinar si es aguda o crónica), por la presencia de anticuerpos contra dicho virus.

Bibliografía: Manual CTO de Digestivo y Cirugía General 5a Edición, pág. 79

Perfil: Reflujo gastroesofágico-SiCl; diagnóstico especifico, tratamiento completo, seguimiento y control. 94.- Hombre de 45 años que presenta pirosis y regurgitación diaria en los últimos dos meses. Endoscopía digestiva alta: Esofagitis por reflujo y test de Helicobacter pylori:(-) ¿Cuál de los siguientes es el tratamiento más adecuado? A.- Inhibidor de la bomba de protones B.- Antagonista H2 C.- Antiácidos D.- Antibióticos. E.- Antiespasmódicos

Bibliografía: Manual CTO 3ª Ed., Digestivo y C. General. Farreras 14ª Ed., págs. 392-395.

Respuesta correcta A La enfermedad por reflujo gastroesofágico es un proceso muy frecuente. Puede presentarse en sujetos sanos. Puede haber síntomas de reflujo sin lesión esofágica evidente y la esofagitis puede ser asintomática. El grado de esofagitis que presenta el paciente, amerita el uso de inhibidores de la bomba de protones (Omeprazol).La utilidad de los procinéticos es inferior a la de los antisecretores, y la combinación de antagonistas H y procinéticos no supera la eficacia de los inhibidores de la bomba de protones aislados.

Perfil: Colelitiasis-SiCl: diagnóstico especifico, manejo inicial y derivación. 91.- Mujer de 55 años colecistectomizada por vía laparoscópica hace seis años por colelitiasis. Desde hace seis meses presenta episodios de dolor cólico en flanco derecho. Exámenes de laboratorio: bilirrubina 1,4 mg/dl, fosfatasas alcalinas 80 UI/l. Ecotomografía abdominal: coledocolitiasis. ¿Cuál de las siguientes es la conducta a seguir?

Bibliografía: Manual CTO 3ª Ed., Digestivo y C. General. Farreras 14ª Ed., págs. 116-119

A.- Indicar tomografía computarizada abdominal con contraste B.- Indicar colangiografía con contraste C.- Indicar colangiografía retrógrada endoscópica. D.- Solicitar colangioresonancia magnética.

66

indicados cuando no se inicia la diuresis con restricción salina (respuesta D incorrecta) y el reposo en cama, que es otra medida que ayuda a movilizar el líquido ascítica en los pacientes rebeldes al tratamiento, pero una vez que se inicia la diuresis conviene aumentar poco a poco la actividad (respuesta E incorrecta).

Perfil: Síndrome de malabsorción-SiCl; diagnóstico de sospecha, manejo inicial y derivación. 102.- Hombre de 23 años portador de enfermedad celíaca, sometido a dieta sin gluten desde hace 2 años. Después de la indicación de régimen recuperó peso pero siguió con dos a tres deposiciones al día de heces blandas. Examen: grasas en deposiciones: 10 g/día. Anticuerpos anti endomisio tipo IgA, persisten positivos tras dos años de tratamiento. ¿Cuál de las siguientes es la causa más probable de la mala respuesta al tratamiento?

Bibliografía: Manual Washington de Terapéutica Médica. 10ª Edición. Págs. 392-393

Perfil: Pancreatitis aguda y crónica-SiCl: diagnostico de sospecha, manejo inicial y derivación.

A.- Intolerancia a la lactosa. B.- Linfoma intestinal. C.- Esprue colágeno. D.- Esprue refractario. E.- Incumplimiento de la dieta sin gluten.

175.- Hombre obeso y alcohólico de 32 años, que presenta dolor abdominal mesogástrico irradiado a espalda, con náuseas, vómitos e hipotensión. ¿Cuál de los siguientes diagnósticos es el más probable?

Respuesta correcta E La primera causa de fracaso terapéutico en un celíaco, es el incumplimiento de la dieta sin gluten. Si ésta se excluyera debemos entonces considerar la posibilidad de que el paciente presente un linfoma intestinal.

A.- Ulcera péptica B.- Oclusión vascular mesentérica C.- Infarto Agudo Miocardio D.- Pancreatitis Aguda E.- Vasculitis

Bibliografía: Manual CTO de Digestivo y Cirugía General, 5a Edición, pág. 32.

Respuesta correcta D El diagnóstico de Pancreatitis aguda se plantea ante todo paciente con dolor agudo e intenso en el hemiabdómen superior, con o sin irradiación al dorso, acompañado de nauseas y vómitos frecuentes. El cuadro puede acompañarse de compromiso del estado general y hemodinámico (taquicardia e hipotensión) Se confirma con valores elevados de amilasa y lipasa en suero (respuesta correcta D).

Perfil: Ascitis-SiCl : diagnóstico de sospecha, manejo inicial y derivación. 144.- Hombre de 45 años con cirrosis hepática y ascitis. Respecto al tratamiento de la ascitis, la medida terapéutica inicial más importante es: A.B.C.D.E.-

Restricción de la ingesta de agua. Paracentesis. Restricción salina. Diuréticos. Reposo en cama.

Bibliografía: Harrison 16ª Edición Pag. 2089

Respuesta correcta C La restricción salina es la medida inicial más importante del tratamiento de la ascitis. La ingesta diaria de sodio debe situarse por debajo de los 1000 mg, (respuesta C correcta). No es necesaria la restricción sistemática de agua (respuesta A incorrecta). La paracentesis está indicada con fines diagnósticos o para evacuar el líquido en exceso, pero no es la primera medida a tomar (respuesta B incorrecta). Los diuréticos están

67

68

SECCION UNO TEMA CUATRO ENDOCRINOLOGIA

REVISADO POR DR GUSTAVO MONTALDO SRTA. LEONOR MUÑOZ

69

70

71

MEDICINA INTERNA : ENDOCRINOLOGÍA, DIABETES Y NUTRICIÓN. Perfil: Cáncer Tiroides SiCl Bibliografía: Farreras 14ª ed., pág 2140, 2193.

3.- Paciente de 27 años, hace 2 meses con el diagnóstico de feocromocitoma. Se solicitó un cintigrama tiroideo que informa nódulo tiroideo frío, único; calcitonina plasmática: 2600 pg/ml. ¿Cuál es el diagnóstico más probable?

Perfil:. Hipercolesterolemia SiCl 23.- Mujer de 45 años en tratamiento por hipertensión e hipercolesterolemia. Presenta dolor y debilidad muscular generalizada. Examen de laboratorio: Creatininkinasa elevada en sangre. ¿Cuál de los siguientes fármacos causó con mayor probabilidad el cuadro?

A.- Adenoma tiroideo. B.- Carcinoma medular de tiroides. C.- Hiperparatiroidismo primario. D.- Carcinoma folicular de tiroides. E.- Carcinoma paratiroídeo. Respuesta: B. Los feocromocitomas pueden presentarse en forma hereditaria, aislados, o lo que es más frecuente, asociados a otras neoplasias formando parte de los síndromes MEN tipos II, IIb o III. La afectación neoplásica multiglandular de diversos órganos endocrinos se conoce como neoplasia endocrino múltiple (MEN por sus siglas en inglés). Se conocen 3 patrones fundamentales de NEM: a) hiperplasia de paratiroides asociada a adenoma de islotes pancreáticos y a adenoma de islotes o hiperplasia de hipófisis (MEN-I) b) carcinoma medular de tiroides asociado a feocromocitoma y a hiperplasia de paratiroides (MEN-II o MEN-IIa). c) carcinoma medular de tiroides asociado a feocromocitoma y a neuromas mucosos múltiples. (MEN-III o MEN-Iib) (respuesta B correcta) El carcinoma medular tiroideo es semiológicamente indistinguible de otros nódulos fríos en el cintigrama. El nivel normal de calcitonina es menor a 10 pg/ml ,y su elevación, constituye un marcador tumoral precisamente de carcinoma medular de tiroides. El carcinoma folicular de tiroides suele presentarse como un nódulo tiroideo único, indoloro, a veces muy duro, pero no parece ser la clínica de este paciente (respuesta D incorrecta). El hiperparatiroidismo se caracteriza por hipercalcemia que es un estímulo para la producción de calcitonina, y puede formar parte de MEN I y II, pero el hallazgo en el cintigrama orienta a compromiso tiroideo (respuesta C y E incorrectos).

A.- Captopril. B.- Hidroclorotiazida. C.- Lovastatina. D.- Ácido acetilsalicílico. E.- Propanolol. Respuesta: C La tolerabilidad de las estatinas es muy alta. Pueden producir molestias gastrointestinales, aumentos ocasionales y transitorias de CK, miopatías (0.1%), rabdomiolisis, miopatía mitocondrial y dermatomiositis. También pueden elevar las transaminasas hepáticas (AST y ALT) hasta más de tres veces por encima de su nivel normal (0-35 U/L, ambas) Bibliografía: Florez, J; Farmacología Humana, 3ª edición, pag. 952.

Perfil: Hipocalcemia SiCl. 78.- Paciente de 46 años ingresado al servicio de cirugía, que a los dos días de ser intervenido, presenta el signo de Trousseau. ¿Cuál de las siguientes alteraciones metabólicas es más probable que presente el paciente? A.- Hipocalcemia. B.- Hipokalemia. C.- Hipomagnesemia. D.- Hipercalcemia.

72

¿Cuál de los siguientes eventos es desencadenante más probable de este cuadro?

E.- Hiperkalemia. Respuesta: alternativa A El signo de Trousseau es una prueba diagnóstica de la tetania latente consistente en inducir espasmos del carpo al inflar el manguito de un esfigmomanómetro alrededor del brazo a una presión superior a la sistólica durante tres minutos. Resulta positiva en hipocalcemia (respuesta A correcta). Incrementos en los niveles de calcio reducen la excitabilidad neuromuscular, lo que lleva a una hipotonicidad en el músculo tanto liso como estriado, y no a tetania. Entre los síntomas neuromusculares se encuentran la debilidad y la disminución de reflejos profundos en los tendones (respuesta D incorrecta). Las manifestaciones más acusadas de hipopotasemia se refieren al sistema neuromuscular, con concentraciones de potasio sérico entre 2 y 2,5 mEq/L aparece debilidad muscular que puede transformarse en parálisis arrefléxica si el potasio sérico disminuye aún más (respuesta B incorrecta). La hiperpotasemia se manifiesta principalmente en forma de alteraciones neuromusculares y cardíacas. En el sistema neuromuscular puede producir parestesias, debilidad muscular e incluso parálisis fláccida (respuesta E incorrecta). En numerosos enfermos, la disminución de magnesio sérico no ocasiona manifestaciones clínicas. Cuando aparecen síntomas (generalmente con niveles de magnesio inferiores a 1,5 mg/dL) se localizan de modo principal en el sistema neuromuscular, con astenia, hiperexcitabilidad muscular manifestada en forma de fasciculaciones, mioclonías, signos de Chvostek y Trousseau positivos o incluso tetania, pero en el caso de hipomagnesemia no suele presentarse el signo de Trousseau aislado (respuesta C incorrecta).

el

A.- Ingesta de alcohol. B.- Embolia cerebral. C.- Transgresión alimentaria. D.- Abandono de hipoglicemiantes. E.- Automedicación con insulina. Respuesta: A El cuadro concuerda con un episodio de hipoglicemia (respuestas C y D incorrectas puesto que son causa de hiperglicemia). Pacientes con daño hepático en general tienden a la coagulopatía (disminución del tiempo de protrombina y trombopenia) y no a estados protrombóticos que predispongan a embolias (respuesta B incorrecta). Como se trata de un paciente con el antecedente de hepatopatía alcohólica, se asume una ingesta importante de alcohol en el pasado, lo que aumenta el riesgo de ingesta de alcohol en el presente. En estos pacientes el consumo de etanol y las sulfonilureas son causa relativamente frecuente de hipoglicemia (respuesta A correcta). Bibliografía: Harrison 15ª ed., pág 2503, 2049

Perfil: Diabetes Mellitus SiCl. 96.- Hombre de 75 años diabético no insulino dependiente en tratamiento con dieta y sulfonilureas. Actualmente en coma. Dentro de los exámenes de urgencia se pesquisa glicemia de 50 mg/dL. Se administra glucosa intravenosa produciéndose una mejoría evidente en su estado clínico. ¿Cuál de las siguientes es la conducta inmediata a realizar?

Bibliografía: Harrison 15ª ed., pág 2597. Farreras 14ª ed., pág 1841.

Perfil: Alcoholismo SiCl.

A.- Iniciar tratamiento con insulina. B.- Ajustar requerimientos de hidratos de carbono. C.- Hospitalizar para manejo. D.- Enviar a su domicilio con control de hemoglucotest seriado. E.- Enviar a su domicilio con control de glicemia venosa a las 12 horas.

93.- Hombre de 45 años, con antecedente de hepatopatía alcohólica y diabetes en tratamiento con hipoglicemiantes orales. Presenta en forma brusca sudoración, palpitaciones, parestesias, ansiedad y a los 10 minutos marcha inestable, comportamiento agresivo y confusión mental.

Respuesta: C Las sulfonilureas pueden producir con relativa frecuencia estados de hipoglicemia, que en un anciano se puede presentar como una urgencia neurológica. La vida media de las sulfonilureas es bastante larga y la hipoglicemia provocada por

73

tórax y abdomen y gammagrafía ósea (respuesta B y E incorrectas).

éstas puede reaparecer después de varias horas o incluso días, es por esto está indicado ingresar y vigilar al paciente un mínimo de 24 horas (incluso 3 días), e ir suministrando glucosa según las necesidades (respuesta C correcta). La insulina es hipoglicemiante per se por lo que no está indicada en este paciente (respuesta A incorrecta).

Bibliografía: Farreras 14ª ed., pág 2098, 2107. Harrison 15ª ed., pág 2583-90.

Perfil: Tiroiditis Aguda

Bibliografía: Farreras 14ª ed., pág 1957.

118.- Mujer de 22 años presenta dolor de aparición brusca en la región anterior del cuello. Examen físico: T 38 ºC, aumento de tamaño del tiroides con ligero eritema de la zona; exámenes de laboratorio: VHS: 70 mm/hr, T4 libre: 3 mg/dL (normal: 0.5-4 mg/dL), TSH 3.0 mU/mL (normal: 0.5-5 mU/ml). ¿Cuál de los siguientes es el diagnóstico más probable?

Perfil: Hiperparatiroidismo SiCl 117.- Mujer de 55 años, intervenida de cáncer de mama hace 3 años, con buen estado general. En exámenes de control se pesquisa una calcemia de 11, 1 mg/dL. ¿Cuál es la primera prueba a realizar?

A.- Nódulo solitario hiperfuncionante B.- Bocio Hipertiroídeo C.- Tiroiditis subaguda de Quervain D.- Tiroiditis de Hashimoto E.- Tiroiditis aguda

A.- Niveles de PTH. B.- Densitometría ósea. C.- Niveles de 25-OH-D. D.- Niveles de péptido relacionado con la PTH (PrPTH). E.- Gammagrafía ósea.

Respuesta: E La tiroiditis aguda es un proceso infrecuente debido a infección supurativa de la glándula tiroides. En niños y adultos jóvenes, la segunda causa más frecuente de este trastorno es la presencia de un seno piriforme, vestigio de la cuarta bolsa branquial que conecta la bucofaringe con el tiroides, comprometiendo predominantemente el lóbulo izquierdo. Los pacientes acuden a consulta con dolor tiroideo, a menudo referido a la garganta o los oídos y un bocio pequeño e hipersensible que puede ser asimétrico. Son frecuentes la fiebre, la disfagia, el eritema sobre la glándula tiroides, al igual que los síntomas sistémicos de enfermedad febril y la linfadenopatía. El dolor de aparición brusca ayuda al diagnostico diferencial. La VHS está elevada y el recuento leucocitario también, pero la función tiroidea es normal. La biopsia muestra PMN. El tratamiento se basa en la antibioticoterapia. La tiroiditis subaguda de Quervain cursa con alteraciones de la hormona tiroidea. Además como su nombre lo indica, no es un cuadro de aparición brusca. Las respuestas A, B y D no cursan con dolor.

Respuesta: A El nivel normal de calcio total es de 9.0 a 10.5 mg/dL por lo tanto se trata de una hipercalcemia asintomática cuya causa más frecuente es el hiperparatiroidismo primario, que debe confirmarse con la medición de PTH que debiera encontrarse anormalmente elevada para los niveles de hipercalcemia (respuesta A correcta). La hipercalcemia también puede ser la primera manifestación de una neoplasia maligna (segunda frecuencia en causa de hipercalcemia) pero lo frecuente es que la enfermedad de fondo sea evidente, y donde la hipercalcemia sería causada por la elaboración de mediadores tumorales por parte de células malignas como PrPTH, que no es el caso (respuesta D incorrecta). Las hipercalcemias relacionadas a la vitamina D son mucho menos frecuentes y pueden deberse a ingesta excesiva o a el metabolismo anormal de la misma. La prueba tiene escasa especificidad y no resulta rentable para el diagnóstico diferencial (respuesta C incorrecta). Cuando existe hipercalcemia sin niveles altos de PTH, se debe sospechar una neoplasia oculta y debe realizarse estudio exhaustivo en busca de neoplasia mediante radiografía de tórax, TC de

Bibliografía: Farreras 14ª ed., pág 2355, Harrison 15ª ed., pág 2426

74

C.- Diabetes insípida. D.- Enfermedad de Addison. E.- Síndrome paraneoplásico.

Perfil : Diabetes Mellitus SiCl 145.- Mujer de 18 años, diabética desde los 13 años, en tratamiento regular e intensivo con insulina, Hemoglobina glicosilada:7 %; IMC:20. Comienza a tener crisis de mareo y sudoración al final de la mañana. ¿Cuál de las siguientes es la conducta correcta?

Respuesta: D. Los valores normales son: Na+ 136-145 mEq/L; HCO3: 24-31 mEq/L; K: 3.5-5.0 mEq/L; Na+ urinario 40-220 mEq/L. Por lo tanto nos encontramos frente a una hiponatremia, hiperpotasemia, bicarbonato bajo y sodio urinario normal. El síndrome de secreción inadecuada de ADH (SIADH) presenta sodio elevado en orina, bajo en sangre por hipovolemia, pero el potasio no está alterado (respuesta A incorrecta). En el síndrome hepatorenal, en cambio, la concentración de sodio urinaria suele ser menor a 20 mEq/L (respuesta B incorrecta). La diabetes insípida causada por una deficiencia de ADH, presenta natremia ligeramente elevada con potasio conservado (respuesta C incorrecta). El carcinoma pulmonar de células pequeñas puede ser una causa de SIADH. La diabetes insípida se debe a la disminución de la secreción o acción de la ADH constituyendo un síndrome que se caracteriza por la producción de cantidades anormalmente elevadas de orina diluida, cursando con osmolaridad sérica aumentada (respuesta E incorrecta). La insuficiencia suprarrenal (enfermedad de Addison) se manifiesta por debilidad muscular generalizada , pigmentación de la piel, inapetencia progresiva y pérdida de peso, hipotensión (disminución del tono vascular adrenérgico y pérdidas urinarias de sodio). En el laboratorio el cloro, bicarbonato y sodio están disminuidos, existe una gran pérdida de sodio urinario (déficit de aldosterona) y se eleva el potasio en la sangre (respuesta D correcta)

A.- Retirar la insulina. B.- Descartar insulinoma. C.- Agregar colación a mitad de la mañana. D.- Agregar hipoglicemiantes orales. E.- Sustituir insulina por hipoglicemiantes orales. Respuesta: C Los pacientes con DM tipo1, son habitualmente de difícil manejo y son proclives a hipoglicemias. En este caso, podemos inferir que ella lleva un muy buen control metabólico: el peso normal y tiene una excelente hemoglobina glicosilada. Por lo tanto, la solución no pasa por introducir cambios en la dosis de insulina que recibe en la mañana, ni menos agregar hipoglicemiantes orales, sino que la solución eficaz ,fácil y sin riesgo corresponde a administrar una colación a mitad de mañana. (respuesta C correcta). El objetivo del tratamiento es alcanzar el óptimo control glicémico, que en el caso de Diabetes Mellitus (DM) tipo 1 requiere forzosamente el uso de insulina (respuesta A y E incorrectas). Se han descrito casos de insulinomas en pacientes con DM tipo 2, pero nunca con DM tipo 1, además los síntomas de hipoglicemia de este cuadro debieran ser descritos fundamentalmente luego de ayuno prolongado, debido a la secreción continua y abundante de insulina, como ocurre previo al desayuno (respuesta B incorrecta) Bibliografía: Farreras 14ª ed., pág 1969-1971.

Bibliografía: Farreras 14ª ed., pág 2394, Harrison 15ª ed., pág 2454.

Perfil: Enfermedad de Adisson SiCl

Perfil: Hipotiroidismo SiCl

152.- Mujer de 35 años con cuadro de debilidad, pérdida de peso y lipotimias. Exámenes de laboratorio: Na+: 115 mEq/L; HCO3 17 mEq/L; K:5.3 mEq/L, Sodio urinario : 43 mEq/L. ¿Cuál de los siguientes es el diagnóstico más probable?

160.- Hombre de 60 años, hipotiroideo, que inicia tratamiento con levotiroxina, 25 mcg que aumenta en forma progresiva. Durante esta fase del tratamiento aparece angina estable que resulta de difícil control. Los niveles de TSH aún no llegan al valor normal. ¿Cuál de las siguientes es la conducta apropiada?

A.- Síndrome de secreción inadecuada de ADH. B.- Síndrome hepatorenal.

75

A.- Suspender la tiroxina. B.- Incrementar tiroxina en forma más lenta. C.- Indicar diálisis. D.- Derivar para revascularización coronaria. E.- Agregar hidrocortisona al tratamiento. Respuesta: B El objetivo terapéutico del hipotiroidismo primario consiste en ajustar la dosis según los niveles de TSH, pero ésta no sirve para ajustar el tratamiento en el hipotiroidismo secundario. La enfermedad coronaria puede agudizarse con el tratamiento de sustitución, por lo que se debe aumentar lentamente la dosis y vigilar si empeora la angina de pecho, si aparecen arritmias o si empeora la insuficiencia cardíaca. Si los síntomas cardíacos empeoran a pesar del tratamiento médico hay que pensar en la revascularización coronaria, que se puede efectuar sin ningún inconveniente en pacientes hipotiroideos (respuesta D incorrecta). Como se trata de un paciente hipotiroideo que aún no resuelve su patología no está indicada la suspensión de tiroxina (respuesta A incorrecta). En el caso de pacientes con cardiopatía isquémica la terapia hormonal de sustitución con levotiroxina, suele ser un factor desencadenante o agravante de crisis de angina, por lo que las dosis iniciales de levotiroxina deben ser muy bajas ( 12.5 mcg. o 25 mcg.) y los incrementos deberán implementarse lenta y espaciadamente (respuesta B correcta). La hidrocortisona se debe agregar en el caso de hipotiroidismo secundario o terciario con hipocortisolismo asociado, que no es el caso de este paciente (respuesta E incorrecta). Bibliografía: Manual Washington de Terapéutica Médica 10ª Edición págs. 488 – 489. Harrison 15ª edición págs. 2420, Farreras 14ª ed., pág 2072

76

SECCION UNO TEMA CINCO NEFROLOGIA

REVISADO POR DR. GUSTAVO MONTALDO SR. MAXIMILIANO ROBLES.

77

78

79

MEDICINA INTERNA - NEFROLOGÍA

Perfil: Situación clínica - Insuficiencia renal crónica11.- Hombre de 57 años, portador de Insuficiencia Renal Crónica etapa II. Usted constata que sus cifras de presión arterial han aumentado a pesar de la restricción de sodio y el empleo de diuréticos. Si se decide incorporar antihipertensivos ¿cual de los siguientes tendrían además un efecto beneficioso frente a su condición renal?

Perfil: Situación clínica - Nefropatía diabética-SiCl

A.- Antagonistas de la Aldosterona B.- Diurético tiazídicos C.- Antagonistas angiotensina II D.- Betabloqueantes E.- Antagonistas del Calcio

A.- Metildopa B.- Hidralazina C.- Amiloride D.- IECAS E.- Betabloqueadores

ALTERNATIVA CORRECTA: C El control estricto de las cifras de presión arterial cercanas a 130 / 80, influye de manera beneficiosa sobre el ritmo de progresión de una IRC. No sólo el efecto favorable per se que conlleva un buen control de presión arterial influye, sino que también la selección de los antihipertensivos agrega ventajas importantes. Se ha demostrado tanto experimental como clínicamente que los fármacos que bloquean el sistema renina-angiotensina II (IECA y ARA II), contrarrestan de manera notable la progresión del daño renal, tanto en nefropatías diabéticas como no diabéticas, demostrando que los pacientes tratados con IECA presentaban una pérdida de filtrado glomerular significativamente más lenta que los tratados con otros hipotensores, sin diferencias en la presión arterial que explicaran esta evolución divergente. A diferencia de lo obtenido con la restricción proteica, la influencia favorable del bloqueo de la angiotensina II sí que tiene trascendencia clínica, con enlentecimientos llamativos en muchos casos o incluso estabilizaciones de la función renal. La repercusión de los IECA o ARAII sobre la progresión de la IRC guarda una relación más estrecha con su conocido efecto antiproteinúrico que con el control de la PA. (alternativa C correcta)

ALTERNATIVA CORRECTA: D En manejo de la nefropatía diabética consiste en retardar la progresión de esta a través del control de la glicemia, presión arterial sistémica e intraglomerular. Los inhibidores de la ECA (IECA) y los ARA II (antagonistas de receptores de angiotensina II) constituyen los medicamentos de elección para el control de la HTA sistémica e intraglomerular ya que inhiben las acciones de la angiotensina II en la vasculatura sistémica y de la arteriola eferente. Además atenúan el efecto estimulador de la angiotensina II en el crecimiento de la célula glomerular y la producción de matriz extracelular En consideración a que varios ensayos controlados y aleatorizados han demostrado que los IECA retrasan la progresión de la nefropatía diabética en el 50% de los DM tipo 1 y probablemente lo hacen también con los DM tipo 2 , que se recomienda que en todo paciente con diabetes mayor de 12 años en que se detecte microalbuminuria debe recibir IECAs incluso en ausencia de HTA sistémica (Respuesta D correcta).

113.- Mujer de sesenta años, portadora de Diabetes tipo 2 desde hace 11 años, en quien se pesquisa microalbuminuria persistente. ¿Qué fármaco de los siguientes elegiría para intentar frenar la evolución de la incipiente nefropatía diabética?:

BIBLIOGRAFÍA: 1.- Harrison’s Principles of Internal Medicine, 16a ed Part IX, Cap. 264 pág. 1689

BIBLIOGRAFÍA: 1.- Harrison’s Principles of Internal Medicine 16a ed. Part XI Disorders of the Kidney and Urinary Tract; Cap. 261 Chronic Renal Failure , pág. 1658 2.- Avendaño, Nefrología Clínica, 2ª Ed. 2003; Capítulo 18.4. Tratamiento conservador de la IRC. Medidas generales y manejo dietético

Perfil: Situación clínica - Hiponatremia aguda grave 114.- Hombre con antecedentes de alcoholismo y daño hepático crónico es hospitalizado por cuadro de 3 días de evolución caracterizado por aumento de volumen de ascitis, dificultad respiratoria progresiva

80

líquidos orgánicos: alteraciones del sodio Tabla 2.2.4 y 2.2.5

y edema en extremidades. En el transcurso de la hospitalización aparece cefalea, náuseas, vómitos, debilidad muscular progresiva y finalmente convulsiones. Se le toman exámenes donde destaca natremia 120 meq/l, potasio 3,7 meq / l, cloro normal y función renal límite. ¿Cuál de los siguientes cuadros clínicos puede corresponder a una complicación del tratamiento?

Perfil: Situación clínica - Pielonefritis aguda no complicada 155.- Mujer de 25 años presenta desde hace 48 hrs. un cuadro inicial de disuria, polaquiuria, urgencia miccional y dolor suprapúbico. Refiere que el dolor luego se ha extendido al flanco y fosa renal izquierda, acompañándose de sintomatología general y fiebre de 38,5°C. Ante este cuadro clínico ¿cuál de las siguientes actitudes considera menos adecuada?:

A.- Mielinolisis pontina. B.- Rabdomiolisis C.- Edema cerebral D.- Hipertensión arterial maligna. E.- Degeneración microvacuolar cerebral ALTERNATIVA CORRECTA: A Se ha sugerido que la rápida corrección de la natremia es un factor importante pero otros datos más recientes indican que son la corrección en términos absolutos y la presencia de otras patologías concomitantes los principales factores que influyen en la génesis del Síndrome de Desmielinización Osmótico (Mielinolisis pontina central), que se caracteriza por parálisis fláccida, disartria y disfagia, que no tiene tratamiento específico y se asocia a significativa morbi - mortalidad, siendo los pacientes con hiponatremia crónica los más susceptibles, explicándose por los fenómenos de adaptación

A.- Solicitar una orina completa B.- Solicitar cultivo corriente de orina. C.- Inicio tratamiento empírico con cefalosporina de 2a generación por 7 días. D.- Preguntar a la paciente si está o cree que pudiera estar embarazada. E.- Realizar urocultivo de control entre quince y treinta días después post - tratamiento. ALTERNATIVA CORRECTA: C La presencia de fiebre, síntomas generales, dolor en flanco y fosa renal y síntomas irritativos sugiere que

la viñeta clínica se refiere a una pielonefritis aguda (PNA) no complicada. Salvo en ITU bajas no complicadas en mujeres, un urocultivo u orina completa debe ser solicitado para confirmar la infección antes de iniciar tratamiento empírico (Respuestas A y B correctas), para luego con los resultados del antibiograma ajustar la terapia antibiótica. En general las ITU bajas no complicadas responden a cursos cortos de antibióticos (no menos de 3 días, puesto que aumentan las recidivas). En cambio para las ITU altas (PNA) cursos cortos, incluso de 7 días son insatisfactorios en algunas ocasiones, requiriendo éstas tratamiento más prolongado. Por lo tanto en una mujer con una PNA no complicada (sin evidencia de patología urológica

cerebral. (alternativa A correcta) Puesto que el riesgo de muerte o daño neurológico en pacientes hiponatrémicos no tratada es mayor que la posibilidad teórica de desmielinización tras su corrección, en la clínica no debería plantear dudas el uso de suero salino hipertónico en pacientes sintomáticos, considerando los límites mencionados durante las primeras 24 a 48 horas. BIBLIOGRAFÍA: 1.- Harrison’s Principles of Internal Medicine, 16a ed Part II Cardinal Manifestations and Presentation of Diseases; Cap 41 Fluid and Electrolyte Disturbances; Pág. 256. 2.- Avendaño, Nefrología Clínica, 2ª Ed. 2003; Capítulo 2.2. Trastornos de la osmolaridad de los

81

C.- Quinolonas. D.- Diuréticos. E.- AINES

asociada, ni comorbilidad), cuyo agente es predominantemente E. Coli, la antibioticoterapia (aunque la ruta óptima y duración no han sido claramente establecidos) debe ser de un curso de por lo menos 7 a 14 días con fluoquinolonas, aminoglucósidos o cefalosporina de tercera generación, los que resultan ser usualmente adecuados. (Respuesta C incorrecta, puesto no se contempla en la literatura el empleo de cefalosporinas de segunda generación y además iniciar tratamiento antibiótico sin solicitar urocultivo). El conocer si la paciente está o no embarazada resulta relevante por la antibioticoterapia a emplear y por que aumenta la morbimortalidad de la materno fetal. (respuesta D correcta). La realización de un urocultivo de control para evaluar la respuesta a tratamiento es adecuado dentro del manejo general de la paciente, de modo de confirmar esterilidad de la vía urinaria al final del tratamiento o en su defecto ajustar la terapia antibiótica (respuesta E correcta). BIBLIOGRAFÍA: 1. Harrison’s Principles of Internal Medicine, 16a ed pag. 1720

ALTERNATIVA CORRECTA: E El cuadro clínico que se presenta es compatible con una nefritis intersticial alérgica asociada a una glomerulonefritis de cambios mínimos. El cuadro clínico podría ser confundido con una glomerulonefritis aguda aislada, pero la asociación de retención de desechos nitrogenados, hematuria, eosinofilia y rash cutáneo, obligan a pensar en una nefritis tubulointersticial (triada de fiebre, rash y eosinofilia asociada a hiperazoemia es sugerente de nefritis intersticial alérgica). El Sedimento de orina de las nefritis intersticiales alérgicas muestra característicamente hematuria, a menudo piuria aséptica y ocasionalmente eosinófilos. La Proteinuria es usualmente leve a moderada salvo en los casos inducidos por AINEs, puesto que se asocia a glomerulonefritis por cambios mínimos. Una reacción tubulointersticial aguda y difusa puede resultar de fenómenos de hipersensibilidad a un número determinado de medicamentos , tales como las sulfas, penicilinas, cefalosporinas, fluoroquinolonas, drogas antiTBC (HIN, rifampicina), diuréticos (tiazidas y del asa), antiulcerosos, allopurinol y AINEs. En algunos pacientes la nefropatía tubulointersticial que se desarrolla post ingesta de AINEs se asocia a proteinuria en rango nefrótico e histología de cambios mínimos (Respuesta E correcta)

Perfil: Situación clínica - Nefritis intersticial no infecciosa (aguda o crónica)-SiCl 158.- Hombre de 44 años, consulta por presentar rash cutáneo, fiebre y eosinofilia. Los exámenes en sangre muestran una creatinina de 3 mg/dl. El sedimento urinario presenta eosinófilos, hematuria y proteinuria de rango nefrótico. En la biopsia se observa glomerulonefritis de cambios mínimos. ¿Qué fármaco de los siguientes es, con mucha probabilidad, el responsable de este cuadro?:

BIBLIOGRAFÍA: 1. Harrison’s Principles of Internal Medicine 16a ed. pag. 1705

A.- Sulfamidas. B.- Penicilinas.

82

SECCION UNO TEMA SEIS NEUROLOGIA

REVISADO POR DR OSVALDO OLIVARES SR. MAXIMILIANO ROBLES

83

84

85

86

MEDICINA INTERNA - NEUROLOGÍA Perfil : Situación clínica Compromiso conciencia y vascular. Coma. Diagnostico.

últimos 6 meses. Bajo rendimiento académico en 2 medio. Consume marihuana 1 vez por semana. Estando en una discoteque "disco" después de 3 horas presenta una crisis generalizada convulsiva tónico clónica, con testigos confiables. Al examen de urgencia, obnubilado, amnesia del evento. Glicemia 100mg%, alcoholemia 0.8 gr, por lt, kit de drogas en orina positivo + para canabinoles. Cual de los siguientes es el diagnostico mas probable

de

72.- Varón de 55 años, mecánico automotrIz. Hipertenso desde los 40 años ,mala adherencia terapéutica al tratamiento. Fumador de 10 cigarrillos al día últimos 10 años. Durante el trabajo, presenta bruscamente cefalea, perdida de conciencia, déficit motor derecho con plejia, pupilas de 4 mm.,respiración polipneica regular, temperatura 38.5 grados. Al examen conserva oculomotilidad ojos de muñeca, presenta sinergias de decorticación bilaterales simétricas. Cual es el nivel anatómico de la lesión hemorragia y de HIC.

a. Intoxicación por EXTASIS y alcohol. b. Epilepsia mioclónica juvenil. c. Crisis convulsiva generalizada por privación de sueno. d. Hemorragia subaracnoidea por MAV rota. e. Intoxicación aguda por etanol.

a. Hematoma extenso de ganglios basales talámico izquierdo. b. Hematoma hemisferio cerebeloso derecho. c. HIC con nivel lesional por encima del núcleo rojo. d. Hematoma de la protuberancia. e. HIC con nivel lesional por debajo del núcleo rojo.

Respuesta correcta b. La epilepsia mioclónica juvenil debuta con crisis GTC inducidas por fotosensibilidad y ruidos. Antecedentes de cambios conductuales previos y compromiso cognitivo. Es la forma de epilepsia mas frecuente en adolescentes. Confirmación Dg. registro EEG. patrón epiléptico de poliespigas fotosensibles asociada a mioclonías. La intoxicación por éxtasis da positivo kit de anfetaminas, asociado a hipertermia , agitación y polidipsia. Arritmia cardiaca y hemorragia cerebral. Las crisis convulsivas por privación de sueno, se presentan en epilepsia focales. HSA signos meníngeos , focalidad jamás convulsiones. Intoxicacion por etanol, mantiene la ebriedad clínica por horas, las convulsiones se acompañan de hipoglicemia.

Respuesta correcta c. Sinergias de decorticación se les denomina gamma.,por interrupción de los fascículos corticoespinales descendentes, proyecciones corticorubricas y corticoreticulares. Se explica entonces por excesiva activación de moto neuronas gamma a través de las fibras reticuloespinales. La lesión tiene por tanto un nivel sobre el tentorio. Valor pronostico en el coma. Bibliografía 1.Principios de Neurociencia 2a Edición. Duante H. Haines. Elsiever Science.2004. 2.Stroke.Henry_Barnet;JP. Mohor.3a edición 1998. 3.Journal of Clinical Neurophysiology .Vol17.Num.5 2000 4.Estupor y coma. Fred Plum, Jerome Posner.1981. 5 Neurología Clínica. Bradley y cols. Editorial Elsevier.2005.

Bibliografía. 1. Epilepsia. Diagnostico y tratamiento. M.Campos, A. Kanner. Ed. Mediterráneo,1a ed. 2004. 2. Epilepsia. Vol.45 Supp.2 2004. Raven Press. 3. Epilepsia Vol.34 Supp.3. 1993. Raven Press. 4. Revista de Neurologia.Vol 35 Supp 1 Sept 2002. Ed Vignera. 5. Journal of Clinical Neurophisyology. Vol16. Num4 .1999 6. Emergencias neurológicas. Antoni Roy Perez. Ed. Masson.2005 7. Tratado SET. Trastornos adictivos, Soc española de neurología. Ed. Panamericana.

Perfil Situación Clínica : Epilepsias. Diagnostico especifico. 103.- Varón de 16 años, impulsivo e irritable en

87

Perfil Situación Clínica Diagnostico.

E.- Metamizol intramuscular. . Respuesta correcta a Cefalea en racimos por su clínica. Nos describen una cefalea tipo cluster como un dolor más frecuente en varones que en mujeres que aparece como episodios a rachas que pueden durar varios días. El paciente puede sufrir varias cefaleas al día que se inician de forma súbita típicamente por las noches. La localización es periocular con lagrimeo y congestión, ipsilateral al dolor. En la fase aguda el tratamiento de elección es oxígeno inhalado a flujo elevado durante 1 minuto.

:Mielopatía cervical.

110.- Mujer de 65 anos, modista con antecedentes de hipertensión arterial controlada y diabetes adulto tipo ll. Desde hace 6 meses relata claudicación de la marcha, disminución de fuerzas en ambas manos, cervicalgia moderada con irradiación ocasional a cintura escapular. Leve paresia de EEII, discreta atrofia hipotenar asimétrica. Plantares indiferentes, ROT exaltados bilateral. Discreta ataxia y sensibilidad normal. Cual es el diagnostico.

Bibliografía. Neurocirugía 5a Edición, pág. 59.

a. Malformación de Arnold Quiari tipo ll b. Ependimoma intramedular. c. Malformacion A-V de la medula espinal. d. Mielopatía cervico artrósica. e. Poli neuropatía diabética.

Perfil , Situación Clínica :Hemorragia sub aracnoidea. Diagnóstico Patología AUGE. Complejidad diagnostico y complicaciones. Requiere intervención del especialista.

Respuesta correcta d. Arnold Chiari es congénito y se manifiesta en la segunda década. Un ependimoma presenta un sindrome siringomiélico, fasciculaciones y jamás paresia. MAV medular síntomas isquémicos sensitivo motores por robo .Poli neuropatía diabética no da piramidalismo. La mielopatia cervical es frecuente en mujeres sexta década de la vida, compromiso motor por isquemia a barras espondilóticas y compromiso arteria espinal anterior, indemnidad de esfínteres, ausencia compromiso sensitivo, dolor cervical por espondilosis. Confirmación : Rx col cervical, raquiestenosis. TAC col cervical ídem. MRI cambios de señal en T1 intramedular. Patología importante de diagnóstico tardío correlaciona clínica y niveles anatómicos.

143.- Mujer de 55 anos de edad, climaterio post menopáusico sin terapia hormonal sustitutiva. Presenta HSA confirmando con angiografía invasiva aneurisma carótido oftálmico izquierdo roto. Escala clínica de HUNT _ HESS grado ll . Glasgow 12. A las 72 horas se opera, craniotomía frontal izq. clipaje adecuado de aneurisma , control radiológico post clipaje. A las 24 horas del post operado, Glasgow 9, hemiparesia derecha armónica, afasia global. Cual de los siguientes es el diagnostico mas probable a. Resangramiento del aneurisma por clipaje incompetente. b. Trombosis de senos venoso longitudinal superior. c. Vasoespasmo post operatorio. d. Meningitis post operatoria. e. Higroma sub dural izq. post operatorio.

Perfil Migrañas. Situación clínica Diagnostico especifico. Tratamiento 123.- Paciente de 54 años que desde hace 10 días presenta dolor ocular derecho nocturno, con lagrimeo y gran nerviosismo. Esto le ocurre dos a tres veces durante la noche, despertándolo y obligándolo a salir de la cama. ¿Cuál de las siguientes medidas es la más eficaz para calmar el dolor?

Respuesta correcta c. El vasoespasmo es una complicación frecuente en HSA, tanto pre como post opereatorio. Puede ser radiológico y clinico. Se confirma con angiografia. El resangramiento es poco habitual en un aneurisma clipado con control angiográfico da sinología meníngea y coma. La trombosis del seno venoso ,no es complicación en un aneurisma roto, tiene otra expresión radiológica. La meningitis post operatoria da fiebre alta y signos meníngeos, no es tan precoz. El higroma es de baja presión y

A.- Oxígeno intranasal. B.- Flunarizina oral. C.- Ciclobenzaprina oral. D.- Tramadol oral.

88

no da focalización tan severa y precoz. Bibliografía 1. Neurología Clínica. Bradley y cols. Ed. Elsevier .2005 2. Emergencias neurológicas. Anthony Tey. Ed Masson. 3. Decisión making in adult neurology. Second ED. Mosby.1993 4. Cuidados intensivos neurológicos. L.Castillo, C. Romero, P.Mellado.Ed. Mediterraneo .2004 5. Current therapy in neurological disease. Richard Johnson. â edición. Mosby .2002 6. Stroke. Henry Barnett, J.P. Mohr.3a. Ed.1998.

89

90

SECCION UNO TEMA SIETE HEMATOONCOLOGIA

REVISADO POR DR GUSTAVO MONTALDO SRTA. CLAUDIA MARDONES

91

92

93

94

MEDICINA INTERNA – HEMATOLOGÍA Las anemias se pueden clasificar de varias formas, según la cantidad de fierro, la forma y color de la hemoglobina, y también si son regenerativas o arregenerativas. Esto último se mide según la cantidad de reticulocitos, que son glóbulos rojos recién formados en la médula ósea, sobreviven 24 horas en la sangre, y se liberan para compensar la falta de eritrocitos en las anemias regenerativas. Se miden con el índice reticulocitario (recuento de eritrocitos * htc actual/hematocrito normal). El porcentaje normal es de 0,5 – 1,5%. Causas de anemia regenerativa: Hemólisis, Hemorragia oculta. Causas de anemia arregenerativa: Ferropenia, Anemia de trastornos crónicos, insuficiencia renal

Perfil: anemia hemolítica 35.- Mujer de 35 años con diagnóstico de crisis hemolíticas a repetición asociadas a infecciones urinarias. Tiene antecedentes de cuadros de trombosis. Actualmente presenta orinas oscuras. Exámenes de laboratorio: Hemograma: pancitopenia y test de hemólisis ácida: positivo. ¿Cuál de los siguientes es el diagnóstico más probable? A.- Déficit de glucosa 6 fosfato deshidrogenasa. B.- Déficit de piruvato-kinasa. C.- Hemoglobinuria paroxística nocturna. D.- Aplasia medular. E.- Policitemiavera. Alternativa correcta: C La asociación de leucopenia y/o trombocitopenia o episodios de crisis hemolíticas que cursan con hemoglobinemia y hemoglobinuria deben plantear el diagnostico de HPN. Tiene test de hemolisis ácida positivo, aumenta la LDH, disminuye la fosfatasa alcalina granulocitaria (FAG). En policitemia vera y aplasia medular hay FAG alta. Para el déficit de piruvato kinasa hay que usar test de autohemólisis, y para el déficit de glucosa 6 fosfato deshidrogenada tiene que haber cuerpos de Heinz y esferocitos.

Bibliografía: - Compendio Harrison 14ª Págs. 256-257. - Harrison. 15ª edición, pág.41

edición.

Bibliografía: 1.- Harrison, 15ª edición pág. 821. 2.- Farreras. 14ª edición pág. 1910.

Perfil: anemia hemolítica 74.- Una mujer adulta presenta un síndrome anémico. Exámenes de laboratorio: Hb: 10 g/dl. Hto: 36%. Indice reticulocitario: 3%. ¿Cuál de las siguientes causas es la más probable? Perfi: mieloma

A.- Hemolisis. B.- Ferropenia. C.- Anemia de trastornos crónicos. D.- Insuficiencia renal. E.- Hemorragia aguda.

109.- Hombre de 61 años presenta dolores óseos generalizados, especialmente en la columna, desencadenados por el movimiento. Exámenes de laboratorio: Hb 9.9 mg/dl, VCM 82fl (82-98 fl), calcio 11.1 mg/dl, electroforesis

Alternativa correcta. A

95

de proteínas: peak monoclonal IgG de 6 g/dl. Mielograma: 15% células plasmáticas. ¿Cuál de los siguientes es diagnóstico más probable?

C.- Por enfermedad crónica D.- Por ingesta de quelantes E.- Talasemia.

A.- Leucemia mieloide aguda B.- Leucemia mieloide crónica C.- Enfermedad de Hodgkin D.- Mieloma Múltiple E.- Anemia hemolítica

Respuesta Correcta B Características de una anemia ferropénica son: Hemoglobina baja, hierro sérico bajo y TIBC aumentado. La anemia sideroblástica tiene ferremia normal o aumentada, al igual que la talasemia. La anemia por enfermedad crónica tiene disminuida la TIBC. La anemia por ingesta de quelantes se comporta como anemia ferropénica.

Respuesta correcta D. Es una proliferación maligna de células plasmáticas derivadas de un solo clon. La tríada es plasmocitosis medular>30%, lesiones osteolíticas y componente M en el suero, orina, o ambos. Dolores Óseos (70%) generalmente en espalda y costillas, desencadenado por los movimientos (a diferencia de las metástasis óseas). Puede haber fracturas patológicas por proliferación de células tumorales y activación de osteoclastos. Hipercalcemia aguda y crónica. Susceptibilidad a infecciones bacterianas (75&) debido a la hipogammaglobulinemia difusa. También se alteran los granulocitos y el complemento Las manifestaciones más corrientes son neumonías y pielonefritis. Daño (50%) o Insuficiencia Renal (25%) producto de varias causas: hipercalcemia. Hiperuricemia, infecciones repetidas, depósito de amiloide en glomérulos, e infiltración por células mielomatosas. Anemia NN (80%) por ocupación de la MO por las células tumorales e inhibición de la hematopoyesis secundaria a los productos elaborados por el tumor. Además puede existir ligera hemólisis. VHS elevada. Síndrome de hiperviscosidad: Sd. Raynaud.

Bibliografía: Harrison 15ª edición

Perfil : coagulopatías adquiridas 162- Paciente de 55 años en hemodiálisis por insuficiencia renal crónica que requiere extracción urgente de una pieza dentaria. ¿Cuál de los siguientes tratamientos le administraría para reducir el riesgo de sangrado? A.- Testosterona B.- Estrógenos conjugados C.- Desmopresina D.- Factor VIII E.- Factor de Von Willebrand Respuesta correcta: C En pacientes con- IRC se ha descrito disminución de la agregación plaquetaria y déficit de producción de tromboxano, lo que se puede traducir en tendencia al sangrado, con tiempo de sangría prolongado y alteración variable de la agresión plaquetaria, la alteración de las plaquetas puede tratarse con desmopresina.

Bibliografía: Harrison: 15ª edición, pág. 839.

Perfil: anemia ferropénica Bibliografía: Farreras, 14º Edición. página 2028 Harrison, 15º Edición, página 1822

150.- Mujer de 23 años usuaria de DIU (T de cobre), que consulta por decaimiento, palidez y palpitaciones. Exámenes de laboratorio: Hb 7 mg/dl, hierro sérico 20 ng/dl y TIBC (capacidad de transporte de hierro) de 390 ng/dl (valores normales 250 a 370). ¿Cuál de las siguientes anemias es la más probable? A.- Sideroblástica B.- Ferropénica.

96

SECCION UNO TEMA OCHO INFECCIOSO

REVISADO POR DR GUSTAVO MONTALDO SR FELIPE ZAMORANO

97

98

99

100

MEDICINA INTERNA - INFECTOLOGIA

puede aparecer también al tragar o por tensión emocional. Su ubicación es retroesternal y es uno de los diagnósticos diferenciales de la cardiopatía coronoraria. Los antecedentes de uso de corticoides y antibióticos no son concordantes con espasmo esofágico y nos orientan a otra causa ( alternativa E incorrecta)

Perfil: Candidiasis oral y esofágica-SiCl 59.- Hombre de 40 años, portador de Asma Bronquial descompensada por infección respiratoria alta. Tratado con prednisona e inhalaciones con beclometasona, asociados a amoxicilina. Al séptimo día de tratamiento refiere dolor retroesternal y disfagia ¿Cuál es el diagnóstico más probable?

Bibliografía Harrison 15ª edición, pág. 1385-1387 Harrison 15ª edición Pág. 1920

A.- Esofagitis por cándida B.- Angina de Prinzmetal C.- Esofagitis por reflujo D.- Hipersensibilidad a la penicilina E.- Espasmo esofágico

Perfil: Síndrome mononucleósico-SiCl 105.- Hombre de 26 años consulta por fiebre alta y prolongada, con intenso malestar y fatiga. Además refiere mialgias y cefalea. En la exploración sólo destaca discreta esplenomegalia y una leve irritación faríngea. No se hallan adenopatías cervicales. Sólo destaca en el hemograma, linfocitosis relativa con 11% de linfocitos atípicos. No se detectan anticuerpos heterófílos. ¿Cuál es el agente etiológico más probable?

Respuesta correcta A Dentro de la situación clínica se debe sospechar infección por cándida en este pacientes con tratamiento antibiótico y corticoides. La candidiasis esofágica puede ser asintomática o producir dolor retroesternal o molestias deglutorias. El tratamiento indicado es fluconazol oral 100-200 mg/ 24 hrs. La angina de prinzmetal es una forma infrecuente de angina inestable, causado por el espasmo intermitente y localizado de una arteria coronaria epicárdica. El tipo de dolor es de carácter coronario, los antecedentes de nuestro paciente no son indicadores de angina de prinzmetal y la disfagia no es característica tampoco ( alternativa B incorrecta). La esofagitis por reflujo, es una complicación del reflujo que aparece cuando las defensas de las mucosas son incapaces de contrarrestar el daño producido por el ácido, la pepsina o la bilis que refluyen. Las características clínicas son de una pirosis de larga data a la que luego se le sobreagrega una disfagia, no como en nuestro caso de disfagia en un tiempo corto. (alternativa C incorrecta), La hipersensibilidad a la penicilina Es el efecto adverso más importante de este tratamiento antibiótico, ocurriendo hasta en el 5% de los pacientes. Puede ser inmediata (2-30 minutos), acelerada (1-72 horas) o tardía (más de 72 horas). La gravedad es variable desde simples erupciones cutáneas pasajeras hasta shock anafiláctico, el cual ocurre en el 0,2% y provoca la muerte en el 0,001% de los casos, características que nuestro paciente no presenta (alternativa D incorrecta). El espasmo esofágico se caracteriza por contracciones no peristálticas de gran amplitud y larga duración. El cuadro clínico es un dolor toráxico intenso, suele aparecer en reposo, pero

A.- Isospora belli. B.- S. viridans. C.- Citomegalovirus. D.- Chlamydia trachomatis. E.- Virus herpético humano tipo 7. Respuesta correcta C La manifestación clínica más frecuente de la infección por CMV pasado el periodo neonatal es un síndrome de mononucleosis con anticuerpos heterófílos negativos. Como en este caso, se caracteriza por fiebre elevada prolongada, acompañado a veces de escalofríos, fatiga intensa y malestar. También son frecuentes las mialgias, cefaleas y esplenomegalia (respuesta C correcta). La faringitis exudativa y adenopatias linfáticas cervicales son raras al contrario que en la nononucleosis del VEB) aunque posibles, lo que nos ha de servir de guía para el diagnóstico etiológico si no tenemos más datos. El cuadro clínico de isospora belli es fiebre diarrea acuosa no sanguinolenta, comúnmente se asocia a un cuadro de SIDA (alternativa A incorrecta) El estreptococo viridans es una causa de endocarditis, dentro del cuadro clínico es mas orientador de infección viral no

101

bacteriana (alternativa B incorrecta). La chlamydia es una Infección de transmisión sexual que generalmente cursa con uretritis que se denominan no gonocócicas (alternativa D incorrecta). Virus herpes 7 fue aislado en 1990 a partir de linfocitos T de un hombre sano de 26 años, hasta la fecha no se le ha relacionado definitivamente a enfermedad humana, tiene relación con el exantema súbito (alternativa E incorrecta)

incorrecta). La equinocococis es una infección producida por el cestodo equinococus granulosus, que produce lesiones quísticas uniloculares, que por lo general permanecen asintomáticas hasta que su tamaño o el efecto ocupante de espacio genere sintomatología dependiendo del órgano afectado. Los órganos más comúnmente afectado son hígado y pulmones (alternativa C incorrecta). La toxocarosis es la infección por la toxocara canis en la mayoría de los casos, esta entidad clínica afecta a niños en edad preescolar , discordante con la edad de nuestra paciente por otro lado sus síntomas son fiebre, malestar, anorexia, perdida de peso, tos, sibilancias y exantema por esto la alternativa E es incorrecta.

Bibliografía Harrison 15ª Edición. Págs. 1312-1313. Harrison 16ª Edición Pag. 1167

Bibliografía I. Harrison 15ª Edición Cuadro 211-2. Pag 1400. 1450 2. Harrison 16ª Edición pag. 1389-1400 3. Harrison 15ª Edición pag. 1470

Perfil : Triquinosis-SiCl 165.- Mujer de 40 años que consulta por mialgias y aumento de volumen en el trapecio y deltoides derechos. Hace tres semanas comió embutido de Cerdo. En la exploración es destacable el edema periorbitario. Los análisis de sangre demuestran eosinofilia y aumento de IgE y CPK. ¿Cuál es el diagnóstico más probable?

Perfil: Prevención de infecciones hospitalariasPrDe

A.- Toxoplasmosis B.- Teniasis C.- Equinococosis. D.- Triquinosis. E.- Toxocarosis

180.- Enfermera que tras extraer sangre a un HIV positivo se pincha profundamente con la misma aguja. ¿Cuál es el tratamiento profiláctico más adecuado?:

Respuesta correcta D La triquinosis se desarrolla después de la ingestión de carne con quistes de Trichinella spiralis. Cuando se inicia el enquistamiento de las larvas en el músculo, 2 a 3 semanas después de la infección, aparecen síntomas de miositis (mialgias, edema, debilidad muscular). Aparece eosinofilia en el 90% de los pacientes. Los niveles séricos de IgE y enzimas musculares están elevados en la mayoría de los pacientes (respuesta D correcta). La toxoplasmosis es una zoonosis que no presenta sintomatología en el 80-90% de los casos en el inmunocompetente y en el inmunocomprometido cursa con un cuadro linfoproliferativo de gravedad variable; por lo tanto no es concordante con este caso (respuesta A incorrecta). La teniasis es una infección por cestodos de dos tipos Tenia solium y Tenia saginata. La primera se trasmite por el consumo de carne de cerdo como en este caso, pero el cuadro clínico es con epigastralgia, nauseas, baja de peso y diarrea. Además de un importante numero de casos asintomático (alternativa B

A.- Zidovudina (AZT) + Lamivudina (3TC) 6 meses. B.- Zídovudina (AZT) 1 mes. C.- Zidovudina (AZT) + Lamivudina (3TC) 4 semanas. D.- Ritonavir 3 meses. E.- Ninguno Respuesta correcta C La profilaxis del personal sanitario debe iniciarse tan pronto como sea posible ( 1 ó 2 horas después de la exposición).En este sentido se han asociado varios factores que aumentan el riesgo de transmisión laboral de la infección por VIH: la profundidad de la lesión, la presencia de sangre visible en el instrumental que produjo la exposición, la lesión con un dispositivo que ha estado dentro de una vena o una arteria del paciente que actúa como foco, la enfermedad terminal en el paciente que actúa de foco y la falta de tratamiento antirretrovírico . Después de la exposición en el profesional sanitario la pauta básica incluye el uso de inhibidores de la

102

transcriptasa inversa análogos de los nucleótidos. La combinación más recomendada es la de Zidovudina (AZT) + Lamivudina (3TC). La duración se planteará para 4 semanas (respuesta C correcta). Podemos recordar que el riesgo de transmisión del HIV después de una punción percutánea es de 0.3%. El tratamiento con zidovudina + Lamivudina no ha mostrado ser mas efectivo por 6 meses que por 4 semanas por tanto se prefiere utilizar el tratamiento abreviado por la cantidad de efectos adversos y el costo del tratamiento (alternativa A incorrecta). La combinación de dos antiretrovirales es altamente más efectiva que la aplicación de sólo uno de ellos. (alternativa B incorrecta). El ritonavir corresponde a un fármaco inhibidor de la proteasa, en la norma Minsal lo mencionan como un suplemento de los fármacos inhibidores de la transcriptasa reversa y no como monoterapia ( alternativa D incorrecta). La aplicación de profilaxis farmacológica ha demostrado que disminuyen en un 79 % el riesgo de seroconversión después de un accidente laboral con sangre VIH + confirmado (alternativa E incorrecta) Bibliografía 1-Harrison, 15° edición, pag. 2231; 2-Norma de manejo post exposición laboral a sangre en el contexto de la prevención de la infección por VIH, Norma técnica N º 48 Ministerio de Salud, Marzo 2000

103

104

SECCION UNO TEMA NUEVE GERIATRIA

REVISADO POR DR EDURADO GALLEGOS SRTA CLAUDIA MARDONES

105

106

107

MEDICINA INTERNA – GERIATRIA Perfil: Demencia SiCI 30.- Paciente de 70 años con temblor de reposo de 4 hertz y torpeza en extremidad superior derecha desde hace un año. Al caminar el braceo esta desminuido en el lado derecho. Se inicia tratamiento con levodopa y carbidopa. Al control 1 de los 7 días los síntomas han desaparecido. Cual de los siguientes es el diagnostico mas probable?

Perfil: Artritis reumatoide 44.- Paciente de 70 años bebedora crónica, portadora de artritis reumatoidea y espondilitis anquilosante tratada con metotrexate y prednisona. ¿Cuál de las siguientes patologías tiene mayor riesgo de padecer? A.- Discrasia sanguinea B.- Anemia Ferropriva C.- Leucemia D.-Osteoporosis E.- Amiloidosis

A.- Corea de Huntington B.- Parálisis supranuclear C.- Enfermedad de Parkinson D.- Enfermedad de Creutzfel Jacob E.- Temblor esencial familiar

Alternativa Correcta: D La artritis reumatoide es una enfermedad que se asocia aparición de Sd. de Welter, enfermedad miocárdica, neurológica, infecciones, amiloidosis y osteoporosis, sin aumento del riesgo de cáncer. La amiloidosis se produce en riñón, hígado, bazo, tracto gastrointestinal y vasos sanguíneos en 10-15% de las artritis. La osteoporosis se asocia a activación de arritis reumatoide, asociado a enfermedad de actividad local. La anemia es de tipo crónica, y no es común la alteración de la coagulación en artritis,

Alternativa correcta: C Es una enfermedad caracterizada por movimientos involuntarios de carácter tembloroso, con disminución de la fuerza muscular que afectan a partes que están en reposo y que incluso provocan una tendencia a la inclinación del cuerpo hacia delante y a una forma de caminar a pasos cortos y rápidos. Los sentidos y el intelecto permanecen inalterados. La enfermedad de parkinson se caracteriza por el temblor de reposo (clásicamente de 4 hertz), rigidez, bradicinesia e inestabilidad postural. Se produce por degeneración de las neuronas dopaminergicas de la sustancia negra.

Bibliografía: Oxford textbook of Medicine, capítulo 18, año 2004.

El tratamiento no es curativo, se realiza con fármacos, orientados a restituir la dopamina. Copian o imitan su acción, o reducen su eliminación natural, constituyendo la piedra angular del tratamiento de ésta enfermedad.

Perfil: demencia 52.- Paciente de 70 años, sin antecedentes neurológicos, psiquiátricos ni tratamientos farmacológicos previos, presenta desde hace 8 meses deterioro mental progresivo, fluctuaciones en su nivel de atención y rendimiento cognitivo, alucinaciones visuales. En la exploración neurológica presenta signos parkinsonianos leves. ¿Cuál de los siguientes es el diagnóstico más probable?

Debido a que la dopamina no puede penetrar directamente en el cerebro desde la sangres, se administra en forma de un precursor llamado Ldopa o levodopa, que si penetra en el cerebro y allí es transformada en dopamina, y reduce rápidamente los síntomas. Luego de algunos año, disminuye progresivamente su acción. Bibliografía: Harrison 15tm Ed., Pags. 2806-07

A.- Enfermedad de Alzheimer.

108

B.- Demencia vascular. C.- Demencia con cuerpos de Lewy. D.- Enfermedad de Huntington E.- Psicosis hebefrenica.

21252006000400010&Ing=es&nrm=iso.IS SN 0864-2125. Perfil: Ulceras por presión 64.- En un anciano postrado que padece de ulcera por presión grado 3, los principios en el cuidado son:

Alternativa Correcta: C La enfermedad de Lewy representa del 15 al 25% de los casos y se establece como la tercera causa de demencia, superada solo por la enfermedad de Alzheimer y la demencia predominantemente vascular. Las características esenciales para su diagnóstico son: las fluctuaciones de la función cognitiva con variaciones importantes de la atención y del estado de alerta, las alucinaciones visuales recurrentes bien formadas y detalladas, y las características motoras espontáneas de parkinsonismos. Los extrapiramidales que se observan con más frecuencia son la bradicinesia, la facies inexpresiva y la rigidez

A.- Evitar la presión y la humedad. B.- Desbridar tejidos necroticos, limpieza y cuidados de la ulcera C.- Reparación quirúrgica. D.- Disminuir la fricción y usar emolientes. E.- Antibióticos tópicos. Alternativa correcta: B Las ulceras por presión constituyen un tipo especial de lesiones causadas por un trastorno de irrigación sanguínea y nutrición tisular como resultado de presión prolongada sobre prominencias óseas o cartilaginosas. Se clasifican por evolución en:

La enfermedad de Alzheimer tiene como clínica el deterioro progresivo de la memoria reciente, que se acompaña de conductas desinhibidas y socialmente inadecuadas, agitación psicomotora agresividad, alteraciones del sueño, labilidad emocional y delirium. Es la causa más frecuente de demencia.

CLASIFICACIÓN DE LA EVOLUCIÓN DE LAS UP (SEGÚN AHCPR) Estadio I: Eritema que no palidece tras presión. Piel intacta (en pacientes de piel oscura observar edema, induración, decoloración y calor local).

En la enfermedad de Huntington el inicio suele ser insidioso, precedido por cambios en la personalidad y en la capacidad de adaptación al medio. Los pacientes más jóvenes sufren una forma clínica más grave, pues la rigidez estriatal, las convulsiones y los síntomas extrapiramidales son más marcados, y la incidencia de psicosis es también mayor en edades más avanzadas con un predominio del temblor intencional. El diagnóstico de sospecha clínica se caracteriza por los movimientos coreiformes involuntarios de la cara, manos y hombros o de la marcha, que suelen preceder a la demencia, unidos a antecedentes familiares de enfermedad de Huntington o demencia. Más de la mitad de los pacientes son diagnosticados erróneamente de trastorno psiquiátrico primario.

Estadio II: Pérdida parcial del grosor de la piel que afecta a epidermis, dermis o ambas. Ulcera superficial con aspecto de abrasión, apolla o cráter superficial. Estadio III: Pérdida total de grosor de la piel con lesión onecrosis del tejido subcutáneo, pudiendo extenderse hasta la fascia subyacente pero sin atravesarla. La ulcera aparece como un cráter profundo y puede haber socavamiento en el tejido adyacente. Estadio IV: Pérdida total del grosor de la piel con necrosis del tejido o daño muscular, óseo o de estructuras de sostén (tendón cápsula articular, etc). También puede asociarse a socavamiento y tractos fistulosos en el tejido adyacente.

Bibliografía: Harrison 13a Ed., P.G. 2.618 PEREZ MARTINEZ, Víctor T. Demencias: su enfoque diagnóstico. Rev. Cubana Med. Gen Integr. (online) Oct-Dic. 2006, vol.22, no.4 (citado 27 Octubre 2007), p.0-0 Disponible en la World Wide Web: http://scielo.sld.cu/scielo.php?script=sci_arttext &pid=S0864-

Su tratamiento sistémico incluye nutrición calórico proteica adecuada, aporte vitamina C, Zinc, uso de antibióticos por vía sistémica: solo indicado en casos de bacteremia y/o sepsis, celulitis u osteomielitis; los gérmenes más

109

frecuentes son bacilos gram (-), entero coco y polimocrobiano. Las medidas locales son la indicación de antibióticos tópicos y antisépticos (controvertido). Lo mejor es la irrigación con suero fisiológico. No se recomienda la aplicación tópica de povidona ya que disminuye la posibilidad de cicatrización por poseer un efecto toxico sobre los fibroblastos. Eliminar el tejido necrótico: promueve la cicatrización por poseer un efecto toxico sobre los fibroblastos. Eliminar el tejido necrótico: promueve la cicatrización y previene la infección. Cuando esta cubierta por una escara dura, se sugiere una evaluación para un debridamiento quirúrgico. Usar apositos especializados, como los de poliuretano (Tegaderm) o de hidro coloide (Duoderm) facilitan la cicatrización cuando se aplican sobre ulceras superficiales limpias y sin tejido necrotico. Acumulan líquido seroso sobre la herida y permite la migración de células epiteliales. No tienen utilidad sobre ulceras profundas.

Bibliografía: GONZALEZ PEREZ, A gusto César y LIBRE RODRIGUEZ, Juan de Jesús. Estado confusional agudo en el paciente anciano. Rev Cub Med Mil. (online) Dic.2001, vol.20 supl.5 (citado 26 Octubre 2007 l,p.89-96. Disponible en la World Wide Web: http://scielo.sld.cu/scielo.php?script=sci_arttext &pid=S013865572001000500014&Ing=es&nrm=iso.ISSN0 138-6557

Bibliografía: Farreras, 13tm Edición, página 1296 http://esscuela.med.puc.cl/publ/ManualGeriatria /PDF/EscarasUlceras.pdf

Perfil: síndrome confusional 140.- Paciente de 80 años que presenta estado confusional agudo. ¿Cuál de las siguientes es la conducta más correcta para su menejo? A.- Realizar evaluación clínica periódica B.- Realizar un Minimental de Folstein C.- Identificar la causa que lo provocó D.- Indicar haloperidol E.- Indicar Tomografía axial computarizada cerebral. Alternativa correcta: C Lo más adecuado en un síndrome cofusional es aplicar medidas generales. Vigilar la hidratación del paciente, nutrición parenteral en caso de déficit vitamínico o hipoalbuminemia, suplemento de oxigeno en caso de hipoxia secundaria a neumonías o neoplasia pulmonar, habitación iluminada, clara durante el día y con luz en la noche y buscar la etiología del síndrome confusional.

110

SECCION UNO TEMA DIEZ REUMATOLOGIA

REVISADO POR OFEM

111

112

113

MEDICINA INTERNA - REUMATOLOGÍA La artritis psoriásica se define como una artropatía inflamatoria que se manifiesta en pacientes con psoriasis y se caracteriza por la ausencia de factor reumatoide. De todos los determinantes genéticos, el HLA Cw6 es el principal alelo tic predisposición a la psoriasis, mientras que el HLA-B27 define aquellos pacientes con psoriasis destinados a desarrollar artritis. La psoriasis antecede cronológicamente a la artritis en el 70% de los pacientes, La artritis precede a la psoriasis en el 15% de los casos. La mayoría de los pacientes presentan sinovitis periférica que puede ser de una 0 pocas articulaciones o bien una poliartritis de grandes y pequeñas articulaciones casi siempre asimétrica y con afectación de las articulaciones interfalángicas distales. Algunos pacientes (menos de un 5%) presentan una artritis grave y destructiva -artritis mutilante- de las pequeñas articulaciones de las manos y de los pies. En la radiología existe osteoporosis, presencia de erosiones, reducción del espacio articular y la proliferación ósea. En algunos pacientes con Formas agresivas la destrucción de los extremos óseos de una articulación -habitualmente de las manos, pies o ambos- es muy llamativa y origina imágenes características como la deformidad en "lápiz y copa", en la cual un extremo óseo se afila y el otro se excava. Artritis reumatoide: se asocia al HLA-DR4 y no afecta a las articulaciones interfalángicas distales. Espondilitis anquilosante: no se asocia al HLA-CW6 ni con onicolisis. Cursa con sacroileitis bilateral. La artritis periférica no suele ser mutilante. Esclerodermia: puede cursar con resorción de las falanges distales, pero no cursa con onicolisis ni se asocia a los HLAB27 ni CW6. Cursa con afectación cutánea y visceral característica. Artritis reactiva: la artritis no suele ser erosiva. Existe una infección urogenital o digestiva dos semanas antes. Se asocia al HLA-DR4

Perfil: Artritis séptica Sicl 07.- Hombre 60 años portador de insuficiencia cardiaca, luego de sufrir una caída, consulta por fiebre, calofríos y compromiso del estado general. El examen físico pone en evidencia aumento de volumen, rubor y dolor en la rodilla derecha con impotencia funcional. ¿Qué tratamiento antibiótico empírico debe elegir una vez tomado cultivo del líquido sinovial?: A.- Ampicilina y Gentamicina. B.- Vancomicina. C.- Penicilina y gentamicina. D.- Cloxacilina. E.- Ceftriaxona. Respuesta correcta D El paciente de la pregunta presenta una artritis séptica en relación con la caída. La causa principal de artritis séptica en relación con traumatismos para el grupo de edad del paciente es el S aureus. Además del dolor, es frecuente la aparición de fiebre y la articulación que se afecta en el mayor número de los casos es la rodilla. Los hemocultivos son + en un 50% reflejando la presencia de bacteriemia. La nafcilina, cloxacilina y resto de penicilinas resistentes a betalactamasas son los fármacos de elección del tratamiento parenteral de las estafilococias graves. Es frecuente iniciar el tratamiento de la bacteremia por S. aureus con una combinación de aminoglucósido+betalactámico que presentan un efecto sinérgico (respuesta D correcta). Bibliografía: 1. Harrison. 15" edición, pág. 1059. 2. Farreras. 14° edición, pág 2901. Tabla 336-3.

Perfil: Artritis reumatoídea-Sicl

Bibliografía: 1. Harrison, 15ª edición, pág. 2343

15.- Mujer de 34 años de edad presenta artritis de las articulaciones interfalángicas distales. Es HLA-B27 y CW6 positiva. En la radiografía de manos, se observa la imagen de deformidad en "lápiz y copa". ¿Cuál es el diagnóstico más probable?:

Perfil: Hombro doloroso-Sicl A.- Artritis reumatoide. B.- Artritis psoriásica. C.- Espondilitis anquilosante. D.- Esclerodermia. E.- Artritis reactiva

22.- Mujer de 58 años de edad acude quejándose de dolor en el hombro derecho. No recuerda alguna lesión previa pero siente que el hombro se le está volviendo cada vez más rígido desde hace varios meses. Con anterioridad tuvo varios ataques de bursitis del hombro derecho, los cuales se trataron de manera satisfactoria con antiinflamatorios no

Respuesta correcta B

114

acompañado levantarse y limitación en lateralización solicitar será:

esteroídeos e inyecciones de esteroides. Portadora de diabetes mellitus no insulino requiriente. En la exploración física, el hombro derecho no presenta ni calor y rubor, pero sí doloroso al tacto. El arco de movilidad pasivo y activo está limitado en la flexión, la extensión y la abducción. Una radiografía del hombro derecho muestra osteopenia sin datos de erosión articular u osteofitos. ¿Cuál es el diagnóstico más probable?

de rigidez matutina. Mejora al caminar. El examen físico muestra los movimientos de flexoextensión y de la columna. El primer examen a

A.-Presencia del antígeno HLA-B27. B.-Radiografía simple de sacroilíacas. C.-Cintigrafia de Columna Lumbar. D.- TAC de columna Lumbosacra. E.- Radiografía simple de columna dorsolumbar en distintas proyecciones.

A.- Capsulitis adhesiva B. Necrosis avascular C. Tendinitis bicipital D. Osteoartritis E. Desgarro del manguito

Respuesta correcta B En la espondilitis anquilosante suele haber una sacroileitis radiográficamente demostrable, siendo uno de los criterios diagnósticos (respuesta 2 correcta). La TC y el cintigrama pueden detectar alteraciones en las articulaciones sacroilíacas antes que la radiografía simple pero no se ha establecido aún su valor en el diagnóstico sistemático (respuestas 3 y 4 incorrectas). Las alteraciones en la columna lumbar son más tardías que las de las articulaciones sacroilíacas (respuesta 5 incorrecta). El HLA-B27 no es criterio diagnóstico ni factor pronóstico de la enfermedad

Respuesta correcta A La capsulitis adhesiva se caracteriza por dolor y restricción de movimiento del hombro. Esto suele ocurrir sin que haya una enfermedad intrínseca del hombro, como osteoartritis o necrosis avascular. Sin embargo, es más común en pacientes que han padecido bursitis o tendinitis con anterioridad, igual que en individuos con otras enfermedades generales, como neumopatía crónica, cardiopatía isquémica o diabetes mellitus. No se ha esclarecido la etiología, pero la capsulitis adhesiva al parecer se instaura en pacientes con inmovilidad prolongada. La distrofia simpática refleja también se presenta en personas con capsulitis adhesiva. Desde el punto de vista clínico, este trastorno es más común en mujeres mayores de 50 años de edad. El dolor y la rigidez se desarrollan en el transcurso de algunos meses a años. En la exploración física, la articulación afectada está dolorosa a la palpación y hay restricción del arco de movilidad. El estudio estándar para el diagnóstico es la artrografía y la cantidad de contraste inyectable se limita a menos de 15 ml. En la mayoría de los pacientes, la capsulitis adhesiva experimentará regresión espontánea en un lapso no mayor de uno a tres años. Las medidas terapéuticas de utilidad son: antiinflamatorios no esteroídeos, inyecciones de glucocorticoide, fisioterapia y la movilización temprana del brazo.

Bibliografía Harrison 15°Edición. Págs. 2282-2283.

Bibliografía Harrison 15 ed, pág 2358

Perfil: Exámenes e Imágenes Pelviespondilopatías seronegativas-Sicl 135.- Hombre 25 años consulta por dolor desde hace un año en la región lumbar baja y glúteos

115

116

SECCION DOS OBSTETRICIA Y GINECOLOGIA

REVISADO POR OFEM DR RODERICK WALTON

117

118

119

OBSTETRICIA Y GINECOLOGIA

Perfil: Patología maligna del endometrio-SiCl

paredes vaginales con formación de exudados blanquecinos. ¿ Cuál de los siguientes agentes etiológicos es más probable ?

20 .- Paciente de 65 años, obesa, sin terapia hormonal de reemplazo, presenta un episodio de metrorragia de una semana de duración. Al examen físico se constata un útero levemente aumentado de tamaño. El resto de la evaluación clínica es normal. ¿ Cual de las siguientes es la primera posibilidad diagnostica ?

A.- Cándida albicans B.- Trichomonas vaginalis C.- Chlamydia trachomatis D.- Gardnerella vaginalis E.- Mobilunco Sp.

A.- Cáncer de endometrio B.- Cáncer de cuello uterino. C.- Atrofia genital avanzada D.- Atrofia endometrial E.- Cáncer de colon

A Cándida albicans tiende a prolifera en ambientes donde las bacterias han sido destruidas. Esta micosis es muy frecuente, sobre todo en pacientes que han sido tratadas con tetraciclina o Azitromicina (respuesta A correcta). Un buen tratamiento sería miconazol en pomada vaginal durante dos semanas. Además debe recordarse que cada vez que se usen antibioticos en una mujer en edad reproductivo o post menopáusica debe advertirse a la paciente sobre este efecto secundario, especialmente si hay factores de riesgo adicionales predisponentes a las micosis

C En una mujer posmenopáusica , la primera causa de una hemorragia genital es la atrofia endometrial. El fundamento de ello es que la atrofia endometrial es producida por la ausencia de estimulación hormonal, además de un proceso genera involutivo del útero que hará que el cuerpo uterino disminuya progresivamente su tamaño, aumentando la proporción relativa del cuello en el tamaño uterino S bien la viñeta da antecedentes como la obesidad ( mayor conversión periférica a hormonas ) esto no cambia el hecho que la mayor frecuencia sigue siendo la atrofia endometrial

Bibliografía Botella, "Tratado de Ginecología" 14" edición, Pág. 825-S27

Perfil: Embarazo ectópico complicado-SiCl Bibliografía Botella, "Tratado de Ginecología" 14" edición, Pág. 825-S27

32.- Mujer de 24 años con 8 semanas de amenorrea , se presenta en el servicio de urgencia con dolor abdominal difuso, permanente. Examen físico: FC 120/min. ,PA 90/50 mmHg , sudorosa, pálida, sin fiebre. Examen ginecológico : sensibilidad a la movilización del cuello. ¿Cuál de los siguientes procedimientos le da con mayor certeza un diagnóstico?

Perfil: Enfermedades de transmisión sexual-SiCl 21.- Mujer de 32 años que hace dos semanas recibió tratamiento por 7 días con tetraciclina, sin actividad sexual. Actualmente con prurito vulvar. Examen Físico : genitales externos eritematosos. Visión por espéculo : enrojecimiento de las

A.- Culdocentesis. B.- Ecografía transvaginal. C.- Subunidad beta-HCG seriada.

120

D.- Laparoscopía. E.- Biopsia endometrial

El cuadro clínico que se describe se corresponde con una corioamnionitis. Debe recordarse que el diagnostico tiene elementos clínicos que se presentan en la viñeta y de laboratorio, también presentes en la viñeta. Hay compromiso fetal por lo que se debe finalizar la gestación lo antes posible, además de realizar tratamiento antibiótico endovenoso a la madre en el parto y postparto. El resto de las opciones se invalidan al no contemplar esta posibilidad.

D El diagnóstico de embarazo ectópico se hace de las siguientes formas, en orden de especificidad decreciente: laparotomía-laparoscopía: procedimiento que además es diagnóstico y terapéutico En la ecografía solo son de apoyo : - presencia de saco gestacional con LCF (+) fuera del útero - Ausencia de saco gestacional con una bhCG de 1500 mUI/mL o más.

Bibliografía Williams, Obstetrics, 21° edition

Todas las otras alternativas nos hablan de hallazgos que nos ayudan al diagnóstico, pero no necesariamente son específicos de esta condición, ya que también pueden verse en casos como cuerpo lúteo hemorrágico y otros diagnósticos diferenciales de embarazo ectópico

Perfil: Control prenatal-PrDe 38.- Primigesta de 26 años con embarazo de 28 semanas por FUR confiable. Examen Físico : normal IMC : 27 Exámenes de laboratorio : Hcto : 35% HB: 11,5 g/lt Colesterol total : 240 mg/dl HDL: 60 mg/Dl Triglicéridos : 175 mg/Dl Fosfatasas alcalinas : 40 ( normal sin embarazo : hasta 30) Glicemia Ayunas : 98 ( n : 70- 100) Glicemia post carga 75 gramos : 120 mg(dl) ¿ Cual de las siguientes es la conducta inmediata a seguir ?

Bibliografía Williams, Obstetrics, 21° edition, chapter 34

Perfil: Parto prematuro-SiCl 33.- Multípara de tres partos prematuros , cursando embarazo de 34 semanas. Consulta por dolor cólico y sensación febril . Examen físico: 38ºC, FC : 120/min. Útero sensible a la palpación. Cuello anterior, blando, delgado Dilatación 1 cm FCF 169 /min. Palpación Abdominal : Feto en cefálica. Se palpan bien partes fetales. EPF : 2600 grs. Exámenes de laboratorio Recuento de blancos 18.500mm3. ¿ Cual de las siguientes es la conducta inmediata seguir ?

A.- Derivar a alto riesgo B.- Indicar restricción de hidratos de carbono C.- Indicar dieta hipograsa D.- Tratar la anemia E.- Continuar con su control regular E Los niveles de hemoglobina y hcto son normales para ese periodo de embarazo El IMC para 27 semanas indica que esta en el peso esperado no esta delgada no con sobrepeso Las fosfatasas alcalinas normalmente se duplican durante el embarazo y el colesterol sube pero se mantiene la proporción colesterol total / colesterol hdl . La glicemia de ayunas y post prandial esta normal. Por tanto este es un embarazo fisiológico que debe seguir en su control habitual

A.- Cultivo vaginal y endocervical B.- Amnioinfusión C.- Inducción del parto. D.- Cultivo de liquido amniótico por punción E.- Prevención de membrana hialina C

121

Bibliografía Expert Panel on Detection, Evaluation, and Treatment of High Blood Cholesterol in Adults. Executive Summary of The Third Report of The National Cholesterol Education Program (NCEP)

por dolor brusco con hipertonía uterina, sufrimiento fetal grave o incluso muerte fetal y hemorragia vaginal externa ausente o escasa (alternativa A incorrecta). El parto prematuro sería antes de las 37 semanas (alternativa B incorrecta). Bibliografía 1.- Current Diagnosis & Treatment Obstetrics & Gynecology, Tenth Edition, 2006, Cáp. 20. Third - Trimester Vaginal Bleeding

Perfil: Metrorragia: placenta previa-SiCl 40.-Mujer con embarazo de 37 semanas. Presenta hemorragia vaginal, con sangre roja fresca, sin dolor, de varios días de evolución. ¿Cuál de los siguientes diagnósticos sospecharía en primer lugar?

Perfil: Embarazo ectópico complicado-SiCl

A.- Desprendimiento prematuro de placenta B.- Parto prematuro C.- Placenta previa D.- Rotura uterina E.- Rotura de vasa previa

41.- Mujer de 28 años portadora de DIU presenta amenorrea de 2 meses. Refiere leve dolor hipogástrico. Examen físico: tacto vaginoabdominal doloroso. Ecografía transvaginal : DIU correctamente situado, resto de la exploración impresiona normal. ¿Cuál de los siguientes procedimientos se debe realizar a continuación?

C La placenta previa es la causa más frecuente de hemorragia vaginal del tercer trimestre, y cursa con un cuadro lento con abundante hemorragia de color rojo, sin dolor espontáneo. con abdomen blando (alternativa C correcta). La rotura uterina es una causa rara de hemorragia del tercer trimestre, que debemos sospechar cuando existen cicatrices previas. Puede producir sensibilidad abdominal con incremente de la irritabilidad uterina y escaso sangrado, progresando a medida que la ruptura avanza llegando manifestarse como dolor abdominal agudo (que cesa momentáneamente para luego hacerse constante) acompañado de taquicardia, hipotensión, hiperventilación y sangrado vaginal inconstante, con franco compromiso del feto (SFA) (alternativa D incorrecta). La rotura de vasa previa se manifiesta por sangrado vaginal escaso coincidiendo con la rotura de la bolsa y sufrimiento fetal agudo (respuesta E incorrecta). El desprendimiento prematuro de placenta normo inserta tendrá hallazgos de acuerdo al grado de separación. En un 30% de los casos los desprendimientos son pequeños y producen pocos síntomas. Separaciones grandes se manifiestan

A.- Determinación de b-HCG. B.- Punción del saco de Douglas. C.- Laparoscopía. D.- Histerosalpingografía. E.- Retirar DIU. A Ante una mujer en edad fértil con amenorrea, lo primero es pensar la posibilidad de una gestación. La presencia del DIU y el dolor pueden despistamos hacia una enfermedad inflamatoria pélvica, pero primero habría que descartar un embarazo (respuesta C incorrecta). Ante la sospecha de gestación ectópica y ecografía dudosa hemos de determinar la B-HCG por radioinmunoanálisis (respuesta A correcta). Si el resultado fuese negativo descartaría la gestación y deberíamos reevaluar el caso. Si se confirmase la sospecha, la culdocentesis (segunda opción ofrecida) sería el siguiente paso (respuesta B incorrecta). Bibliografía

122

González Merlo. "Obstetricia"4° Edición. Págs. 336-339.

adecuado

para

lograr

efecto

anovulatorio?

A.- Estrógeno 15 mg más progestágeno B.- Estrógeno 20 mg más progestágeno C.- Estrógeno 30 mg mas progestágeno D.- Progestágeno puro E.- DIU T de cobre

Perfil: Enfermedades de transmisión sexual-SiCl 45.- Mujer de 32 años consulta por flujo genital de mal olor asociado a molestias urinarias. Examen físico: visión por espéculo: leucorrea verde grisácea , cuello eritematoso, ¿Cuál de los siguientes antibióticos es el indicado para manejar su patología?

C Interacciones entre Anticonceptivos orales combinados y otros medicamentos: 1) Disminuyen los efectos de los ACO: - barbitúricos - carbamazepina - fenitoína - felbamato - ketoconazol/itraconazol - griseofulvina - Primidona - Rifampicina - Topiramato 2) Los ACO disminuyen los efectos de las siguientes drogas: - Acetaminofeno, aspirina - Benzodiazepinas Tranquilizantes - metildopa - Anticoagulantes orales - Hipoglicemiantes orales 3) Los ACO aumentan el efecto de las siguientes drogas: - alcohol - benzodiazepinas - aminofilina - antidepresivos - beta-bloqueadores -cafeína - corticoides - teofilina

A.- Doxiciclina B.- Azitromicina C.- Ciprofloxacino D.- Fluconazol E.- Metronidazol E Las características de infección genital por tricomonas, para el diagnóstico son: descarga vaginal de mal olor, color verde-grisáceo, suele haber molestias urinarias, al examen el cuello se encuentra eritematoso (en fresa). El pH vaginal suele ser mayor que 5, a la microscopía de las secreciones se observan tricomonas (microorganismos flagelados) y abundantes leucocitos. El tratamiento es Metronidazol 2 gr. en una dosis o 500 mg 2 veces al día por 7 días. Es una enfermedad de transmisión sexual por lo que la pareja también debe ser tratada. Bibliografía Novak, Gynecology 13° ed. chapter 15

Perfil: Anticoncepción-PrDe

Bibliografía Williams 21° edition, table 58-4, 58-5, 58-6

62.- Mujer de 25 años portadora de una epilepsia que se maneja con carbamazepina y fenitoina. Tuvo su primer parto hace 6 meses y acude a control ginecológico para inicio de terapia anticonceptiva. ¿Cual de las siguientes tratamientos es el más

123

Perfil: Hormonoterapia PrDe Menopausia-SiCl

en

la

menopausia-

D.- Edema corneal. E.- Astigmatismo

63.- Mujer de 49 años que no presentaba regla en los últimos 13 meses. En el último mes a presentado bochornos , mareos , sudoración y perdida de flujo rojo escaso todos los días al levantarse . ¿Cuál de los siguientes fármacos es el indicado para el manejo de sus síntomas ?

D Los fármacos pueden producir diversas manifestaciones oftalmológicas adversas. El edema corneal es típico de los anticonceptivos orales (respuesta D correcta). Muchos fármacos pueden producir glaucoma (bromuro de ipratropio. midriáticos, simpático miméticos, etc.) (respuesta A incorrecta); cataratas (busulfán) . clorambucilo, fenitoínas , corticoides., etc. ) (respuesta B incorrecta); retinopatía (cloro quina, fenotiacinas, etc.) (respuesta C incorrecta); alteración de la visión cromática (barbitúricos, digital, estreptomicina, tiazidas). sulfamidas, etc.) (respuesta E incorrecta),

A.- Estrógenos conjugados B.- Tibolona C.- Veraliprida D.- Progesterona micronizada E.- Anticonceptivos orales C Contraindicación absolutas para el inicio de terapia de reemplazo hormonal son: - embarazo - hemorragia uterina no diagnosticada - tromboflebitis activa - enfermedad hepática - enfermedad de vesícula actual

Bibliografía Harrison 5ª Edición. Cuadro 71-2. Pág. 514

Perfil: Enfermedad del trofoblasto - SiCl 80.- En la exploración de una paciente que ha expulsado una mola hidatidiforme, se comprueba la existencia en ambos ovarios de grandes quistes teca- luteínicos. La conducta más apropiada en este situación es: A.- Histerectomía + anexectomía bilateral. B.- Gestágenos a altas dosis. C.- Evacuación por punción bajo control ecográfico. D.- Estrógenos balanceados con gestágenos. E.- Esperar la regresión espontánea.

relativas: - historia de cáncer de mama - historia de tromboflebitis Bibliografía Novak, Ginecology, 12° edition table 29.2

Perfil: Anticoncepción-PrDe 71.-Mujer de 32 años toma anticonceptivos orales. Sin antecedentes de trastornos visuales previo al uso del fármaco . Ahora refiere que ve mal, sobre todo por la noche, pues las luces aparecen difuminadas y con un halo azulado. ¿ Cual de los siguientes trastornos relacionados con los anticonceptivos orales mas probablemente presenta ?

E

Perfil: Mioma uterino- SiCl 81.- Mujer de 46 años que al control ginecológico sano se le descubre un mioma de pequeño tamaño y asintomático. ¿Cuál es la conducta más apropiada?

A.- Glaucoma. B.- Cataratas. C.- Retinopatía.

124

A.Histerectomía por vía abdominal B.Histerectomía por vía vaginal C.- No hacer nada, ya que resolverá espontáneamente en la menopausia D.- Revisión cada 6 meses para evaluar el tamaño y la aparición de síntomas E.- Tratamiento con testosterona.

preservar fertilidad, rechazo a histerectomía, contraindicación a la cirugía, preparación para histerectomía vaginal .El uso de testosterona no está contemplado en el tratamiento médico (alternativa E incorrecta) Bibliografía 1.- Current Diagnosis & Treatment Obstetrics & Gynecology, Tenth Edition Chapter 39 2.- Berek & Novak's Gynecology, 14th Edition 2006. Cáp. 22 Págs. 480 – 481

D La elección del tratamiento depende de muchos factores: de los síntomas, ubicación del leiomioma, la edad del paciente, paridad, deseo fertilidad, presencia o no de embarazo, salud general, etc. . Los leiomiomas asintomáticos son usualmente manejados en forma expectante, estando indicado la observación, seguimiento y reexaminación periódica para controlar tumores que no están creciendo (alternativa C incorrecta) Ya que en general con el cese de la función ovárica los leiomiomas tienden a disminuir de tamaño, hay algunos pocos que crecen, y / o se vuelven sintomáticos, situación que hay que valorar cada 3-6 meses para proceder a su extirpación cuando sea necesario. (Alternativa D correcta) La cirugía no está indicada en leiomiomas asintomáticos, reservándose para situaciones específicas como: (alternativas A y B incorrectas). - Sangrado anormal que genere anemia con mala respuesta al tratamiento médico. - Dolor pélvico crónico asociado a dispareunia, dismenorrea, sensación de presión abdominal baja, - Dolor agudo como una torsión de un mioma pedunculado o prolapsado. - Síntomas o repercusiones urinarias derivadas de la obstrucción. - Infertilidad donde los leiomiomas son el único hallazgo anormal, - Aborto recurrente asociado a distorsión de cavidad endometrial - Agrandamiento uterino que genere fenómenos compresivos.

Perfil: Patología cervical maligna-SiCl 86.- Paciente de 38 años, nuligesta que busca embarazarse. Exámenes : Papanicolau lesión escamosa de alto grado. Colposcopía lesión acetoblanca extensa del exocérvix que se introduce por el canal endocervical. Biopsia NIE III con canal endocervical positivo. ¿Cuál de las siguientes es la conducta adecuada?: A.- Indicar conización B.- Indicar histerectomía C.- Indicar radioterapia D.- Tratar con 5 fluoracilo tópico E.- Indicar crioterapia A Pregunta sencilla sobre el cáncer de cérvix, su estadificación y tratamiento. Para, conocer el tratamiento del cáncer. de cérvix debes conocer el estadio. Nos presentan un tumor pequeño dentro de los límites del IA (5x7 mm), por lo que el tratamiento correcto sería la realización de histerectomía simple. Sin embargo, debes darte cuenta de que es una mujer joven, con deseos de descendencia, por lo que podemos ser conservadores realizando conización cervical y control posterior (opción 4). Bibliografía Manual CTO 4a Ed. Ginecología y Obstetricia: Pág. 19.

El tratamiento médico también se indica si hay síntomas o infertilidad. Contempla el uso de análogos de GnRh en mujeres con deseo de

125

D Esta pregunta se puede responder de forma directa por las características de la paciente. En efecto esta mujer de edad fértil, con dismenorrea y dispareunia nos orienta de forma primaria a endometriosis. También las alteraciones menstruales son muy indicativas de esta patología. Las opciones C y E son fácilmente descartables puesto que al existir otros hallazgos clínicos, no se puede admitir que sea una dismenorrea funcional ni una torsión ovárica En cuanto a la EIP, a pesar de que el síntoma más frecuente es el dolor, los demás hallazgos no son tan sugerentes. Finalmente la insuficiencia luteinita es una alteración que se ocasiona por el déficit de secreción de progesterona por el cuerpo lúteo, lo que ocasionará un déficit de la transformación secretora endometrial. En la clínica destaca la esterilidad y también pueden producirse spotting intermenstruales. Sin embargo, la paciente no tiene ningún tipo de dolor y por lo demás se encuentra asintomática.

Perfil: Parto prematuro - SIC 87.- Mujer de 17 años con embarazo de 26 semanas que consulta por sensación febril y dolor lumbar. Examen físico : T 39ºC puño percusión positiva a derecha . Exámenes de laboratorio: Recuento de glóbulos blancos 16.000 mm3 con desviación a izquierda. Además de hospitalizar a la paciente ¿Cuál de los siguientes es la conducta inmediata más adecuada? A.- Indicar corticoides B.- Solicitar ecografía renal C.- Indicar Analgésicos D.- Solicitar ecografía obstétrica E.- Indicar antibióticos endovenosos E Esta paciente presenta una pielonefritis, complicación médica bastante frecuente en la gestación. El tratamiento es antibiótico endovenoso, hasta conseguir un mínimo de 48 horas sin fiebre. Luego, dependiendo del estado general de la paciente puede concluir el tratamiento oral en su domicilio. El primer diagnóstico diferencial es con el cólico nefrítico, en el que no se presenta fiebre ni signos analíticos de infección.

Bibliografía: Manual CTO 3a Ed., Ginecología y Obstetricia 8, 9 González Merlo 7a Ed., Págs. 517-518.

Perfil: Retardo del crecimiento intrauterino-SiCl 99.- Mujer cursando embarazo de 32 semanas en la cual se realiza ecotomografía que informa DBP bajo percentil 10 , abdomen en percentil 15 y fémur en percentil 8 . ILA 10. Placenta normo inserta con un grosor de 38 mm. ¿Cual de las siguientes es la mejor interpretación de estos hallazgos ecográficos?

Perfil: Endometriosis- SiCl 98.- Mujer de 24 años, nuligesta, sexualmente activa desde hace 2 años, sin anticoncepción. Presenta desde hace un año dismenorrea severa, dispareunia y sangrado vaginal intermenstrual. ¿Cuál de los siguientes es el diagnóstico más probable?

A.B.C.D.E.-

A.- Insuficiencia de fase lútea B.- Enfermedad inflamatoria pélvica. C.- Torsión ovárica peri menstrual. D.- Endometriosis. E.- Infertilidad secundaria

Insuficiencia placentaria crónica Malformación fetal renal. Displasia esquelética Infección connatal Aneuploidía

D Se trata de un retraso del crecimiento simétrico, lo que sugiere que sea debido a alteraciones

126

cromosómicas, malformaciones congénitas o secuelas de infecciones víricas del primer trimestre, lo que orienta mas a una infección connatal es el hecho de tener una placenta inflamada. ( grosor mayor al esperado)

Bibliografía González-Merlo. "Obstetricia" 4ª Edición, Págs. 272-274.

Bibliografía González Merlo. "Obstetricia." 4a edición, Págs. 664-665. (bibliografía dada en documento preparación MIR). estos subtipos de RCIU no salen el Williams 22 ed. ni en Currents 10 edición.

Perfil: Aborto espontáneo-SiCl 108.- Mujer de 31 años cursando embarazo de 18 semanas. Se presenta al servicio de urgencia por contracciones uterinas dolorosas y sangrado vaginal. Examen físico: Especuloscopía :el orificio cervical interno se encuentra con una dilatación de 3 cm. Ecografía obstétrica :ausencia de latidos y feto en posición transversa. ¿Cuál de las siguientes es la conducta más adecuada?

Perfil: Evaluación anteparto - Proc 100.- Embarazada de 40 semanas en trabajo de parto a la cual se le aplica analgesia epidural lumbar con bupivacaína. A los 10 minutos de administrada la analgesia se comprueba hipotensión que revierte con administración de líquidos y 5 mg. de adrenalina. ¿Cuál de las siguientes medidas habría prevenido más efectivamente este efecto secundario?

A.- Legrado uterino. B.- Analgesia endovenosa. C.- Peridural. D.- Antibióticos. E.- Tocólisis . B Estamos frente a un caso de aborto: interrupción del embarazo antes de la semana 20 (o con un feto < 500 gr.) En este caso se trata de un aborto inevitable: ruptura de membranas en presencia de dilatación cervical. Lo más probable es que en cuento comiencen las contracciones uterinas, el feto sea expulsado, de no ser así puede desarrollarse una infección. Si la pérdida de fluido se acompaña de fiebre, dolor o sangrado, el aborto de considera inevitable y el útero debe ser vaciado.

A.- Interrogar a la paciente por antecedentes de hipersensibilidad a anestésicos locales. B.- Agregar adrenalina al bolo anestésico. C.- Expandir volumen previamente. D.- Evitar la deambulación durante el trabajo de parto. E.- Comprobar que la aguja de inyección no se encontraba en e! espacio intradural. C Uno de los efectos secundarios más frecuentes de la analgesia epidural lumbar es la hipotensión materna secundaria a simpatectomía médica que produce el anestésico local. Es poco importante para la madre pero puede ser grave para el feto. Por eso se debe perfundir previamente como mínimo 500 mi de Rínger lactato pura aumentar la volemia y compensar la vasoplejia. El útero debe mantenerse desplazado de la línea media mediante el decúbito lateral semilateral para evitar el síndrome de hipotensión supina (respuesta C correcta).

Bibliografía Williams 21° edition, chapter 33

Perfil: Síndrome hipertensivo del embarazo- SiCl 116.- Primigesta de 36 años, 34 semana de embarazo por FUR. Examen Físico : PA : 150/100 mmHg en dos tomas separadas mediando reposo, proteinuria aislada positiva y edema generalizado. Súbitamente inicia un cuadro de

127

cefalea y trastornos visuales. ¿Cual de los siguientes medicamentos debe indicar en forma inmediata a este paciente?

valores ascendentes implica afectación fetal. El peak sistólico valora la posibilidad de anemia . La indicación es de esperar la madurez fetal y extraer el feto; la madurez viene marcada por la presencia de fosfatidil glicerol en líquido amniótico, hecho que se ha confirmado. Por ello el siguiente paso es finalizar el embarazo.

A.- Nifedipino B.- Hidralazina. C.- Alfametildopa. D.- Sulfato de magnesio. E.- Labetalol.

Bibliografía Williams

D En el caso de esta paciente debemos prevenir la aparición de una eclampsia, para lo que el único medicamento útil de esta lista es el sulfato de magnesio. El resto son hipotensores que se utilizan principalmente para disminuir los valores de la tensión arterial diastólica en las crisis hipertensivas.

21°

edition

Perfil: Control prenatal- PrDe 128.- Mujer de 29 años que en screening ecográfico obstétrico de las 11-14 semanas se encuentra una moderada dilatación ambos uréteres maternos. ¿ Cuál de la siguientes es la mejor interpretación de este hallazgo ?

Bibliografía Williams 21° edition

A.- Pielonefritis crónica asintomática B.- Cálculo uretral no diagnosticado C.- Urolitiasis bilateral D.- Reflujo ureteral no diagnosticado E.- Cambios fisiológicos propios del embarazo

Perfil: Prevención de enfermedad hemolítica perinatal - PrDe 119.- Multípara de tres cursando embarazo de 34 semanas. Rh negativo sensibilizada con valores de peak sistólico en arteria cerebral media en rango de anemia moderada. Se realizó una amniocentesis que determina madurez fetal ( fosfatidilglicerol presente) y cordocentesis que reveló anemia moderada. ¿Cuál de las siguientes es la conducta más adecuada en este paciente?

E La dilatación ureteral es un hallazgo fisiológico del embarazo, Las litiasis serian sintomáticas. Una mujer con reflujo tendría historia clínica de ITU a repetición al igual que la pielonefritis crónica Bibliografía González.-Merlo. "Obstetricia" 4ª Edición. Págs. 127-128.

A.- Nueva evaluación en una semana. B.- Derivar para seguimiento hospitalizada C.- Indicar corticoides para inducir la madurez fetal. D.- Derivar para interrumpir el embarazo E.- Indicar IgG contra Ac anti D a la madre

Perfil: Metrorragia, Desprendimiento previo de placenta normo inserta (DPPNI)- SiCl

D En un feto cuya madre está isoinmunizada, el seguimiento supone determinaciones seriadas del test de Coombs indirecto, que cuando presenta

130.- Mujer en tercer presenta una hemorragia con mal estado general útero tiene

128

trimestre de embarazo genital escasa y oscura, e importante dolor. El carácter leñoso.

¿Cuál de los siguientes es el diagnóstico mas probable?:

D.- Cáncer de endometrio E.- Terapia de reemplazo estrogénica

A.- Placenta previa. B.- Rotura uterina. C.- DPPNI. D.- Embarazo ectópico. E.- Aborto.

C Causas de sangrado uterino en la postmenopausia: 1) Atrofia endometrial: 60-80% 2) Terapia de reemplazo hormonal con estrógenos: 15-25 % 3) Pólipo endometrial: 2-12 % 4) Hiperplasia endometrial: 5-10 % 5) Cáncer de endometrio 10%

C El desprendimiento prematuro de placenta es la principal causa de hemorragia en el tercer trimestre de embarazo. Cursa con dolor abdominal súbito, hemorragia genital escasa y de color oscuro; puede presentar anemia y shock desproporcionado a la hemorragia visible. Es característica la hipertonía uterina. En un 50% existen signos de toxemia asociados. (respuesta C correcta). Otras causas importantes de hemorragia del tercer trimestre: placenta previa (respuesta A incorrecta), con hemorragia genital abundante y roja paralela al estado general de la paciente ; rotura uterina (respuesta B incorrecta), con hemorragia escasa roja y característicamente con atonía uterina.

La atrofia endometrial es la causa más común de sangrado en la post menopausia, sin embargo es importante siempre descartar el cáncer de endometrio Bibliografía Novak, Gynecology 13° edition, table 30.3

Perfil: Proceso inflamatorio pelviano- SiCl 137.- Paciente de 35 años que presenta dismenorrea secundaria e hipermenorrea en el último año. Menarquia a los 12 años. Método anticonceptivo : Ogino Nauss hace 2 años. FUR hace 35 días. Examen Físico : T: 38,2 ºC. Dolor de todo el hemiabdomen inferior que se acentúa con la tos. Mama : dolor la palpación de cuadrante supero externo bilateral, resto normal. ¿ Cuál de los siguientes cuadros es el más probable?

Bibliografía Merlo 6º edición, Pág. 471

Perfil: Metrorragia- SiCl Patología benigna del endometrio (hiperplasia, pólipos) - SiCl

A.- Proceso inflamatorio pelviano. B.- Hiperprolactinemia C.- Embarazo inicial D.- Mioma submucoso E.- Endometriosis

133.- Mujer de 62 años ,fumadora de 10 paquetes año, posmenopáusica, sin terapia de reemplazo hormonal que presenta hemorragia uterina. Examen físico: IMC 20 , PA 120/70 mmHg, genitales con pelo ralo y lacio encanecido, visión con espéculo dificultada por sequedad vaginal, cuello sano con sangrado positivo a través de cuello en escasa cantidad. ¿Cuál de las siguientes es la causa más probable de su sangrado? A.- Pólipo endometrial B.- Hiperplasia endometrial C.- Atrofia endometrial

A 35 días de amenorrea es normal el ser usuaria de método Ogino por 2 años da seguridad del método además de informar de la relaciones sexuales. las molestia mamarias son irrevalentes en el contexto del cuadro clínico Tiene fiebre e irritación peritoneal Bibliografía

129

Williams

21°

edition

151.- Paciente de 21 años con amenorrea secundaria. Exámenes : niveles plasmáticos de LH y FSH inferiores a 10 mUI/ml., prolactina y TSH normales. Prueba de progesterona negativa y prueba de estrógenos mas progesterona positiva. ¿Cuál de los siguientes es el diagnóstico más probable? A.- Síndrome de ovario poliquístico B.- Falla ovárica autoinmune C.- Tumor hipotalámico o hipofisiario D.- Déficit congénito de hormona liberadora de gonadotropinas E.- Disgenesia gonadal

Perfil: Control prenatal- PrDe 142.- Primigesta de 32 años cursando embarazo de 11 semanas. Antecedente de alergia a la penicilina. Exámenes : orina completa bacterias +++ y urocultivo Escherichia Coli mas de 100.000 colonias /ml. ¿ Cual de las siguientes es la conducta a seguir? A.- Repetir urocultivo B.- Tratar con cefradina. C.- Tratar con cotrimoxazol . D:- Tratar con nitrofurantoína E.- Repetir examen de orina completo

C. El caso clínico nos aporta todos los datos necesarios para llegar al diagnóstico etiológico de la amenorrea secundaria que presenta la paciente. La opción 1 (SOP) la descartamos al no responder al test de supresión con gestágenos. Las opciones 2 y 5 las podemos descartar ya que, al ser de origen ovárico, tendrían las gonadotropinas elevadas. La respuesta 4, al ser un déficit congénito, produciría una amenorrea primaria, no secundaria. Así pues, la respuesta correcta es la 3.

D A Incorrecta porque SIEMPRE debemos tratar una bacteriuria asintomática en una mujer embarazada. B Nos dicen claramente en el enunciado que la paciente es ALERGICA A PENICILINA, y por tanto a fármacos beta - lactámicos, no debemos administrar CEFALOSPORINAS. C Existen dudas, sobre su indicación en el embarazo, pero parece que varios textos coinciden en su prohibición por el riesgo de hepatoxicidad fetal y Kernicterus (fundamentalmente en el tercer trimestre) D Creemos que el ministerio dará esta respuesta como correcta pero hay textos que contraindican su uso en el embarazo. E En este caso no hay duda, las QUINOLONAS están absolutamente contraindicadas en el embarazo, porque alteran el cartílago de crecimiento.

Bibliografía: Manual CTO 4a Ed. Ginecología y Obstetricia: Pág. 6.

Perfil : Amenorrea-SiCl 157.- Mujer de 17 años en amenorrea secundaria, relata que desde la adolescencia sus menstruaciones nunca fueron regulares. Examen físico: Estatura 159 cm Peso 70 Kg. . Vello facial supralabial y de mentón , vello en zona intermamaria. ¿Cuál de los siguientes exámenes solicitaría para continuar su estudio?

Bibliografía Guía Perinatal Pagina 308 Infección urinaria en el embarazo Manual CTO 4a Ed. Capitulo 2.3 2 Ed. Goodman and Gilman 10 Ed. Harrison 15 Ed.

A.- Subunidad beta HCG B.- T.R.H. C.- 17 beta estradiol D.- Testosterona libre E.- Progesterona

Perfil : Amenorrea-SiCl

130

D La viñeta define a la paciente como amenorreica y no como atraso menstrual o sea da la pista de que esta con un ciclo irregular secundario a patología El síndrome del ovario poliquístico (Stein -Leventhal) se ha descrito con la triada clásica de amenorrea, hirsutismo y esterilidad aunque, en realidad, los síndromes de anovulación crónica por señales de retroalimentación inadecuadas constituyen un grupo heterogéneo de trastornos con gran variabilidad clínica y bioquímica. De forma característica, los niveles de testosterona libre están elevados (respuesta D correcta) , también se encuentra alterada la relación LH/FSH pero no se encuentra como alternativa de la viñeta Aunque los pacientes pueden presentar amenorrea primaria o secundaria, un rasgo común es que todos están clínicamente bien estrogenizadas. con abundante moco cervical al examen por lo cual el nivel de 17 beta estradiol es normal. No es embarazo por la historia ( amenorrea anovulatoria). y la progesterona no sirve en el diagnóstico

otros dos. Diabetes gestacional es la diabetes que se inicia o pesquisa por primera vez durante la gestación. En este grupo se distinguen 2 categorías clínicas de diferente severidad, pronostico y tratamiento. a. gestacional propiamente tal (clase A de white) , se manifiesta en el 2do o 3er trimestre del embarazo. se diagnostica por una PTOG prueba de tolerancia a la glucosa alterada o por 2 glicemias de ayuno elevadas. Es metabólicamente estable y de mejor pronostico perinatal. las normas técnicas del ministerio distinguen aquí 2 situaciones clínicas : -disminución de la tolerancia a la glucosa o intolerancia a los hidratos de carbono : glicemias post carga entro 140-199mg/dl. -Diabetes gestacional propiamente tal : glicemias >=200mg/dl a las 2 hrs. post carga. b.- pre gestacional no diagnosticada previamente, se manifiesta precozmente en el embarazo (1º y 2º trimestre). Presenta hiperglicemia de ayuno y post prandiales elevadas en mayor cuantía (>200mg/dl). metabólicamente inestable y es de mayor riesgo perinatal. El tratamiento de la DG se basa en una dieta y el autocontrol de la glucemia (respuesta A correcta, respuesta E incorrecta). La insulina sólo estará indicada cuando con una dieta adecuada no mantengamos un perfil glicémico normal (respuesta C incorrecta).

Bibliografía Manual Merck. 9ª edición, Págs. 1995 y 1998.

Perfil Diabetes y embarazo-SiCl 170.- Mujer de 25 años cursando embarazo de 24 semanas IMC: 29. Dentro de los exámenes de rutina se presenta con una PTGO de 90mg/dl basal y 150 mg/dl a las dos horas ¿Cuál de los siguientes es la conducta más adecuada? A.- Tratamiento dietético estricto B.- Catalogar como intolerancia a la glucosa C.- Indicar insulina D.- Repetir PTGO al cabo de 3 semanas, E.- Control habitual del embarazo.

Bibliografía http://www.cedip.cl/Guias/Guia2003/capitulo16.s wf

A La diabetes gestacional no produce en general síntomas por lo que se debe investigar su presencia en todas las embarazadas. El momento ideal para el estudio es el segundo trimestre (2428 semanas), pero también puede hacerse en los

131

132

SECCION TRES

PEDIATRIA

REVISADO POR DR PEDRO AGUILAR SRTA EVA ARAYA SR MANUEL ROJAS SR MARCOS PEÑAILILLO

133

134

135

136

137

138

PEDIATRÍA Perfil: Desarrollo psicomotor SiCl Respuesta correcta B La tríada sintomática típica de la enterocolitis necrotizante consiste en: distensión abdominal, heces mucosanguinolientas y cuadro suboclusivo, siendo el más común signo de presentación la distensión abdominal, que puede acompañarse por vómitos y residuo gástrico. Es un hallazgo altamente sugerente de enterocolitis necrotizante la neumatosis intestinal y aire en el sistema portal. Los otros diagnósticos no cursan con las características del cuadro descrito.

1.- Un niño camina bien solo y sube escaleras ayudado, habla unas 10 palabras sin hacer frases y se alimenta parcialmente. Puede hacer una torre de 4 cubos. ¿Qué edad debería tener? A.- 12 meses B.- 18 meses C.- 24 meses D.- 36 meses E.- 48 meses

Bibliografía Current Pediatric Diagnosis & Treatment 18th edition, 2007 Capítulo 1. The Newborn Infant and Capítulo 20. Gastrointestinal Tract Nelson “Textbook of Pediatrics”, 17ed Págs. Capítulo 91 - Digestive System Disorders Tabla 91.2-1

Respuesta correcta B 12 meses : camina ayudado, dice algunas palabras y ayuda a vestirse 18 meses : camina bien solo, sube escaleras ayudado, pasa más de una página del libro al mismo tiempo, habla unas 10 palabras, estira juguetes de un hilo, se alimenta solo parcialmente. Los logros descritos deben estar a los 18 meses.

Perfil: Malformaciones congénitas frecuentesSiCl-URG

Bibliografía The Merck Manual of Diagnosis and Therapy, Sección 19. Pediatrics. Capítulo 256 Nelson: Textbook of Pediatrics, 17 Ed. 2004

Perfil URG

Recién

nacido

de

19.- Escolar de 8 años, que nació con una CIA y CIV, se encuentra cursando por quinta vez una neumonía basal izquierda. El examen de elección para confirmar la hipótesis diagnostica es:

pretérmino-SiClA.- Broncoscopía B.- Radiografía tórax C.- TAC Pulmonar D.- Ecografía de tórax E.- Ecocardiografía

Enterocolitis necrotizante -SiCl-URG 6.- Recién nacido pretérmino pequeño para la edad gestacional al nacer y Apgar 3 al minuto 5 a los 5 minutos y pH de cordón 7,2. Comenzó hace 3 horas con irritabilidad y rechazo alimentario. Tras esto presenta un cuadro ssuboclusivo con distensión abdominal, heces mucosanguinolientas y aspecto séptico. Radiografía de abdomen revela neumatosis de la pared abdominal y aire en sistema portal. La radiografía de tórax es normal. ¿ Cuál de los siguientes es el diagnóstico mas probable ?

Respuesta correcta C Entre las causas de neumonía recurrente, en un mismo sitio, están las malformaciones pulmonares, como por ejemplo: malformación adenomatosa quística y secuestro pulmonar. También se debe considerar un cuerpo extraño y otras (inmunodeficiencias, diskinesia ciliar, fibrosis quística, etc.), pero destaca en este caso la recurrencia en el lóbulo inferior izquierdo. El mejor examen para detectar una malformación pulmonar el la TAC pulmonar. Los otros exámenes son complementarios en caso de: cuerpo extraño (broncoscopía), cardiopatía (ecocardiograma), neumonía complicada cerca de

A.- Agenesia anorrectal B.- Enterocolitis necrotizante C.- Invaginación intestinal D.- Perforación de divertículo de Meckel E.- Vólvulo intestinal

139

usualmente bajos (90% casos), recuperándose entre 6 a 8 semanas, los de C4 están normales y CH50 está deprimido (respuesta A incorrecta).

la pared o diafragma o malformación pulmonar (ecografía) Bibliografía: Nelson: Textbook of Pediatrics, 17 Ed. 2004; Enfoque Clínico de las Enfermedades Respiratorias del niño, Ediciones Universidad Católica de Chile, 1ª Edición 2007.

Perfil:

Bibliografía: Nelson “Textbook of Pediatrics”, 17ed Págs. 1740 - 1741 Current Pediatric Diagnosis & Treatment 18th edition Chapter 22. Kidney & Urinary Tract Harrison’s Principles of Internal Medicine 16ed Págs. 1680 – 1681

Perfil: Shock anafiláctico-SiCl-URG

Síndrome nefrítico-SiCl-URG

29.- Niño de 8 años presentó hace dos semanas una faringoamigdalitis que no fue tratada. Consulta con hipertensión arterial, oliguria y edema. Examen de Orina : hematuria macroscópica. ¿ Cual de los siguientes exámenes es característico y apoya el diagnóstico ?

57.- Escolar que recibe en urgencia una dosis de Penicilina sódica por un foco amigdaliano purulento. A los pocos minutos refiere disnea, disfonía, tos seca, rubicundez generalizada y prurito extenso e intenso. ¿ Cuál de los siguientes es el diagnóstico mas probable ?

A.- Complemento sérico elevado B.- Ausencia de proteinuria C.- Creatininemia normal D.- Hipokalemia E.- Antiestreptolisina 0 positivo

A.- Escarlatina aguda B.- Reacción anafiláctica a PNC C.- Laringitis aguda a estreptococo D.- Laringotraqueobronquitis infecciosa E.- Urticaria aguda

Respuesta correcta E La viñeta clínica hace referencia a una Glomerulonefritis aguda post estreptocócica (GNA). La presentación clásica es un síndrome nefrítico florido asociada a una insuficiencia renal aguda oligúrica, aunque en la mayoría de los casos resulta ser una manifestación leve, teniendo a menudo una creatinina levemente elevada al inicio del cuadro (respuesta C incorrecta). La respuesta D es incorrecta dado que es la hiperkalemia la asociada a la falla renal aguda. El sedimento de orina es nefrítico con glóbulos rojos dismórficos, cilindros hemáticos, leucocitos y proteinuria subnefrótica. (respuesta B incorrecta) El antecedente de la infección puede ser aún evidente, haberse resuelto en forma espontánea o por antibióticos, por lo que muchas veces la infección debe ser demostrada por medio de cultivos (faríngeos o de piel) o por serología. La mayoría de los pacientes (alrededor del 90%) tiene anticuerpos circulantes tales como antiestreptolisina O (respuesta E correcta), antidesoxirribonuclease B (anti-DNAsa B), entre otros. Los niveles de C3 en sangre están

Respuesta correcta B Escarlatina: exantema maculopapular extenso, prurito +-, sin signos laríngeos ni tos. No aparece inmediatamente post-PNC. Laringitis: estreptococo no es agente común de laringitis, generalmente viral: parainfluenza. Poco común Haemophilus influenzae (este es causal de epiglotitis). Rapidez del cuadro y signos descritos son clásicos de la reacción anafiláctica (respuesta correcta B). La urticaria no se asocia a signología laríngea. Bibliografía Nelson: Textbook of Pediatrics, 17 Ed. 2004

Cardiopatías congénitas frecuentes-SiCl-URG 65.- Niño de 18 meses que hace 12 horas sufrió un episodio de atoro con un juguete no apto para personas de su edad. Ahora se encuentra inquieto y presenta tos paroxística. ¿Cuál de los siguientes exámenes deberá solicitar

140

El estridor de la Laringomalacia es de tipo inspiratorio, de tono alto, aparece después del parto o algunos días o semanas después, tiende a aumentar desde RN hacia el 2º ó 3er mes, se estabiliza y luego disminuye en intensidad. En asma no hay estridor inspiratorio desde RN. La laringitis aguda se presenta habitualmente después de los 6 meses y cursa con pródromo viral, disfonía, tos perruna y estridor predominantemente inspiratorio, rudo, áspero. Traqueobronquitis tiene fundamentalmente tos intensa, sin estridor inspiratorio del tipo laringomalacia.

primero para reforzar la sospecha de obstrucción bronquial? A.- Fibrobroncoscopía. B.- Radiografías de Tórax en inspiración y espiración. C.- Tomografía computarizada. D.- Broncografía con contraste. E.- Broncoscopía rígida. Respuesta correcta B En las vías aéreas de los niños se alojan con cierta frecuencia cuerpos extraños. Después de los síntomas iniciales (síndrome de penetración), ocasionalmente no recordado o consignado, puede haber un intervalo asintomático, que puede durar horas a semanas. Si la obstrucción es total, el aire de la porción distal se reabsorbe, dejando una atelectasia. Pero si la obstrucción es parcial (tipo válvula) aparece hiperinsuflación obstructiva. Las diferencias radiográficas entre los dos pulmones se acentúan en espiración, al espirar mejor el pulmón sin cuerpo extraño, mientras que el lado afectado se mantiene hiperinsuflado (signo del bamboleo del mediastino). Los hallazgos auscultatorios pueden ser variados. El resto de los examenes descritos podrían ser útiles para la detección del cuerpo extraño, pero no son los iniciales. El primer examen a realizar es la radiografía de tórax AP y lateral, y si el paciente colabora idealmente una AP en inspiración y espiración (respuesta B correcta)

Bibliografía Enfoque Clínico de las Enfermedades Respiratorias del niño, Ediciones Universidad Católica de Chile, 1ª Edición 2007.

Perfil: Hemorragia digestiva-SiCl-URG 67.- Niño de 2 años es traído a la consulta por presentar al finalizar las deposiciones sangre roja fresca. La madre está angustiada ya que tanto ella como el padre han sido operados de hemorroidectomía, recientemente. La analítica indica moderada anemia ferropénica. ¿ Cuál de los siguientes es el diagnóstico más probable ? A.- Fisura anal. B.- Divertículo de Meckel. C.- Enfermedad de Hirschprung. D.- Hemorroides grado I. E.- Defecto de rotación intestinal.

Bibliografía Nelson: Textbook of Pediatrics, 17 Ed. 2004 Enfermedades Respiratorias del niño, Ediciones UC, 2006

Perfil:

Respuesta correcta B La persistencia de la parte proximal del conducto onfalomesentérico en forma de divertículo de Meckel puede ser totalmente asintomática o cursar con patología abdominal en forma de clínica hemorrágica (40%), oclusiva (30%) o infecciosa (20%). Se puede presentar como hemorragia masiva o con anemia por perdidas pequeñas y recurrentes de sangre por heces. La fisura anal se asocia a constipación, dolor al defecar, con escasa estrías o gotas de sangre al defecar o después de ello. Los defectos de rotación intestinal cursan como obstrucción intestinal, con vómitos, dolor y distensión abdominal. Hirschprung se asocia a constipación crónica más que a hemorragia digestiva, salvo cuando está en el contexto de una enterocolitis.

Estridor congénito-SiCl-URG

66.- Varón de un meses, con estridor inspiratorio intermitente, sin fiebre, ni tos. Resto del examen físico normal ¿ Cuál de los siguientes es el diagnóstico mas probable ? A.- Fístula traqueobronquial B.- Laringomalacia. C.- Asma. D.- Laringitis subglótica. E.- Traqueobronquitis Respuesta correcta B

141

¿ Cuál de los siguientes es el diagnóstico mas probable ?

Es infrecuente las hemorroides a esta edad como causa de rectorragia, habitualmente predomina una historia de constipación crónica.

A. Hernia umbilical congénita B. Onfalocele C. Gastrosquisis D. Diástasis de los rectos E. Hernia de la línea media congénita

Bibliografía Compendio de Pediatría. M, Cruz, Pág. 84 Nelson: Textbook of Pediatrics, 17 Ed. 2004

Perfil:

Respuesta correcta B La incidencia del onfalocele es de 1/5000 mientras que la incidencia de la gastrosquisis es de 1/20.000 El contenido está formado siempre por asas intestinales y a veces por otros fragmentos intestinales o bien el hígado, en la línea media, en relación al ombligo (respuesta B correcta). Suele aparecer como la única malformación visible externamente en un 80% de los casos, o bien forma parte de los siguientes síndromes con malformaciones visibles en el nacimiento como son la Pentalogía de Cantrell o el síndrome de Beek-wuh-Wiedemann. Las malformaciones internas asociadas al onfalocele son las cardíacas (las más graves), y las digestivas (las más frecuentes). El onfalocele se asocia siempre en mayor o menor grado a una disminución del tamaño de la cavidad abdominal que no ha recibido el estímulo y presión necesaria de las asas exteriorizadas. En la gastrosquisis la emergencia de las vísceras es lateral al ombligo y no cubierta por peritoneo. La hernia umbilical está cubierta por piel y emerge desde el ombligo, pequeña habitualmente. Diástasis de los rectos tiene piel, sin evidencia de vísceras a través de membranas. Las hernias de la línea media no se presentan como el caso descrito, son pequeñas, frecuentemente dolorosas, se palpa anillo herniario en la línea media.

Dolor abdominal recurrente-SiCl-URG

70.- Lactante de dos meses que presenta llanto agudo, en crisis, desde hace 20 días, síntomas motores y heces normales para su edad. ¿ Cual de los siguientes es el diagnóstico mas probable ? A.- Gastroenteritis aguda. B.- Intolerancia a la lactosa. C.- Cólico del lactante. D.- Otitis media aguda. E.- Hernia inguinal. Respuesta correcta C El cólico del lactante aparece en el primer trimestre de vida. Su etiología y fisiopatología son desconocidas. El diagnóstico es fundamentalmente clínico caracterizado por llanto de intensidad variable, de predominio vespertino, irritabilidad y deposiciones normales, lo que descarta la primera alternativa (gastroenteritis aguda). La intolerancia lactosa se da en lactantes con lecha de vaca, habitualmente en el contexto de una diarrea aguda. La otitis media aguda está dentro de los diagnósticos diferenciales de llanto inconsolable del lactante, pero no cursa como llanto recurrente en crisis. La hernia inguinal puede dar dolor pero es evidente en el examen físico.

Bibliografía Cruz, 8° edición, Pág. 212 Nelson: Textbook of Pediatrics, 17 Ed. 2004

Bibliografía Nelson: Textbook of Pediatrics, 17 Ed. 2004 Perfil:

Perfil: SiCl

Obstrucción intestinal-SiCl-URG Abdomen agudo-SiCl-URG

89.- Niño de 7 meses consulta por decaimiento, palidez y periódicamente fases de agitación con llanto intenso. Examen Físico : presenta vómitos biliosos. Tacto rectal : sangre en deposiciones. Abdomen blando y se palpa una masa no dolorosa y móvil. Ecografía con una imagen en donut o rosca, diagnosticándose invaginación íleo-

Malformaciones congénitas frecuentes

76.- Un RN nace con protrusión de vísceras abdominales (hígado, estómago e intestino) en la región umbilical, línea media, con una cubierta indemne de tipo membrana transparente delgada .

142

D.- Diabetes insípida. E.- Intoxicación por monóxido de carbono.

ileal. ¿Cuál de los siguientes es la conducta inmediata a seguir?:

Respuesta correcta B La insuficiencia suprarrenal cursa habitualmente de forma gradual, pero puede presentarse de forma brusca con deterioro rápido, hipotensión, hipoglucemia, hiponatremia e hiperpotasemia. La recuperación resulta llamativa tras la administración de corticoides endovenosos. No es característica de la Insuficiencia cardiaca el debut brusco asociado a trastornes electrolíticos luego de vómitos. Las diabetes cursan con colapso circulatorio que responde mejor a volumen. La intoxicación por CO no tiene el patrón clínico descrito, determina hipoxia por falla del transporte de O2 más que por colapso circulatorio.

A.- Indicar reducción hidrostática a 90 cmH2O. B.- Observar por 24 horas C.- Solicitar Colonoscopía D.- Repetir Ecografía abdominal con reducción manual. E.- Derivar para cirugía Respuesta correcta E La reducción con presión hidrostática retrógrada da magníficos resultados si se utiliza con indicaciones estrictas y por personal entrenado. El porcentaje de reducción es del 80%. Se realiza en la sala de ecografía procurando que el niño esté tranquilo. Se introduce una sonda de Foley por vía rectal hinchando el globo en la ampolla rectal y se coloca la bolsa con el líquido a una altura de 90 cm del paciente. Se introduce líquido por gravedad en el colon controlando su progresión en todo momento mediante ecografía. Hay cinco contraindicaciones:

Bibliografía Nelson: Textbook of Pediatrics, 17 Ed. 2004

Perfil: 1. Invaginación de más de 24 horas. 2. Invaginación recidivante. 3. Más de tres intentos de reducción. 4. Si se sospecha causa orgánica. 5. Invaginación íleo-ileal (respuesta A incorrecta).

Vasculitis-SiCl-URG

111.- Varón de 4 años tiene lesiones purpúricas palpables, simétricas, de 3 días de evolución en las extremidades inferiores. Examenes Laboratorio : Hemoglobina: 10 g/dl; Glóbulos Blancos 16.500/mm3; Plaquetas : 240.000/mm3 VHS de 45 mm/hora. ¿ Cuál de las siguientes es la etiología mas probable ?

Estará indicado el tratamiento quirúrgico (respuesta E correcta) con laparotomía en fosa ilíaca derecha y reducción del intestino invaginado mediante expresión retrógrada suave de! mismo.

A. Maltrato infantil. B. Púrpura de Schoenlein Henoch. C. Enfermedad de Kawasaki. D. Meningococcemia. E. Enfermedad de Von Willebrand.

Bibliografía Cruz, Tratado de Pediatría 8° edición, Pág. 1001.

Respuesta correcta B La evaluación de un niño con púrpura y petequias de aparición aguda debe incluir un recuento plaquetario para descartar una trombopenia. Una cifra de plaquetas normales sugiere vasculitis de pequeños vasos o alteraciones funcionales plaquetarias . Es característica del Schoenlein Henoch un exantema purpúrico palpable sin plaquetopenia, asociado a dolor abdominal, articular y poca fiebre o afebril. Las otras condiciones descritas se asocian a petequias o equimosis, por tanto, lesiones hemorrágicas planas.

Perfil: Shock-SiCl-URG 97.Niño que de forma brusca presenta hipotensión, vómitos y colapso cardiovascular que no responde a la administración de drogas vasoactivas. Examenes de Laboratorio : hiponatremia e hiperpotasemia. ¿ Cuál de los siguientes es el diagnóstico mas probable? A.- Insuficiencia cardiaca. B.- Insuficiencia suprarrenal. C.- Diabetes juvenil.

143

Físico : eritema multiforme y adenopatía cervical anterior, dolorosas a la presión. Rx de tórax : focos de consolidación dispersos sobre todo en lóbulos inferiores, de tipo neumonía atípica. ¿ Cuál de los siguientes agentes etiológicos es el más probable ?

Bibliografía Nelson: Textbook of Pediatrics, 17 Ed. 2004

Perfil: Síndrome nefrítico-SiCl-URG A.- Neumococo B.- Legionella pneumophila C.- Mycoplasma pneumoniae D.- Coxiella burnetti E.- Clamidia psitacci

131.- Un niño que como único antecedente destacable refiere haber presentado una faringitis aguda febril hace dos semanas. Acude a urgencia por edema palpebral, hipertensión arterial y hematuria macroscópica. El análisis de orina muestra cilindros hemáticos y discreta proteinuria. ¿Cuál de los siguientes es el tratamiento a aplicar?

Respuesta correcta C Mycoplasma pneumoniae puede causar neumonía, traqueobronquitis y faringitis siendo la causa más frecuente de neumonía en niños desde los 5 hasta los 15 años de edad. La denominación de “neumonía primaria atípica” se corresponde clínicamente con un cuadro de neumonitis de comienzo subagudo, de distribución habitualmente no lobular, esputo escaso y no purulento y en el que a menudo hay disociación entre las imágenes radiológicas y los escasos signos físicos. Los síntomas más importantes son la tos y la fiebre, que prácticamente no faltan nunca. Con frecuencia hay cefalea y presencia concomitante de síntomas correspondientes a rinitis, faringits, miringitis o traqueobronquitis. La exploración general del ap22:43:51arato respiratorio puede revelar rinorrea, faringe eritematosa, acompañada a veces de adenopatías cervicales anteriores dolorosas a la presión e hiperemia timpánica. Radiológicamente suele observarse un infiltrado heterogéneo y poco denso, con aspecto de vidrio deslustrado, que tiende a estar situado cerca del hilio, sobre todo en los lóbulos inferiores; a menudo los infiltrados afectan a varios lóbulos, de ambos pulmones. Entre las complicaciones pulmonares destaca la extensión del infiltrado a varios lóbulos determinando insuficiencia respiratoria; esta complicación es más frecuente en paciente con drepanocitosis y en niños con Síndrome de Down. En los pacientes con anemia drepanocítica, la neumonía por M. pneumoniae puede asociarse a gangrena digital. Los otros agentes son muy infrecuentes en niños, salvo neumococo que da neumonía típica sin exantema.

a) Plasmaféresis y tratamiento de sostén b) Interferon y corticoides c) Corticoides + Inmunosupresores en función de la respuesta inmune d) Corrección del equilibrio hidroeléctrolitico + antibióticos + reposos en cama e) Azatriopina, diuréticos e IECA Respuesta correcta D La glomerulonefritis post- estresptocócica es la causa más frecuente de glomerulonefritis en al infancia. Sigue a algunas infecciones respiratorias altas o infecciones cutáneas por estreptococos B hemolíticos del grupo A. No se conoce ningún tratamiento específico, sino que se aconsejan medidas de soporte (corrección del equilibrio hidroeléctrolitico, reposo en cama durante la fase inflamatoria aguda y tratamiento antibiótico para erradicar la infección). En la mayoría de los casos la enfermedad cura espontáneamente, aunque en los adultos el pronóstico es menos favorable y es más probable que persistan alteraciones urinarias. Los demás tratamientos descritos no se consideran en el tratamiento habitual de la GNA postestreptocócica (se pueden emplear en otras nefropatías) Bibliografía Nelson: Textbook of Pediatrics, 17 Ed. 2004 Manual Washington de Terapéutica Medica 10ª Edición Pág. 226

Bibliografía Red Book 2006, 27 edición Banfi: Enfermedades Infecciosa en Pediatría, 3ª Edición, 2004 Enfoque Clínico de las Enfermedades

Perfil: Bronconeumonía bacteriana-SiCl 132.- Niño de 10 años con síndrome de Down, consulta por aparición de fiebre de 38 ºC, junto con tos seca, congestión nasal y disnea. Examen

144

www.minsal.cl Calendario de vacunas

Respiratorias del niño, Ediciones Universidad Católica de Chile, 1ª Edición 2007.

Perfil: Meningitis bacteriana-SiCl Perfil:

Faringo-amigdalitis-SiCl 146.- Recién nacido nacido prematuro por una rotura prematura de membrana, tratado con un antibiótico por meningitis por Haemophilus influenzae presenta distensión abdominal, cianosis, colapso vasomotor y shock. ¿Cuál de los siguientes antibióticos es mas probable sea responsable de este cuadro?

136.- Paciente presenta bruscamente un cuadro caracterizado por fiebre, odinofagia e infiltración faríngea difusa con vesículas de color blanco grisáceo rodeada por una aréolea roja en pilares anteriores y paladar blando , sin otra sintomatología. ¿ Cuál de los siguientes es el diagnóstico mas probable ?

A.- Rifampicina B.- Eritromicina. C.- Tetraciclina. D.- Cloranfenicol. E.- Metronidazol.

A.- Primoinfección herpética B.- Herpangina C.- Rinosinusitis D.-Infección primaria por VIH E.- Mononucleosis infecciosa

Respuesta correcta D El cloranfenicol sigue siendo una de las opciones terapéuticas ante una meningitis por H. Influenzae, aunque no se indica en recién nacidos por el riesgo de producir el síndrome gris del recién nacido, objeto de este caso clínico. Se debe a la inmadurez del sistema hepático de la glucuronil transferasa y a la deficiente excreción renal del fármaco (respuesta D correcta).

Respuesta correcta B La localización característica de la herpangina es la mucosa bucal, lingual y faríngea, se caracteriza por presentar un inicio brusco, con odinofagia, fiebre y vesículas en la orofaringe, por lo general en menores de 4 años en épocas de verano. Herpes tiende a extenderse a toda la mucosa oral. Los otros agentes no se presentan como lo descrito en la viñeta.

Bibliografía Harrison 15'Edición. Págs. 1040. Nelson: Textbook of Pediatrics, 17 Ed. 2004

Bibliografía Nelson: Textbook of Pediatrics, 17 Ed. 2004 Banfi: Enfermedades Infecciosa en Pediatría, 3ª Edición, 2004. Red Book 2006, 27 edición

Perfil:

Cuerpo extraño en la vía aérea-SiCl-URG

147.- Niño de 2 años, sano previamente, presenta rinorrea purulenta de larga evolución por fosa nasal derecha, afebril, con mala ventilación nasal. ¿ Cuál de los siguientes es el diagnóstico mas probable ? A.- Rinitis alérgica B.- Cuerpo extraño intranasal. C.- Atresia de coanas D.- Sinusitis etmoidal E- Rinitis vasomotora

Perfil: Programa ampliado de inmunizaciones (PAI)-PrDe 141.- La vacuna BCG se administra al : A.- Recién nacido B.- Niño de 1 año C.- Niño de 18 meses D.- Niño de 4 años E.- Escolar de 1º básico

Respuesta correcta B Respuesta correcta A De acuerdo al PAI vigente en Chile, sólo se administra al RN. Ya no se coloca en 1º básico.

Rinorrea purulenta unilateral debe hacer pensar siempre en un cuerpo extraño nasal. La atresia de coana da síntomas desde RN. Sinusitis etmoidal da otro patrón, tipo celulitis,

Bibliografía:

145

Infección de Embarazo

febril, sin rinorrea unilateral. Rinitis alérgica y vasomotora, habitualmente bilaterales, no supuran y si lo hacen por una sobreinfección bacteriana, no presentan rinorrea unilateral de larga data.

Durante

el

La infección de citomegalovirus (CMV) es una infección viral común que por lo general no produce síntomas en los niños y adultos infectados. El CMV pertenece a la familia del virus del herpes y es más común en los niños pequeños. Aproximadamente entre el 50 y el 80 por ciento de las mujeres en edad fértil ya han tenido CMV antes y, por lo general, no necesitan preocuparse al respecto durante el embarazo.1 Los niños pequeños con CMV pueden transmitirlo a otros miembros susceptibles de la familia y a las personas a cargo de su cuidado. Cuando una mujer embarazada se infecta con el virus, puede transmitirlo al feto. En una minoría de los casos, la infección provoca enfermedades graves, incapacidades permanentes e incluso la muerte del recién nacido. ¿Cómo se transmite el CMV? El CMV puede transmitirse de una persona a otra a través del contacto con fluidos corporales infectados, como la saliva, la orina, la sangre y las mucosidades. También puede transmitirse sexualmente o a través de productos sanguíneos infectados. En ocasiones, los adultos infectados desarrollan una enfermedad similar a la mononucleosis, que puede incluir síntomas como dolor de garganta, fiebre, dolores en el cuerpo y cansancio. En personas con SIDA y otros trastornos del sistema inmunológico y en personas trasplantadas, el CMV puede provocar enfermedades graves como la neumonía e infecciones oculares que pueden causar la pérdida de la vista. La mujer puede transmitir el CMV a su bebé antes del nacimiento, durante el parto o al amamantarlo. No obstante, es poco común que los bebés contagiados con CMV durante el parto o la lactancia sufran problemas graves relacionados con el virus.1,2 ¿Con qué frecuencia se producen las infecciones de CMV en los recién nacidos? El CMV es la infección congénita (presente desde el nacimiento) más común en los EE.UU.1 Cada año cerca del uno por ciento de todos los recién nacidos, o 40.000 bebés, contrae la infección.2 ¿De qué manera afecta al bebé la infección congénita de CMV? Afortunadamente, la mayoría de los bebés no se ve afectado por el virus. Cerca del 85 al 90 por ciento de los bebés infectados con CMV no presenta síntomas al nacer.1 No obstante, aproximadamente un 10 por ciento de ellos desarrolla una o más anomalías neurológicas,

Bibliografía Nelson: Textbook of Pediatrics, 17 Ed. 2004

Perfil:

Citomegalovirus

TORCH-SiCl-URG

148.- Recién nacido pequeño para le edad gestacional, con microcefalia, presenta calcificaciones peri-ventriculares. Se detecta hepatoesplenomegalia , ictericia, trombocitopenia y rash petequial. ¿ Cuál de los siguientes agentes etiológicos es la causa mas probable ? A.-Rubéola. B.- Herpes. C.- Sífilis. D.- Toxoplasma. E.- Citomegalovirus. Respuesta correcta E Infección congénita por CMV: las manifestaciones de presentación más frecuentes son petequias, hepatoesplenomegalia e ictericia (60 a 80 %). En un 30-50 % de enfermos se encuentran microcefalia con o sin calcificaciones cerebrales, retraso del crecimiento intrauterino y retinitis. Las calcificaciones periventriculares son características de CMV, a diferencia de las calcificaciones de las Toxoplasmosis que son más difusas. Los otros agentes descritos tienen otras expresiones clínicas diferenciales. Rubéola: trombocitopenia, hepatoesplenomegalia, poliadenia, bajo peso al nacer, encefalitis, anemia hemolítica, fontanela amplia anterior, cardiopatía congénita, lesiones oculares, cataratas, sordera, compromiso SNC. Herpes: puede dar infección diseminada (fundamentalmente hígado y pulmones), localizada en piel, ojos y boca o localizada en SNC. No tiene como característica calcificaciones periventriculares. Sífilis: puede ser asintomática o presentar hallazgos de TORCH descritos, sumado a osteocondritis , rinitis mucosanguinolenta, sin calcificaciones intracerebrales.

146

de fiebre. Clostridium perfringens tiene un período de incubación más prolongado y no es común que se asocie a vómitos. C. botulinum, si bien puede dar diarrea, tiene un período de incubación mayor a 12 hrs. (mínimo 6 hrs.). Los mismos tiempos son válidos para salmonellas. E. coli tiene período de incubación de 10 hrs. a 6 días.

como retraso mental, problemas de aprendizaje, o pérdida auditiva o de la vista, por lo general durante los primeros años de vida.1 La infección congénita de CMV es una de las causas principales de la pérdida auditiva en los niños.3 Cerca del 10 al 15 por ciento de los bebés infectados manifiesta síntomas de CMV al nacer.1 Estos síntomas pueden incluir un bazo o hígado más grandes de lo normal, ictericia (color amarillento en la piel y en los ojos) y una erupción cutánea característica de la infección. Hasta un 20 por ciento de estos bebés muere y cerca del 90 por ciento de los que sobreviven sufre defectos neurológicos graves, como retraso mental.3,4 Actualmente no existe ningún tratamiento eficaz para el CMV congénito. No obstante, los médicos están investigando una nueva droga antiviral llamada ganciclovir (utilizada para tratar adultos con SIDA u otros problemas del sistema inmunológico que tienen infecciones oculares relacionadas con el CMV) para averiguar si también puede ser eficaz en los bebés que padecen CMV congénito.

Bibliografía Nelson: Textbook of Pediatrics, 17 Ed. 2004 Banfi: Enfermedades Infecciosa en Pediatría, 3ª Edición, 2004. Red Book 2006, 27 edición

Perfil:

153.- Lactante cursa con fiebre, odinofagia, diarrea, edema de extremidades, palidez intensa y anuria. ¿Cuál de los siguientes es el diagnóstico mas probable ?

Bibliografía Cria capítulos 32,33,37,47,50. http://www.nacersano.org/centro/9388_9927.asp Nelson: Textbook of Pediatrics, 17 Ed. 2004 Banfi: Enfermedades Infecciosa en Pediatría, 3ª Edición, 2004. Red Book 2006, 27 edición

Perfil:

Síndrome hemolítico urémico-SiCl-URG

a) Síndrome nefrótico b) Síndrome nefrítico c) Síndrome hemolítico urémico d) Shock séptico e) Pielonefritis aguda Respuesta correcta C La presentación clásica del Síndrome Hemolítico Urémico es: anemia intensa, falla renal con edema y oligo-anuria, en un lactante que cursó previamente con un pródromo de tipo infección digestiva o respiratoria. Los otros diagnósticos pueden llevar a falla renal, pero la primera hipótesis a plantear es el SHU. El shock séptico puede llegar a falla renal, sin tener el grado de anemia observado en el SHU (anemia hemolítica microangiopática). El síndrome nefrótico puede ser gatillado por una infección respiratoria o digestiva pero no tendrá anemia microangiopática. La pielonefritis aguda aislada (sin daño renal crónico previo) no debiera dar edema ni anemia hemolítica.

Diarrea aguda-SiCl

149.- Cuatro horas después de acudir a una cena, llegan a urgencia 8 niños con nauseas, vómitos y dolores abdominales. ¿ Cual de los siguientes es el agente etiológico mas probable ? a) Salmonella typhimurium b) Estafilococo productor de enterotoxina c) Escherichia coli enterotóxica d) Clostridium botulinum e) Clostridium perfringens Respuesta correcta B Los síntomas clásicos de la intoxicación alimentaria por toxinas estafilocócicas se presentan dentro de pocas horas desde la ingesta, son fundamentalmente: nauseas, vómitos, dolor abdominal y diarrea. Es la causa más común de intoxicación alimentaria. Orientan a este diagnóstico: corto período de incubación (30 minutos a 8 hrs.), brevedad del cuadro y ausencia

Bibliografía Nelson: Textbook of Pediatrics, 17 Ed. 2004

Perfil : Faringo-amigdalitis-SiCl

147

A.- Tetralogía de Fallot. B.- Comunicación interauricular tipo ostium secundum. C.- Enfermedad de Barlow. D.- Comunicación interauricular tipo ostium primum, E.- Comunicación interventricular.

156.- Niño de 6 años y medio que presenta de forma brusca fiebre que se asocia en los días siguientes con inapetencia, disfagia y dolores musculares. Examen físico se observan pequeñas vesículas blancas rodeadas por un halo eritematoso, en número escaso de 5 a 15 no confluyentes a nivel de los pilares anteriores, úvula, y velo del paladar blando. ¿ Cuál de los siguientes agentes etiológicos es el mas probable ?

Respuesta correcta B Es un defecto del tabique interauricular (CIA), que permite el paso de la sangre de la aurícula izquierda a la derecha. Es la cardiopatía congénita más frecuente en el adulto y predomina en las mujeres (relación 2:1 a 3:1). Se distinguen tres tipos de CIA: a) el tipo ostium secundum (90% de los casos), que está situado en la parte central del tabique y puede acompañarse de un prolapso de la válvula mitral (15-20%) b) el tipo ostium primum (5%), que ocupa la parte baja y suele producir insuficiencia valvular mitral y c) el tipo seno venoso (5%), junto a la desembocadura de la vena cava superior, y suele acompañarse de un drenaje venoso pulmonar anómalo parcial. El foramen oval permeable, que permite el paso de un catéter pero no da lugar a cortocircuito, no debe considerarse una CIA. Por lo general la CIA es asintomática hasta que aparece la insuficiencia cardiaca derecha o la fibrilación auricular, alrededor de los 40-50 años, aunque con frecuencia se descubre una historia previa de infecciones bronquiales repetidas. El diagnóstico suele sospecharse porque: a) se ausculta un desdoblamiento amplio y fijo del segundo ruido pulmonar que refleja la prolongación de la sístole ventricular derecha; b) se descubre en la radiografía de tórax la hipervascularidad propia del cortocircuito izquierda derecha o c) se detecta un bloqueo incompleto de rama derecha en el electrocardiograma, que es prácticamente constante (90% de los casos). La ecocardiografía de contraste y el Doppler color permiten visualizar el defecto. El ECG suele mostrar desviación del eje a la derecha y un patrón rSr' en las derivaciones precordiales derechas en el tipo ostium secundum (respuesta B correcta). En el tipo ostium primum el eje se desvía a la izquierda (respuesta D incorrecta). La CIA debe intervenirse cuando es de gran tamaño (Q.p/Q.s >= 1.5-2:1). SECUNDUM -> DERECHA PRIMUM -> IZQUIERDA La tetralogía de Fallot cursa con cianosis (respuesta A incorrecta). La enfermedad de Barlow o prolapso de la válvula

A.- Staphilococus aureus B.- Echovirus C.- Virus parainfluenza D.- Coxsackievirus E.- Virus herpes simple Respuesta correcta D La herpangina es una enfermedad causada típicamente por Coxsackievirus tipo A. Afecta principalmente a niños de 3 a 10 años de edad y se caracteriza por un cuadro clínico consistente con fiebre elevada de aparición brusca, cefalea, disfagia, mialgias, vómitos ocasionales, y aparición de pequeñas vesículas blancas rodeadas por un halo eritematoso, en número de 5 a 15 en pilares anteriores, úvula y velo del paladar blando. cura espontáneamente de 5 a 8 días. El virus parainfluenza no provoca lesiones como las descritas. Staphilococcus aureus no genera lesiones vesiculares, sino que un exudado purulento faringoamigdaliano. Herpes virus produce lesiones vesiculares pero no limitadas a los pilares amigdalianos, sino que se extiende por toda la mucosa oral. Bibliografía Dermatología clínica Ferrandiz, 1 edición Pág. 47 Farreras 14 edición, Pág. 102 Banfi: Enfermedades Infecciosa en Pediatría, 3ª Edición, 2004.

Perfil: URG

Cardiopatías congénitas frecuentes-SiCl-

163.- Un lactante de sexo femenino afectado por una cardiopatía congénita. Examen Físico : sin cianosis, soplo de insuficiencia mitral, plétora pulmonar y ECG : desviación a la derecha del eje del QRS y un patón rSr' en las derivaciones precordiales derechas. ¿ Cuál de los siguientes es el diagnóstico más probable ?

148

176.- Escolar refiere dolor abdominal, artralgias erráticas en codos, rodillas y muñecas, con aparición de púrpura palpable en nalgas y muslos,. Examenes Laboratorio : ausencia de anemia, plaquetas normales, IgA elevada, creatinina normal, proteinuria de 1gr/24 hrs. y hematuria microscópica, ¿ Cual de los siguientes es el diagnóstico mas probable ?

mitral no es una cardiopatía congénita. En el ECG suelen observarse trastornos de la repolarización (ondas T bifásicas o invertidas), en especial en las caras inferior y lateral (respuesta C incorrecta). La comunicación interventricular no cursa con alteraciones significativas del ECG salvo una hipertrofia biventricular (respuesta E incorrecta). Bibliografía Harrison, 16° edición, Pág. 1532-1533, Farreras, 14ª edición, Pág.. 707. Nelson: Textbook of Pediatrics, 17 Ed. 2004

Perfil:

a) Vasculitis tipo PAN b) Lupus eritematoso sistémico c) Enfermedad de Wegener d) Síndrome de Goodspasture e) Síndrome de Schöenlein-Henoch

Diarrea aguda-SiCl

171.- Un preescolar de 3 años, sano previamente, consulta a policlínico por diarrea aguda de carácter familiar, con fiebre y deshidratación severa. En el coprocultivo se aísla una Salmonella enteritidis. ¿Cuál de las siguientes es la conducta terapéutica más apropiada? a) Tratar con cloramfenicol b) Tratar con amoxicilina c) Tratar con ceftriaxona d) Tratar con cotrimoxazol e) Tratar sin antibióticos

Respuesta correcta E El patrón clásico del Schöenlein Henoch es: púrpura palpable, predominantemente en extremidades inferiores y nalgas, dolor abdominal variable, compromiso articular y compromiso renal. Goodpasture presenta compromiso renal (hematuria) y pulmonar (hemoptisis) sin lesiones las púrpuricas observadas en Schoenlein-Henoch. LES puede dar compromiso renal y articular, pero la edad, sexo y compromiso cutáneo alejan el diagnóstico de LES. Vasculitis tipo PAN y Wegener tiene otra clínica.

Respuesta correcta E Son limitadas las indicaciones de antibioticoterapia en diarrea aguda, y la recomendación para infección por Salmonella enteritidis es no tratar con antibiótico, por ser de curso autolimitado y porque el antibiótico puede prolongar el tiempo de eliminación del agente por las deposiciones (prolonga el estado de portador). Distinta es la conducta de Salmonella thipy, la que si requiere tratamiento. Se puede tratar con AB la gastroenteritis aguda por Salmonella en pacientes de riesgo como: menores de 3 meses, inmunodeprimidos, enfermedad intestinal crónica, neoplasias, hemoglobinopatías.

Bibliografía Nelson: Textbook of Pediatrics, 17 Ed. 2004

Bibliografía Red Book 2006, 27º Edición Banfi: Enfermedades Infecciosa en Pediatría, 3ª Edición, 2004.

a) Virus b) Mycoplasma Pneumoniae c) Bordetella Pertussis d) Haemophylus Influenzae e) Bramahela Catarralis

Perfil: URG

Perfil: Bronquitis aguda SiCl 177.- Paciente de 10 años de edad presenta cuadro caracterizado de 5 días de evolución caracterizado por secreciones nasales serosas que luego de 3 días se acompañó de tos seca de inicio gradual, sin expectoraciones, sin fiebre y dolor al pecho retroesternal, graduado como 4/10 , que se intensifica con la tos. ¿ Cuál de los siguientes agentes etiológicos es el mas probable ?

Respuesta correcta A La bronquitis aguda se presenta característicamente como afebril o con baja fiebre, rinitis, tos seca no productiva de inicio gradual y dolor al pecho de baja intensidad que se

Púrpura de Schoenlein-Henoch-SiCl-

149

infección puede adquirirse a cualquier edad. Puede comenzar lentamente con fatiga, indisposición general (malestar), dolor de cabeza y dolor de garganta, que empeora progresivamente, a menudo con inflamación de las amígdalas, cubiertas por un exudado blanco amarillento. También los ganglios linfáticos del cuello se inflaman y se tornan dolorosos. Se puede observar una erupción rosada macular similar a la del sarampión y es más probable si al paciente se le está dando ampicilina o amoxicilina para la infección de garganta. (NO se deben administrar antibióticos sin una prueba positiva para estreptococos.) Los síntomas de la mononucleosis desaparecen gradualmente por sí solos en un período de semanas a meses.

intensifica con la tos. La causa etiológica más común es viral. Mycoplasma puede causar bronquitis aguda, pero tiende a adquirir tos en accesos, cefalea moderada, obstrucción bronquial frecuente, incluso neumonía atípica y otitis ampollar. Bordetella pertussis es poco común a esta edad y tiende a presentarse con tos quintosa, fundamentalmente en lactantes. Los otros agentes bacterianos pueden dar bronquitis, pero predominan los virus cuando la clínica es la descrita. Bibliografía Nelson: Textbook of Pediatrics, 17 Ed. 2004 Enfoque Clínico de las Enfermedades Respiratorias del niño, Ediciones Universidad Católica de Chile, 1ª Edición 2007.

Perfil:

Bibliografía http://www.nlm.nih.gov/medlineplus/spanish/ency /article/000591.htm, Nelson: Textbook of Pediatrics, 17 Ed. 2004 Banfi: Enfermedades Infecciosa en Pediatría, 3ª Edición, 2004. Red Book 2006, 27 edición

Faringo-amigdalitis-SiCl

178.- Niño de 10 años de edad que acude a la consulta por amigdalitis pultácea con adenopatías cervicales. Se pone tratamiento con penicilina a dosis correctas. A las 72 horas acude de nuevo al no experimentar mejoría. ¿ Cuál de los siguientes agentes etiológicos es el mas probable? A.B.C.D.E.-

Perfil:

Bronconeumonía viral-SiCl

179.- Lactante de 5 meses, que consulta por fiebre de 39,5ºC axilar . Examen Físico : conjuntivitis, crepitaciones bibasales, exantema maculopapular eritematoso Examenes Laboratorio : Proteína C Reactiva : 200 mg% ¿ Cual de los siguientes es el agente etiológico mas probable de la neumonía? : A.- Virus Respiratorio Sincicial B.- Adenovirus C.- Bordetella Pertussis D.- Chlamydia pneumoniae E.- Neumococo

virus de Epstein-Barr Estreptococo beta hemolítico grupo A. Neumocitis Carinii Virus Parainfluenzae. Rubéola.

Respuesta correcta A El hecho de que un paciente con garganta estreptocócica no mejore con tratamiento en 48 horas hará sospechar la posibilidad de una mononucleosis infecciosa. Los otros agentes no tienen como característica clínica compromiso amigdaliano febril del tipo Epstein Barr. Mononucleósis infecciosa es una infección viral que causa fiebre, dolor de garganta e inflamación de los ganglios linfáticos, especialmente en el cuello. Usualmente está ligada al virus de EpsteinBarr (EBV), pero también puede ser ocasionada por otros organismos como el citomegalovirus (CMV). Causas, incidencia y factores de riesgo. La mononucleosis a menudo se transmite por medio de la saliva. Aunque esta afección, conocida como la "enfermedad del beso", se presenta con más frecuencia entre los 15 y 17 años de edad, la

Respuesta correcta B Entre las características clínicas que sugieren infección por ADV están: compromiso de tipo neumonía (incluso pleuroneumonía), febril, asociado a exantemas y conjuntivitis, con laboratorio compatible con infección bacteriana. No es característico de VRS, Bordetella, Chlamydia pneumoniae ni Neumococo la aparición de exantemas ni conjuntivitis como las vistas con ADV. Chlamydia trachomatis puede presentarse con la asociación de neumonía afebril del primer trimestre y conjuntivitis.

150

Bibliografía Banfi: Enfermedades Infecciosa en Pediatría, 3ª Edición, 2004. Enfoque Clínico de las Enfermedades Respiratorias del niño, Ediciones Universidad Católica de Chile, 1ª Edición 2007. Red Book 2006, 27 edición

151

152

SECCION CUATRO TEMA UNO CIRUGIA GENERAL Y ANESTESIA

REVISADO POR DR JUAN IGNACIO MONGE SRTA MARIA PAZ DELON

153

154

155

156

157

158

CIRUGÍA GENERAL Y ANESTESIA

ácidos grasos que entran en el colon sean hidroxilados por la flora bacteriana, con lo cual tiene e! efecto de inhibir la absorción y estimular la secreción colónica, con el consiguiente empeoramiento de la diarrea (diarrea esteatorreica). La diferenciación entre diarrea coleriforme y esteatorreíca tiene implicaciones terapéuticas importantes, ya que la primera responde a la fijación intraluminal de los ácidos biliares con colestiramina, mientras que la segunda puede agravarse con la ingesta de este fármaco y mejora en cambio al sustituir la grasa dietética por triglicéridos de cadena media, que son absorbidos de manera directa (respuesta B incorrecta).

Perfil: Síndrome de Intestino corto SiCl 04.- Paciente con antecedentes de resección ileal de 70 cm consulta por diarrea crónica. ¿ Cuál de las siguientes medidas es la mas indicada para tratar este cuadro ? A.- Administración de antibióticos. B.- Dieta pobre en grasas. C.- Administración de vitamina B12. D.- Tratamiento con colestiramina. E.- Dieta pobre en lactosa. Respuesta correcta D Síndrome del intestino corto: cuando se realizan resecciones quirúrgicas amplias (más del 50% de la longitud del intestino) o existen lesiones inflamatorias amplias se reduce de forma notable la superficie absortiva y puede aparecer un cuadro clínico de mal absorción, cuya gravedad dependerá de dos elementos: la longitud de la resección y la zona anatómica resecada. Ésta es importante si se tiene en cuenta la especialización funcional de los distintos segmentos intestinales (absorción de hierro, calcio y folatos en el duodeno y el yeyuno próxima!, y de ácidos biliares y vitamina B12 en el íleon). Si la resección ileal es pequeña (menos de 100 cm). la síntesis hepática de ácidos biliares experimenta un aumento compensatorio de las pérdidas fecales excesivas que permite una concentración y una absorción prácticamente normales de los lípidos. Como contrapartida, en el colon se alcanza una elevada concentración de ácidos biliares, cuyas propiedades catárticas determinan una diarrea secretora colónica (diarrea coleriforme), que puede eliminarse con colestiramina (respuesta D correcta).

Bibliografía: 1. Farreras, 14° edición, Pág. 226. 2. Harrison. 15" edición, Pág. 1948.

Perfil: Obstrucción intestinal SiCl 13.- Mujer de 75 años, con dolor abdominal difuso tipo cólico, asociado a vómitos alimentarios y biliosos. Se diagnostica cólico biliar y se indica tratamiento analgésico. Posteriormente aparece febrícula, distensión abdominal y disminución del peristaltismo. Radiografía de abdomen ; asas de intestino delgado dilatadas con distribución en patrón "en escalera" y niveles hidroaéreos, pequeña imagen radiopaca en cuadrante inferior derecho y aire en la vía biliar. ¿ Cuál de los siguientes es el diagnostico mas probable ? A.- Apendicitis aguda B.- Colecistitis enfisematosa C.- Colangitis D.- Íleo biliar E.- Trombosis de la vena mesentérica

Si la resección ileal es superior a 100 cm se produce mal absorción importante de ácidos biliares y el aumento de la síntesis hepática no es suficiente para compensar las pérdidas fecales, por lo que disminuyen sus niveles intraluminales y se compromete la solubilización micelar de las grasas. En estas condiciones predomina la esteatorrea (secundaria a maldigestión por deficiencia de ácidos biliares y a mal absorción por la longitud resecada del íleon) y no es tan importante el efecto catártico de los ácidos biliares sobre el colon por estar diluidos en un gran volumen intraluminal de bajo pH. Por otra parte, la mal absorción de grasa determina que los

Respuesta correcta D El íleo biliar es la obstrucción mecánica del tracto gastrointestinal por un cálculo grande que pasó a través de una fístula enterobiliar espontánea (77% casos entre vía biliar y duodeno) luego de una colecistitis aguda y necrosis asociada que perfora la pared vesicular. Ocurre más frecuentemente en mayores de 70 años y explica menos del 1% de las

159

baja presión resultan seguros. Puede ser realizado el enema baritado una semana o más después del episodio agudo si el paciente se ha recuperado (alternativa B incorrecta). • La rectosigmoidoscopía rígida usualmente no pasa a través de la unión rectosigmoidea por la angulación y disminución del lumen por la inflamación. Pueden verse signos inflamatorios adyacentes y descarga purulenta (alternativa D incorrecta). • La colonoscopía debiera evitarse en el momento agudo, pero si debería diferirse para evaluación de las estructuras y descartar neoplasia (alternativa A incorrecta).

obstrucciones intestinales en general y el 25% en pacientes añosos (sin cirugías y hernias previas). Se presenta con clínica de obstrucción intestinal, con un historia de síntomas asociados a litiasis (dolor HD) en el 50% de los casos, con una Rx que muestra distensión y niveles mostrando en algunas ocasiones areobilia y el cálculo distal a la vesícula (como en la viñeta) que son específicos del íleo biliar. (alternativa D correcta) Bibliografía: 1.- Sabiston Textbook of Surgery 17th, 2004, Chapter 52 - Biliary Tract Pág. 1622

Bibliografía: 1.- Current Surgical Diagnosis & Treatment, 12 ed 2006 Cap. 30. Large Intestine: Diverticular Disease of the Colon. 2.- Sabiston Textbook of Surgery 17th, 2004, Chapter 48 - Colon and Rectum, Pág. 1419

Perfil: Diverticulitis SiCl 28.- Hombre de 60 años refiere sensación febril y dolor en el cuadrante inferior izquierdo, que se irradia a la región suprapúbica, ingle y región lumbar izquierda. Examen físico abdominal : defensa muscular voluntaria y masa palpable sensible en cuadrante inferior izquierdo. ¿Cuál de los siguientes métodos es el más adecuado para confirmar la sospecha diagnóstica?

Perfil: Gastroenterología SiCl

A.- Colonoscopía B..-Enema baritado C.- Radiografía de abdomen D.- Rectosigmoidoscopía E.- TAC con contraste

50.- Mujer de 80 años que presenta melena abundante. Examen Físico : PA: 95/60 mmHg en decúbito y 78/50 mmHg en bipedestación. FC: 140 /min., FR: 20 /min. Hcto: 40%. ¿Cuál de los siguientes es el examen a solicitar?

Respuesta correcta E La viñeta clínica hace referencia a un cuadro de diverticulitis o enfermedad diverticular complicada. Se diagnostica por clínica pero requiere de estudio con imágenes. Los exámenes para realizar el diagnostico; • TAC de abdomen y pelvis con contraste, es el estudio imagenológico inicial y debe ser obtenido temprano en el curso de la enfermedad, se aprecia refuerzo en la grasa pericólico o complicaciones locales. Se ha hecho una clasificación en torno a la observación por TAC de la enfermedad (Hinchey) (alternativa E correcta). • La Rx abdomen simple puede mostrar aire libre abdominal si el divertículo está perforado. Si la inflamación está localizada se apreciará íleo, obstrucción colónica parcial o una masa en cuadrante inferior izquierdo, señales inespecíficas (alternativa C incorrecta) • El enema baritado está contraindicado durante las etapas iniciales del ataque agudo por la eventual perforación y paso de bario al peritoneo, auque el empleo de contrastes solubles en agua y a

A.- Endoscopía digestiva alta. B.- Angiografía. C.- Colonoscopía. D.- Rectosigmoidoscopía. E.- Radiografía de abdomen. Respuesta correcta A Los pacientes con hematoquecia que presentan inestabilidad hemodinámica deben someterse a una endoscopía alta para descartar un proceso hemorrágico de la parte superior del tubo digestivo antes que explorar la parte inferior del mismo (respuesta A correcta). La determinación de la PA y la FC es la mejor manera de empezar el estudio de un paciente con HD. Las hemorragias clínicamente importantes modifican esas constantes al cambiar de postura al paciente. El hematocrito no desciende inmediatamente después de un HD aguda porque el volumen del plasma y de los hematíes disminuye proporcionalmente . Bibliografía: Harrison, 15º edición, Pág. 301 y 302.

160

A.- Indicar Abrasión de la Pleura. B.- Indicar Pleurectomía parietal. C.- Derivar para tubo de drenaje pleural con trampa de agua. D.- Aspiración simple con aguja. E.- Derivar para Ventilación mecánica.

Perfil: Obstrucción intestinal SiCl 53.- Hombre de 70 años con demencia senil, estreñimiento crónico y abuso de laxantes. Presenta náuseas y dolor abdominal en las últimas 24 horas. Radiografía simple de abdomen : gran distensión del colon que se incurva produciendo una imagen en "grano de café". ¿Cuál de los siguientes es el diagnóstico más probable?: A.- Enfermedad diverticular B.- Isquemia mesentérica C.- Pancreatitis D.- Vólvulo del sigmoides E.- Enfermedad de Crohn

Respuesta correcta D El tratamiento del neumotórax espontáneo tiene un doble objetivo: 1.- Reexpandir el pulmón. La reexpansión puede realizarse espontáneamente ya que el aire se reabsorbe. En los neumotórax espontáneos primarios pequeños (menos del 30% de colapso pulmonar) se recomienda la aspiración simple con aguja (Harrison) o la administración de oxigeno y esperar la reabsorción espontánea del aire (respuesta D correcta). En el caso de que el neumotórax sea grande (mayor al 30% de colapso pulmonar) o no se resuelva espontáneamente, será necesario restablecer la presión negativa intrapleural por medio de la aspiración mediante tubo de drenaje bajo sello de agua (respuesta C incorrecta). 2.- Prevenir las recidivas en casos de barotraumas intensos o neumotórax por bulas apicales (20-50% de recidivas) mediante pleurodesis física (abrasión de la pleura) o química, e incluso la pleurectomía parietal (respuesta A y B incorrecta). Por último, y si la situación clínica del paciente lo demanda, podría ser necesaria la ventilación mecánica (respuesta E incorrecta).

Respuesta correcta D El vólvulo del sigmoides es la forma más común de vólvulo en el tracto gastrointestinal y es responsable del 8% de todas las obstrucciones intestinales. El vólvulo del colon sigmoides es particularmente común en personas de edad mayor. Los pacientes se presentan con dolor abdominal, distensión y constipación absoluta, Factores predisponentes incluyen constipación, megacolon y colon con excesiva movilidad además de inactividad y uso de laxantes (ambas condiciones frecuentes en adultos mayores) Los hallazgos en la radiografía simple de abdomen suelen ser diagnósticos hasta en el 6070% de los pacientes. Las paredes del intestino involucrados están edematosas y las paredes contiguas forman una línea blanca y densa en las radiografías. Esta línea está rodeada por el curvo y dilatado lumen lleno de gas, resultando en una estructura con forma de grano de café: este es el signo del grano de café. El enema baritado muestra una estenosis completa, la que asemeja un pico de pájaro

Bibliografía: Farreras, 14º edición, Pág.976 Harrison, 15º edición, Pág. 1773

Perfil: Colonoscopía Proc 90.- Mujer de 70 años que desde hace 4 meses presenta fatiga, decaimiento, escasa pérdida de peso y deposiciones de aspecto normal. Examen físico : palidez de piel y mucosas y masa palpable sensible en hipogastrio. Exámenes : Hcto : 24 %, Hb: 8 mg/dl , VCM: 70 fl ; HbCM; 20 pg . ¿Cuál de las siguientes es la conducta más apropiada en esta paciente? A.- Indicar suplemento de fierro B.- Indicar colonoscopía C.- Solicitar cinética del fierro D.- Repetir hemograma E.- Solicitar test de hemorragia ocultas

Bibliografía: http://www.emedicine.com/radio/topic636.htm; http://www.emedicine.com/emerg/topic931.htm http://radiology.rsnajnls.org/cgi/content/full/216/1 /178/F1

Perfil Broncopulmonar SiCl 83.- Hombre de 30 años sufre un neumotórax espontáneo con un colapso pulmonar del 10%. ¿Cuál de las siguientes es la conducta mas adecuada?:

Respuesta correcta B

161

¿Cuál de los siguientes es el tratamiento más adecuado?

La viñeta hace referencia a un probable cáncer de colon derecho. El colon derecho tiene un lumen mayor, una pared más distensible y el contenido fecal es más líquido habitualmente Dado éstas características, el cáncer de colon puede tener un gran tamaño antes de ser diagnosticado. Los pacientes a menudo consultan por debilidad y fatiga debido a una anemia severa de curso crónico. El hallazgo de anemia microcítica hipocroma siempre pone entre los diagnósticos el cáncer de colon sobretodo considerando el cuadro del paciente. Sangre en las deposiciones pueden no ser evidentes para los pacientes, pero un test de hemorragia oculta en heces puede evidenciarla. Puede haber molestias abdominales vagas en hemiadbomen derecho y las alteraciones en el hábito intestinal y la obstrucción intestinal no son característicos del cáncer del lado derecho. En el 10% de los casos la primera evidencia es una masa palpable al examen físico. Dado que la sospecha es de un cáncer de colon, la conducta más apropiada es la alternativa B. Objetivar hemorragia oculta en heces y suplementación con fierro son buenas conductas iniciales, pero en el contexto de un cáncer de colon no son las mejores. Serían las mejores si se tratara del estudio inicial de una anemia microcítica hipocroma sin una fuente clara (alternativas A, C y E incorrectas). Repetir el hemograma sería adecuado en el contexto que no hubiera correlación con la clínica de la paciente o como evaluación a la respuesta a fierro, lo que no es en este caso (alternativa D incorrecta)

A.- Intervención quirúrgica en las 24 a 48 horas siguientes B.- Indicar colecistotomía de urgencia. C.- Analgésicos hasta que ceda el cuadro y colecistectomía diferida a los 3 meses. D.- Indicar Colangiografía retrógrada endoscópica. E.- Corticoides. Respuesta correcta B Colecistitis aguda enfisematosa: aunque la base del tratamiento de la colecistitis aguda y de sus complicaciones sigue siendo la cirugía, puede ser necesario un período de estabilización en régimen de hospitalización antes de llevar a cabo la colecistectomía. En general está indicado administrar antibioterapia por vía intravenosa a los pacientes con colecistitis aguda grave. El momento adecuado de la intervención quirúrgica en los pacientes con colecistitis aguda depende de su estabilización. La tendencia predominante es la intervención precoz. Probablemente, ¡a colecistectomía o la colecistotomía de urgencia son adecuadas en la mayoría de los enfermos en los que existe la sospecha o la certeza de que se ha producido una complicación de la colecistitis aguda, como empiema, colecistitis enfisematosa o perforación (respuesta B correcta). En la colecistitis aguda no complicada la intervención quirúrgica precoz (en las 24 a 72 horas siguientes al comienzo) es la técnica de elección (respuesta A incorrecta). Les pacientes debilitados o gravemente enfermos con colecistitis deben ser tratados con colecistostomía y colocación de un tubo de drenaje en la vesícula.

Bibliografía: 1.- Current Surgical Diagnosis & Treatment, 12 ed 2006 Cap. 30. Large Intestine: Cancer of the large intestine 2.- Sabiston Textbook of Surgery 17th, 2004, Chapter 48 - Colon and Rectum Págs. 1458

Bibliografía: 1. Harrison, 15" edición, Pág.. 20S3. 2. Farreras, 4a edición, Pág.. 457.

Perfil : Colecistitis Aguda SiCl

Perfil: Paracentesis ExIm.

101.- Mujer de 70 años diabética con náuseas, vomitas y dolor en hipocondrio derecho que se irradia a la zona interescapular. Examen Físico : T 38, 5 °C Palpación abdominal : vesícula biliar distendida y dolorosa. Exámenes : Leucocitos : 17.000 /mm3 y Bilirrubina : 1,3. Rx simple abdomen : gas en el interior de la luz vesicular. Además de administrar antibióticos

134.- Hombre 77 años portador de una cirrosis hepática alcohólica. Desde hace tres semanas con ascitis. A la semana del ingreso por dolor precordial presenta febrícula y dolor abdominal, con disminución del peristaltismo y signos de encefalopatía. ¿ Cuál de las siguientes es la conducta inmediata ?

162

Respuesta correcta D

A.- Practicar paracentesis. B.- Administrar lactulosa. C.- Indicar laparotomía. D.- Administrar neomicina, E.- Administra furosemida .

En un paciente con hemorragia digestiva y con situación de inestabilidad hemodinámica, el volumen intravascular se debe reponer de inmediato (respuesta D correcta). Hay que colocar dos grandes vías intravenosas en venas periféricas de suficiente tamaño. El cateterismo venoso central no ofrece ninguna ventaja sobre una vía periférica adecuada y puede retrasar la perfusión de volumen. La hemoglobina y el hematocrito no siempre reflejan la magnitud de las pérdidas hemáticas agudas (respuesta B incorrecta). La endoscopia diagnóstica precoz no mejora la mortalidad (pero la endoscopia terapéutica reduce la necesidad de cirugía urgente) (respuesta E incorrecta). Hasta que se disponga de sangre se puede administrar suero salino isotónico o solución lactato de Ringer. En algunos casos no queda más remedio que administrar sangre 0- sin cruzar (respuesta A incorrecta).

Respuesta correcta A En todo paciente con ascitis y signos de deterioro clínico hay que realizar tina paracentesis diagnóstica para excluir una peritonitis bacteriana espontánea (PBE) (respuesta A correcta). El diagnóstico se sospecha cuando el líquido ascítico contiene más de 250 neutrófilos/microL y se confirma con cultivo positivo. Los microorganismos más frecuentes son E. Coli y neumococo. Se desaconsejan los amino glucósidos porque pueden precipitar insuficiencia renal, no alcanzan niveles adecuados en el líquido ascítico y se inactivan con el pH ácido (respuesta B incorrecta). La PBE es más frecuente en los pacientes con cirrosis hepática de origen alcohólico. Se caracteriza por fiebre, dolor y distensión abdominal, disminución del peristaltismo y encefalopatía hepática progresiva, aunque la ausencia de esta no excluye la posibilidad del diagnóstico. La laparotomía puede resolver el proceso desencadenante de una peritonitis secundaria (respuesta C incorrecta). La neomicina ayuda a tratar la encefalopatía portosistémica, pero hay que diagnosticar primero la PBE y poner un tratamiento sistémico (cefalosporina de 3a generación o norfloxacino) (respuesta D incorrecta). La furosemida no es responsable de este cuadro (respuesta E incorrecta).

Bibliografía: Manual Washington de Terapéutica Médica. 10"Edición. Págs. 348-350.

Bibliografía: Harrison 15ª Edición, Págs.. 2060-2061. Manual Washington de Terapéutica Médica. 10ª Edición. Págs.. 392-394.

Perfil: Hemorragia digestiva SiCl 164.- Hombre de 65 años presenta hematemesis. Examen Físico : FC 112 /min. y PA 100/50 mmHg. ¿ Cual de las siguientes es la medida inmediata ? A.- Tipificación de la sangre y pruebas cruzadas. B.- Solicitar hematocrito y Hb. C.- Indicar vía venosa central. D.- Reponer volumen E.- Indicar endoscopia

163

164

SECCION CUATRO TEMA DOS TRAUMATOLOGIA

REVISADO POR OFEM SRTA NATALIA LABARCA

165

166

167

TRAUMATOLOGIA

en tercio medio. ¿Cuál de los siguientes es el diagnóstico más probable?

Perfil : Luxaciones SiClDiagnóstico 68.- Paciente de 27 años que sufre una caída sobre el hombro derecho. Examen físico : deformidad del hombro y signo de la "tecla de piano" en la extremidad distal de la clavícula. ¿Cuál de los siguientes es el diagnóstico más probable?

A.- Fractura diafisaria de cúbito y radio. B.- Epifisiolisis de muñeca. C.- Fractura de Smith. D.- Luxo fractura de Monteggia. E.- Fractura de Colles. Alternativa correcta: A Las fracturas de antebrazo se pueden dividir (para términos prácticos de estudio) en fracturas del antebrazo proximal, medio y distal. Dentro de las alternativas señaladas la única que corresponde al tercio medio es la alternativa A (fractura diafisiaria de cúbito y radio). El tratamiento de este tipo de fractura debe ser quirúrgico. La epifisiolisis de muñeca, la fractura de Smith y la fractura de Colles afectan el tercio distal del radio. La fractura de Colles afecta al radio, y en este caso el segmento distal se desplaza hacia radial y dorsal, causando una deformidad en dorso de tenedor y una desviación en bayoneta y es más frecuente en mujeres sobre los 50 años. En la fractura de Smith (Colles invertido) el segmiento distal está angulado hacia delante. La luxofractura de Monteggia se refiere a una fractura cubital con luxación de la cabeza del radio (antebrazo proximal).

A.- Fractura de la cabeza humeral. B.- Disyunción acromio-clavicular. C.- Luxación escápulo-humeral. D.- Fractura de escápula. E.- Fractura de coracoides. Alternativa correcta: B La luxación acromio clavicular es una lesión que afecta con mayor frecuencia a quienes practican deportes de contacto, ciclistas, por lo que son más frecuentes en hombres jóvenes. El mecanismo de producción de este tipo de lesiones es por caídas sobre el hombro, con la extremidad superior en aducción. En este caso se trata de una luxación grado 3, es decir los ligamentos coracoclavicular y acromioclavicular se han roto completamente, por lo que al examen físico se observa el extremo distal de la clavícula sobresaliente. El signo de la tecla se refiere a la protrusión de la clavícula que al presionarla, se desplaza hacia abajo y luego vuelve a la posición inicial, al igual que una tecla de piano. Ninguna de las otras alternativas provoca esta deformidad a nivel de la clavícula, por lo que se descartan automáticamente. La luxación escápulo-humeral produce al examen físico lo que se llama "deformidad en charretera", donde desaparece la redondez del hombro.

Bibliografía: Manual CTO de Traumatología, 4a Edición, pág. 11.

Perfil : Artrosis de rodilla SiClDiagnóstico

Perfil : Fracturas SiClDiagnóstico

112.- Mujer de 75 años con dolor intenso en la rodilla desde hace 2 semanas, sin antecedente de trauma agudo. Examen Físico : IMC = 34. Varo bilateral de rodillas, mínimo derrame articular, movilidad completa y dolorosa de ambas rodillas, no se detectan inestabilidades. ¿Cuál de los siguientes es el diagnóstico más probable?

69.- Niño presenta físico : antebrazo

A.- Meniscopatía. B.- Osteocondritis. C.- Fractura por estrés de meseta tibial. D.- Artrosis de rodilla.

Bibliografía: 1.- Manual CTO de Traumatología, 4a Edición, pág. 18-19. 2.- http://www.emedicine.com/sports/topic3.htm

de 9 años, tras caída de un columpio dolor intenso en antebrazo. Examen impotencia funcional absoluta del y gran deformidad de vértice palmar

168

E.- Artritis reumatoide.

Perfil : Luxaciones SiClDiagnóstico

Alternativa correcta: D En este caso, debemos pensar en una artrosis de rodilla, puesto que la paciente no tiene antecedentes traumáticos, y por su edad y la obesidad que presenta, junto con el genu varo, sabemos que la rodilla está expuesta a un desgaste importante. La osteocondritis y las fracturas de estrés son más frecuentes en pacientes jóvenes con mayor actividad física. La meniscopatía y la artritis reumatoides tienen edades de presentación más tempranas.

173.- Frente a un paciente con enclavijamiento endomedular de tibia cerrado, por una fractura, que a las 24 horas presenta súbitamente dolor intenso en la pierna, resistente a los analgésicos habituales; que se acompaña de aumento de volumen y de dificultad para mover los dedos del pie. ¿Cuál de los siguientes es el diagnóstico más probable? A.- Trombosis venosa profunda. B.- Lesión del ciático común. C.- Síndrome compartimental. D.- Infección precoz de la osteosíntesis. E.- Fractura del pie no diagnosticada.

Bibliografía: Manual CTO de Traumatología, 5a Edición, pág. 47.

Alternativa correcta: C El síndrome compartimental se define como el aumento de presión dentro de los espacios delimitados por las fascias o tabiques aponeuróticos, lo que impide la circulación capilar. Los pulsos distales están conservados, puesto que la circulación general no está afectada. Clínicamente, existe dolor en aumento progresivo, aumento de volumen, aumento de tensión, frialdad y palidez. Al evolucionar el cuadro también pueden presentarse parestesias (compromiso neurológico) Si el cuadro no se resuelve a tiempo, puede llevar a necrosis con pérdida de la extremidad. Es importante recordar que este síndrome se puede producir tanto en fracturas abiertas como cerradas y también en otras patologías (por ejemplo en quemados). Las fracturas que más se asocian al síndrome compartimental son las de tibia, antebrazo y codo, en especial las supracondíleas en niños. La trombosis venosa profunda y la infección se descartan puesto que provocan mayor compromiso del estado general (fiebre) y la lesión del ciático no provoca edema.

Perfil : Lesiones radiculares SiClDiagnóstico 139.- Albañil sufre un accidente, cayéndose desde 6 metros de altura. Al examen Físico dolor lumbar invalidante y déficit de extensión contra gravedad de los dedos del pie derecho. ¿ Cuál de las siguientes es la localización mas probable de la lesión? A.- Raíz L-3. B.- Raíz L-4. C.- Raíz S-1. D.- Raíz L-5. E.- Raíz S-2. Alternativa correcta: D Para localizar una radiculopatía debemos conocer y examinar la función motora afectada, el déficit sensorial y la presencia de reflejos. En la extremidad inferior, las raíces nerviosas que controlan la función motora del pie son L5 y S1. Si la dorsiflexión del pie y la extensión de los dedos están comprometidos, se trata de una lesión de la raíz L5 (corresponde a una hernia L4-L5); el déficit sensitivo corresponde a la cara anterolateral de la pierna, y dorso del pie hasta el primer dedo. Si la raíz L3 estuviera afectada, habría debilidad en la flexión de cadera y la extensión de rodilla; la raíz L4 también controla la extensión de rodilla, mientras que la raíz S1 está involucrada en la flexión plantar del pie.

Bibliografía: Manual CTO de Traumatología, 4a Edición, pág. 6-7.

Bibliografía: Manual CTO de Neurología y Neurocirugía, 5a Edición, págs 75-76.

169

170

SECCION CUATRO TEMA TRES UROLOGIA

REVISADO POR OFEM SRTA NATALIA LABARCA

171

172

173

UROLOGÍA

Bibliografía: 1.-Harrison, 14º edición, pág. 675. 2.-Harrison, 16° edición, pág. 605.

Perfil : Prostatitis SiClDiagnóstico 25. Hombre judío de 28 años, consulta repetidas veces por cistitis. ¿Cuál es la causa más probable?

Perfil : Cancer de Próstata SiClDiagnóstico

A.- Circuncisión B.- Varicocele C.- Hernia inguinal D.- Urolitiasis E.- Prostatitis

84. Hombre de 50 años, al tacto rectal se palpa una próstata aumentada de tamaño de consistencia gomosa. Antígeno prostático : 7 ng/ml . ¿Cuál de las siguientes es la conducta más adecuada para el manejo de este paciente?

Respuesta: E La infección urinaria aguda es infrecuente en pacientes de sexo masculino menores de 50 años. Dentro de las causas de cistitis recidivantes (por el mismo patógeno) se encuentran: infección renal e infección prostática (respuesta E) en el caso de los hombres. La circuncisión (respuesta A) es un factor protector para infecciones urinarias, mientras que el varicocele y la hernia inguinal no tienen ninguna relación (respuestas B y C). La urolitiasis (respuesta D) se presenta en comúnmente acompañada de otra sintomatología.

A. B. C.D.E.-

Indicar biopsia prostática. Control en un año. Repetir antígeno prostático. Realizar un índice PSA libre / PSA total. Indicar antibióticos.

Respuesta Corercta A En este caso, dada la edad del paciente, sospechamos cáncer de próstata por la elevación del antígeno prostático (se deben descartar las otras condiciones que elevan el antígeno). El valor normal del antígeno prostático (PSA) es de 0 a 4 ng/mL, aunque se utiliza un valor de corte superior separado por edades: así para los hombres entre 50 y 59 años, el valor del PSA no debe superar los 3.5 ng/ml. El tacto rectal más sugerente de cáncer de próstata, es un nódulo duro e irregular o un crecimiento nodular de la próstata. Una próstata aumentada globalmente de tamaño es más sugerente de hiperplasia prostática benigna, sin embargo entre un 20 a 25% de los hombres con tacto rectal alterado, presentan cáncer de próstata. Por lo tanto la conducta más apropiada es frente a un tacto rectal alterado, más un PSA elevado (>4 ng/ml): tomar biopsia prostática.

Bibliografía 1. Harrison 16° edición págs. 1890-1891 2. Manual Washington de Terapéutica Médica. 30ª Edición Pag. 310

Perfil : Cáncer de Vejiga SiClDiagnóstico 73.- De las siguientes, señale la molestia de presentación más frecuente en el carcinoma de vejiga: A.- Hematuria macro y microscópica. B.- Disuria. C.- Poliaquiuria y urgencia miccional. D.- Obstrucción ureteral. E.- Dolor pélvico.

Bibliografía: 1.- Harrison 16ª Edición Págs. 610-611 2.http://www.emedicine.com/med/topic3537.htm

Respuesta: A El signo más frecuente en el carcinoma de vejiga es la hematuria que se encuentra hasta en el 80 a 90% de los casos y suele ser consecuencia de un tumor exofítico. La sintomatología irritativa es la segunda manifestación más frecuente de la enfermedad. La obstrucción uretral y el dolor pélvico son mucho menos frecuentes.

Perfil : Urolitiasis SiClDiagnóstico 121.- Una ecotomografía renal muestra una imagen hiperecoica con sombra posterior en la

174

A menudo este tumor se diagnostica ya sea por sus manifestaciones extrarenales (por lo que se le ha llamado el "tumor del internista") o como hallazgo mediante técnicas de imagen (ecografía, TAC, más raramente radiografía simple de abdomen). La manifestación clínica más frecuente al inicio es la hematuria (40% de los casos) y dolor en el flanco (40%), otras manifestaciones son (en orden decreciente de incidencia): reducción de peso, anemia, fiebre, hipertensión, anormalidades en la función hepática, hipercalcemia, eritrocitosis, neuromiopatía, amiloidosis, aumento de la VHS. Estas manifestaciones pueden formar parte de un sd paraneoplásico como por ejemplo: la disfunción hepática no metastásica (sd de Stauffer) y la disfribrinogenemia adquirida. Entre los diagnósticos diferenciales se debe considerar: quistes, neoplasias benignas, pielonefritis o abscesos y otras neoplasias malignas primarias o metástasis, que son menos frecuentes que el adenocarcinoma. En los exámenes radiológicos de la tuberculosis genitourinaria, se observan principalmente calcificaciones y estrecheces uretrales que pudiesen producir hidronefrosis; y no es característica la presencia de alteraciones en la calcemia El tumor de Wilms es el tumor renal más común en edad pediátrica, la edad promedio de diagnóstico es de 3.5 años y su presentación más habitual es por dolor abdominal (la presentación no corresponde al cuadro clínico de la viñeta, por lo que esta alternativa se descarta). Finalmente, el sarcoma es una patología muy poco frecuente en el riñón, aunque este diagnóstico diferencial debe tenerse siempre presente. • valores normales: calcemia: 9-10 mg/dL bilirrubina total: 0.3-1 mg/dL fosfatasas alcalinas: 30-120 U/L SGOT (ASAT) 0-35 U/L

pelvis renal, muy sugestiva de cálculo, que no se visualiza en la radiografía simple de abdomen. ¿Cuál es la composición más probable del cálculo? A.- Oxalato cálcico B.- Fosfato cálcico C.- Estruvita D.- Acido úrico E.- Cistina Respuesta correcta D Los cálculos de sales de calcio (respuestas A y B) son los cálculos más frecuentes (corresponden al 75 a 85% del total), ambos compuestos pueden estar mezclados en el mismo cálculo. Estas alternativas se descartan rápidamente debido a que es sabido que la presencia de calcio es sinónimo de opacidad en la radiografía. Todos los cálculos son radioopacos, los únicos radiolúcidos son los de ácido úrico (respuesta D). Este tipo de litiasis es más frecuente en hombres, tiene un patrón familiar, y la mitad de quienes la sufren están afectados de gota. Bibliografía: Harrison 16ª Edición. Págs. 1885 – 1886

Perfil : Adenocarcinoma renal SiClDiagnóstico 174.- Hombre de 40 años con masa renal en ecotomografía abdominal. Sin antecedentes mórbidos. Examen físico: PA 160/100 mmHg. Exámenes de laboratorio: calcemia 11mg/dl, bilirrubina total 1,4 mg/dl, fosfatasas alcalinas 45 UI/l, SGOT 60 UI/l ¿Cuál de los siguientes es el diagnóstico más probable de la lesión renal? A.Tuberculosis B. Adenocarcinoma C. Sarcoma D. Absceso E. Hamartoma

Bibliografía: 1.- Harrison, 16° edición, pags 607-608 2.- Harrison, 16° edición apéndice A4-A5

Respuesta: B El adenocarcinoma renal (carcinoma de células renales) es la neoplasia renal más frecuente y corresponde al 90 a 95% de las neoplasias malignas primarias originadas en este órgano. La tríada clásica es: hematuria, dolor abdominal y tumoración abdominal palpable; esta tríada se observa sólo en el 10-20% de los casos.

175

176

SECCION CINCO PSIQUIATRIA

REVISADO POR: DRA CAROLINA GONZALEZ H. DRA MARIA EUGENIA HURTADO P. SRTA LINDSAY WALTON

177

178

179

180

PSIQUIATRÍA

Perfil SiCl

: Trastorno de la conducta alimentaria

Perfil : Deterioro Cognitivo SiCl 37.- Hombre de 65 años acompañado por sus familiares, los que refieren haber observado en el paciente desde hace un mes una pérdida sustancial en la memoria reciente y a largo plazo. El propio enfermo se queja de la pérdida de memoria y resalta dicha discapacidad con gran angustia, dando la impresión de que exagera las propias deficiencias. Además refiere agravamiento nocturno de las deficiencias. ¿ Cual de los siguientes es el diagnóstico mas probable que Usted se plantea ?

10.-Paciente femenino, de 17 años de edad, que presenta desde hace varios meses, baja de peso. La madre relata que nota que su hija, piensa demasiado en su alimentación, juega con la comida e intenta no comer en familia. Además considera que se ha puesto apática, y ha presentado meses de amenorrea. El diagnóstico más probable es : A.- Anorexia nerviosa. B.- Bulimia nerviosa C.- Trastorno somatomorfo D.-Trastorno de Ansiedad generalizada E.- Depresión

A.- Demencia B.- Pseudodemencia depresiva C.- Esquizofrenia D.- Delirium E.- Trastorno adaptativo

Alternativa Correcta : A La respuesta correcta es Anorexia Nerviosa. La paciente presenta una baja de peso y conductas en relación a la alimentación que hacen suponer un rechazo a mantener el peso corporal igual o por encima del valor mínimo normal considerando la edad y la talla. La presencia de amenorrea de varios meses también orienta al diagnóstico. Esta paciente no presenta Bulimia, ya que no presenta atracones de comida, presente en estas pacientes, ni conductas compensatorias, que en Bulimia suelen ser vómitos. El peso y la menstruación están muy afectados en la Anorexia Nerviosa, mientras que en la bulimia suelen estar normales. No presenta un Trastorno Somatomorfo, ya que no manifiesta insatisfacción con alguna parte de su cuerpo, o alguna manifestación somática, que nos pudiera orientar hacia este trastorno. No presenta criterios para un Trastorno de Ansiedad Generalizada, ya que no presenta síntomas de ansiedad. Finalmente no cumple con criterios de depresión, como anhedonia y estado de tristeza patológica de al menos 2 semanas de duración.

Alternativa Correcta : A La respuesta correcta es Demencia. El paciente presenta un deterioro cognitivo significativo expresado por él y sus familia, con deterioro marcado de la memoria reciente y de largo plazo, además de la presencia de empeoramiento nocturno, que es más frecuente en los cuadros de base orgánica. Las Pseudodemencias son cuadros de apariencia demencial que se deben a causas funcionales y producen un deterioro intelectual reversible, el que suele responder a tratamiento específico. En el caso de pseudodemencia depresiva, los síntomas afectivos suelen manifestarse previamente a los cognitivos y el agravamiento sería matinal. No hay elementos clínicos de Esquizofrenia. Para el diagnóstico de Delirium, faltaría el compromiso de conciencia, con importante compromiso de la atención. Un trastorno adaptativo no logra explicar los síntomas, principalmente el compromiso de memoria. Bibliografía : Vallejo 4a Ed., págs 616-617.

Bibliografía : Comité elaborador del DSMIV-TR. Manual diagnóstico y estadístico de los trastornos Mentales. Pag. 654 y 665.

Perfil : Síndrome de abstinencia con o sin delirium SiCl

181

convulsivas, se requiere manejo del paciente hospitalizado.

39.- Paciente de 45 años, traído a Urgencia por su familia porque escucharon un ruido y lo encuentran en el suelo, con actividad tónico clónica generalizada. Tiene antecedentes de etilismo crónico, pero en las últimas 24 horas no ha bebido y ha permanecido en casa, por no encontrarse bien. En Urgencias no se observa focalidad neurológica, está orientado y presenta temblor postural en las manos e intranquilidad. ¿Cuál es la conducta más correcta ?

Bibliografía : Kaplan 7a Ed., pág. 418.

Perfil : Trastorno de pánico SiCl 43.- Mujer de 35 años que desde hace unos meses tiene episodios de palpitaciones, sudores, temblores, sensación de ahogo y miedo a morir. Dice que le ocurren en cualquier lugar y en cualquier situación. El diagnóstico más probable es :

A.- Derivar e forma ambulatoria a tratamiento anti alcohólico B.- Hospitalizar C.- Derivar e forma ambulatoria a neurología D.- Derivar a domicilio y control E.- Derivar a Centro de Salud mental

A.- Depresión B.- Trastorno de pánico C.- Trastorno de ansiedad generalizada D.-Fobia simple E.- Trastorno del ánimo.

Alternativa Correcta : B La respuesta corresponde a indicar la hospitalización inmediata. El paciente presenta un síndrome de abstinencia alcohólica severo. La presencia de convulsiones es expresión de severidad. El síndrome de abstinencia se inicia a las 5-10 horas de haber cesado el consumo ( con temblor de manos, hiperactividad autonómica, ansiedad...), con un máximo a los 2-3 días, cediendo en una semana. En el 5% se da el cuadro más grave, el delirium tremens. Cursa con un síndrome confusional con desorientación, alteraciones perceptivas, como ilusiones y alucinaciones, con frecuencia desencadenadas con la privación sensorial , sobre todo visuales (típico las microzoopsias) escenográficas y complejas. También hay delirios (típico el "delirio ocupacional"), inquietud, clínica vegetativa y posibles convulsiones. Las crisis comiciales son tónico-clónicas generalizadas y estereotipadas. Los pacientes suelen tener más de una crisis en el período de tres a seis horas después de la primera. La actividad comicial en pacientes con historia de abuso de alcohol debería hacer pensar también en otros posibles factores causales, que incluyen lesiones cerebrales, infecciones del líquido cefalorraquídeo, neoplasias del SNC y otras enfermedades cerebro-vasculares. El abuso importante de alcohol durante largos períodos de tiempo puede producir hipoglucemia, hiponatremia e hipomagnesemia, trastornos todos ellos asociados a crisis convulsivas. Debido a la inestabilidad hemodinámica, posibles alteraciones hidroelectrolíticas, metabólicas y posible reaparición de crisis

Alternativa Correcta : B La respuesta corresponde a un Trastorno de Pánico o Angustia. La paciente presenta crisis frecuentes y espontáneas de angustia, asociadas a manifestaciones neurovegetativas como palpitaciones y sudoración, además de miedo intenso y sensación de ahogo No encontramos criterios de Depresión como anhedonia, tristeza, u alteraciones del ciclo sueño vigilia. No presenta un Trastorno de Ansiedad Generalizada ya que no presenta ansiedad y preocupación excesivas (expectación aprensiva) sobre una amplia gama de acontecimientos o actividades (como el rendimiento laboral o escolar). No presenta una Fobia Simple, ya que no se describe un miedo a una situación específica. Finalmente no es un Trastorno del Ánimo, ya que no se evidencian síntomas de la esfera anímica. Bibliografía : 1. Vallejo 5a edición Pág.392 2. Harrison, 15a edición Pág. 2976 3.- http://www.psicomed.net/dsmiv/dsmiv7.html _______________________________________ Perfil : Demencia o sindrome demencial-SiCl 52.- Paciente de 70 años, sin antecedentes neurológicos, psiquiátricos ni tratamientos

182

farmacológicos previos, presenta desde hace 8 meses deterioro mental progresivo, fluctuaciones en su nivel de atención y rendimiento cognitivo, alucinaciones visuales. En la exploración neurológica presenta signos parkinsonianos leves. ¿Cuál de los siguientes es el diagnóstico más probable?

Perfil : Episodios Maníacos SiCl 54.- Acude a consulta una mujer de 35 años acompañada de su marido. Ella dice encontrarse muy bien, mejor que nunca, está pudiendo hacer más cosas e incluso se siente capaz de comprender cuestiones muy complicadas. El marido dice que ella lleva una semana durmiendo poco y que no para; se levanta temprano, sale a la calle, vuelve, se cambia, vuelve a salir, habla sin parar con cualquier persona que se encuentra y no controla lo que gasta. Nunca le había ocurrido nada parecido y ella no acepta tener ningún problema. ¿ Cuál de los siguientes es el diagnóstico mas probable ?

A.- Enfermedad de Alzheimer. B.- Demencia vascular. C.- Demencia con cuerpos de Lewy. D.- Enfermedad de Huntington. E.- Psicosis hebefrénica. Alternativa Correcta : C La respuesta es Demencia por cuerpos de Lewy. Es un trastorno raro que debuta con un cuadro de demencia progresiva que puede presentar síntomas psicóticos. Los episodios de confusión, con gran deterioro de la atención, aparecen ya en fases precoces; puede haber agitación, alucinaciones e ideas delirantes muy intensas. Los signos parkinsonianos, que orientan al diagnóstico en este paciente, inicialmente pueden faltar o ser leves, siendo más frecuentes e intensos a medida que progresa el cuadro. La rigidez es el más frecuente. La enfermedad de Alzheimer es un cuadro de comienzo insidioso, predominando alteraciones de la memoria reciente, trastornos de tipo emocional o rasgos de conducta impredecible. La progresión suele ser lenta y gradual, pudiendo prolongarse durante diez años o más. Pueden aparecer signos extrapiramidales pero sólo en casos muy evolucionados. Las fluctuaciones del nivel de atención podrían hacernos pensar en el diagnóstico de Demencia Vascular; sin embargo, parece poco probable este cuadro en un paciente sin ningún antecedente de carácter isquémico cerebral . En la demencia vascular la progresión suele ser escalonada, con períodos alternados de mejor funcionamiento. El corea de Huntington debuta generalmente en pacientes más jóvenes que el de la pregunta. Desde fases iniciales los pacientes suelen presentar deterioro cognitivo, manifestaciones psiquiátricas (sobre todo de tipo afectivo) y trastornos del movimiento, siendo el corea el más característico. La Psicosis Hebefrénica es un tipo de Esquizofrenia que suele iniciarse en la adolescencia.

A.- Episodio maníaco. B.- Trastorno depresivo. C.- Trastorno psicótico breve. D.- Trastorno de ansiedad generalizada. E.- Trastorno disociativo. Alternativa Correcta : A La respuesta correcta es Episodio Maníaco. La clínica consiste en un estado de ánimo eufórico y aumento de la vitalidad. Existe una exaltación de la actividad psicomotora, inquietud, disminución de la necesidad de sueño y grandiosidad. La paciente presenta un autoconcepto engrandecido, con ideas megalomaníacas respecto a su capacidad intelectual. Presenta además pérdida de la distancia social, con facilitación del contacto interpersonal. No se trata de un Trastorno Depresivo porque no hay presencia de tristeza vital. No presenta síntomas psicóticos, ni síntomas de ansiedad, descartándose la Psicosis Breve o un Trastorno de Ansiedad. No presencia síntomas de disociación de la conciencia propios de los Trastornos Disociativos.

Bibliografía : http://www.psicomed.net/dsmiv/dsmiv6.html#3

Perfil : Ezquizofrenia SiCl 55.- Un paciente de 23 años acude al Servicio de Urgencia acompañado por sus padres por presentar, tras su vuelta de un viaje a Inglaterra, un cuadro de 3 días de evolución de insomnio,

Bibliografía : Harrison 13a Ed., pág. 2.618

183

tratado con éxito) y puede incluir los períodos de síntomas prodrómi-cos y residuales. Durante estos períodos prodrómicos o residuales, los signos de la alteración pueden manifestarse sólo por síntomas negativos o por dos o más síntomas de la lista del Criterio A, presentes de forma atenuada (p. ej., creencias raras, experiencias perceptivas no habituales). D. Exclusión de los trastornos esquizoafectivo y del estado de ánimo: el trastorno esquizoafectivo y el trastorno del estado de ánimo con síntomas psicóticos se han descartado debido a: 1) no ha habido ningún episodio depresivo mayor, maníaco o mixto concurrente con los síntomas de la fase activa: o 2) si los episodios de alteración anímica han aparecido durante los síntomas de la fase activa, su duración total ha sido breve en relación con la duración de los períodos activo y residual. E. Exclusión de consumo de sustancias y de enfermedad médica: el trastorno no es debido a los efectos fisiológicos directos de alguna sustancia (p. ej., una droga de abuso, un medicamento)o de una enfermedad médica. F. Relación con un trastorno generalizado del desarrollo: si hay historia de trastorno autista o de otro trastorno generalizado del desarrollo, el diagnóstico adicional de esquizofrenia sólo se realizará si las ideas delirantes o las alucinaciones también se mantienen durante al menos 1 mes (o menos si se han tratado con éxito). Basándonos simplemente en el criterio de duración, un episodio psicótico de 3 días de duración, no puede ser diagnosticado al menos de momento de esquizofrenia (respuesta 4 falsa). Las otras opciones no es hipomaniaco porque tiene síntomas psicóticos, No presenta síntomas de la línea afectiva con lo que se descartan las dos alternativas relativas a la depresión. La esquizofrenia es un diagnostico etiológico que no se sustenta todavía por buen ajuste social previo y la existencia de otros factores que pueden explicar un episodio psicótico secundario Es un episodio psicótico confuso que obliga a descartar psicosis secundaria (drogas, causas medicas).

irritabilidad, confusión, agitación psicomotriz, comentarios delirantes de contenido místicoreligioso, persecutorio, sexual, megalomaniaco y alucinaciones visuales y auditivas. Como antecedentes personales relevantes destacan una apendicectomía a los 17 años y ocasional consumo de alcohol y hachis. Se trata de una persona con buen ajuste psicosocial previo, así como de un buen estudiante que cursa el 4o curso de Ingeniero . No constan antecedentes personales ni familiares de tipo psiquiátrico. ¿ cual de los siguientes es el diagnostico mas probable ? A.- Episodio psicótico B.- Episodio hipomaniaco C.- Depresión sicótica D.- Esquizofrenia paranoide E.- Depresión mayor

Alternativa Correcta : D Criterios para el diagnóstico de esquizofrenia A. Síntomas característicos: Dos (o más) de los siguientes, cada uno de ellos presente durante una parte significativa de un período de 1 mes (o menos si ha sido tratado con éxito): 1) ideas delirantes. 2) alucinaciones. 3) lenguaje desorganizado (p. ej., descarrilamiento frecuente o incoherencia). 4) comportamiento catatónico o gravemente desorganizado. 5) síntomas negativos, por ejemplo, aplanamiento afectivo, alogía o abulia. Nota: sólo se requiere un síntoma del Criterio A si las ideas delirantes son extrañas, o si las ideas delirantes consisten en una voz que comenta continuamente los pensamientos o el comportamiento del sujeto, o si dos o más voces conversan entre ellas. B. Disfunción social/laboral: durante una parte significativa del tiempo desde el inicio de la alteración, una o más áreas importantes de actividad, como son el trabajo, las relaciones interpersonales o el cuidado de uno mismo, están claramente por debajo del nivel previo al inicio del trastorno (o, cuando el inicio es en la infancia o adolescencia, fracaso en cuanto a alcanzar el nivel esperable de rendimiento interpersonal, académico o laboral). C. Duración: persisten signos continuos de la alteración durante al menos 6 meses. Este período de 6 meses debe incluir al menos 1 mes de síntomas que cumplan el Criterio A (o menos si se ha

Bibliografía : Kaplan 7a Ed., Págs. 485-490. _______________________________________

184

Perfil : Trastornos específicos de la personalidad SiCl

sentimientos ajenos. ¿Qué tipo de personalidad presenta el paciente?:

60.- Paciente de 29 años que acude a urgencias tras una pelea callejera. Como antecedentes personales destaca fumador de 3 paquetes/dia, consumidor habitual de drogas y hábito enólico severo. Desde hace 1 año se encuentra en libertad provisional. Interrogando sobre el motivo de la pelea el paciente comenta que tenía ganas de pegar, como un impulso, sin remordimientos de lo hecho. En los últimos 6 meses ha acudido 5 veces a urgencias por los mismos motivos. Probablemente este paciente presenta un trastorno de la personalidad:

A.- Paranoide. B.- Esquizoide. C.- Esquizotipica. D.- Esquizofrénica. E.- Borderline. Alternativa Correcta : B La respuesta es Personalidad Esquizoide. Estos pacientes parecen indiferentes a las relaciones sociales, son sujetos fríos, de nula expresividad emocional; dan la impresión de vivir ensimismados y ausentes. Se acompaña de vivencias autorrefercncialcs, mecanismos de evitación, disconformidad con la autoimagen; introversión, aislamiento. En el Trastorno Esquizotípico destaca una conducta extravagante, asociada a una tendencia al pensamiento mágico. Los síntomas de este paciente no conforman una psicosis esquizofrénica. La personalidad paranoide consiste en desconfianza excesiva e injustificada, suspicacia, hipersensibilidad a atribución de malaintención a terceros. Se sienten amenazados en todo momento. El Trastorno Borderline se caracteriza por marcada inestabilidad emocional, impulsividad, disfunción del autoconcepto y gran temor por el abandono en sus relaciones interpersonales, por lo cual suelen tener una conducta manipuladora del medio.

A.- Esquizoide. B.- Narcisista. C.- Antisocial. D.- Histriónico. E.- Paranoide. Alternativa Correcta : C La respuesta es Trastorno de personalidad Antisocial. El paciente presenta severa conducta impulsiva asociada a falta de remordimiento frente a daño de terceros. Estos pacientes destacan por su frialdad y falta de miedo, presentan una conducta antisocial continua y crónica, en la que se violan los derechos de los demás. Se presenta antes de los 15 años y persiste en la edad adulta. Se acompaña de fracaso para adaptarse a las normas sociales en lo que respecta al comportamiento legal, como lo indica el perpetrar repetidamente actos que son motivo de detención, deshonestidad, impulsividad o incapacidad para planificar el futuro, irritabilidad y agresividad, irresponsabilidad persistente, falta de remordimientos.

Bibliografía : 1.- Vallejo. 4° edición. Pag. 578. 2.-http://www.psicomed.net/dsmiv/dsmiv16.html

Perfil : Trastornos somatomorfos SiCl Bibliografía : 1.- Introducción a la Psicopaiotogía y la Psiquiatría. Vallejo 4ª edición pág, 575-577. 2.http://www.psicomed.net/dsmiv/dsmiv16.html

166.- Se presenta a la consulta una mujer de 36 años que cuenta, con cierta indiferencia que ha perdido visión de forma brusca en los dos ojos. Durante el interrogatorio se constata que esto ocurrió hace 4 horas, cuando una amiga con la que se iba a ir de viaje de placer le dijo que no podía ir. La exploración por lo demás resulta anodina y en la anamnesis se recoge un episodio de anestesia en un brazo hace 1 año. ¿Cual de los siguientes es el diagnostico mas probable?:

Perfil : Trastornos específicos de la personalidad (situación clínica) 125.- Una persona presenta dificultad para establecer amistades, gran frialdad afectiva y gran indiferencia tanto a la crítica como a los

A.-Trastorno de conversión. B.-Trastorno facticio. C.-Trastorno de somatización.

185

de lo que ocurre en el trastorno facticio o en la simulación). D. Tras un examen clínico adecuado, el síntoma o déficit no se explica por la presencia de una enfermedad médica, por los efectos directos de una sustancia o por un comportamiento o experiencia culturalmente normales. E. El síntoma o déficit provoca malestar clínicamente significativo o deterioro social, laboral, o de otras áreas importantes de la actividad del sujeto, o requieren atención médica. F. El síntoma o déficit no se limita a dolor o a disfunción sexual, no aparece exclusivamente en el transcurso de un trastorno de somatización y no se explica mejor por la presencia de otro trastorno mental.

D.-Simulación. E.-Trastorno por estrés agudo. Alternativa Correcta : A La respuesta correcta es Trastorno de conversión, ya que la descripción de focalidad neurológica son incongruentes, sin hallazgos en la exploración, en que en el episodio actual de ceguera bilateral tiene un desencadenante emocional. Todo ello, junto a la indiferencia con la que la paciente lo relata. No se trataría de un Trastorno facticio, ya que este consiste en la producción de síntomas de forma voluntaria por parte del paciente, con el objetivo de ser considerado enfermo y obtener así beneficio psicológico. No es un Trastorno de somatización (síndrome de Briquet), que consiste en la presencia de múltiples síntomas físicos, predominando los dolores. Se necesitan al menos cuatro dolores distintos, junto a otros síntomas gastrointestinales, sexuales y neurológicos distintos al dolor, con inicio antes de los 30 años, persistentes y que conducen a la búsqueda incesante de atención médica o producen un deterioro significativo en áreas importantes de la vida. No es Simulación, ya que no se observa la motivación de inventar síntomas para obtener un beneficio económico o legal, este no es un trastorno psiquiátrico, sino un problema médicolegal. No se trataría de un Trastorno por estrés agudo, ya que este supone una reacción a un acontecimiento traumático más allá de la experiencia habitual humana (violación, accidente grave, atentado, catástrofe,...). Cursa con reexperimentación del episodio, conductas de evitación, embotamiento emocional y síntomas de hiperalerta. Se diferencia del trastorno de estrés postraumático en que dura menos de un mes.

Bibliografía : DSM IV online :http://www.psicomed.net/dsmiv/dsmiv8.html

Perfil : Trastornos específicos de la personalidad SiCl 167.- Mujer de 46 años, extrovertida y manipuladora que establece con facilidad relaciones que terminan sistemáticamente en conflictos que llevan al distanciamiento. Tiene tendencia a deformar la realidad y a comportarse con teatralidad. ¿Cuál de los siguientes trastornos de personalidad es más probable que padezca?: A. Trastorno de personalidad por evitación. B. Trastorno de personalidad por dependencia. C. Trastorno obsesivo-compulsivo de la personalidad. D. Trastorno histriónico de la personalidad. E. Trastorno límite de la personalidad. Alternativa Correcta : D Los pacientes con un trastorno histriónico de la personalidad son extrovertidos, mal socializados, desajustados emocionalmente y dependientes. Su conducta es teatral, reactiva e hiperexpresiva y sus relaciones superficiales, cargadas de egocentrismo, hipocresía y manipulación (respuesta D correcta). En el trastorno de personalidad por evitación predomina un patrón de inhibición social, sentimientos de incompetencia e hipersensibilidad a la evaluación negativa (respuesta A incorrecta). En el trastorno por dependencia lo que predomina es un patrón de comportamiento

Criterios para el diagnóstico de F44 Trastorno de conversión (300.11) A. Uno o más síntomas o déficit que afectan las funciones motoras voluntarias o sensoriales y que sugieren una enfermedad neurológica o médica. B. Se considera que los factores psicológicos están asociados al síntoma o al déficit debido a que el inicio o la exacerbación del cuadro vienen precedidos por conflictos u otros desencadenantes. C. El síntoma o déficit no está producido intencionadamente y no es simulado (a diferencia

186

sumiso, relacionado con una excesiva necesidad de ser cuidado e incapacidad para poder valerse por sí mismo (respuesta B incorrecta). El trastorno obsesivo-compulsivo corresponde a personas perfeccionistas, meticulosas y rígidas con un patrón de preocupación por el orden, el perfeccionismo y el control (respuesta C incorrecta). Por último, en el trastorno límite de la personalidad, predomina un patrón de inestabilidad en las relaciones interpersonales, en la autoimagen y en los afectos, con una notable impulsividad (respuesta E incorrecta).

conciencia está preservada durante el episodio pero algunos pacientes tienen alucinaciones vividas. La prueba de latencias de sueño múltiples (múltiple sleep latency test. MSLT) es útil en el diagnóstico y debería practicarse antes de iniciar un tratamiento. El tratamiento de la narcolepsia es sintomático.

Bibliografía : Vallejo "Introducción a la Psicopatología y Psiquiatría" 5ª edición Pág. 565.

Perfil : Trastorno de pánico SiCl

Bibliografía : 1. Vallejo. 5a edición, Pág. 261, 2. Farreras, 14° edición, Pág. 1587.

169.- Paciente de 35 años bruscamente presenta un episodio de : sensación de dificultad respiratoria, sudoración , sensación de ahogo (disnea), mareo, sensación de inestabilidad o desfallecimiento, palpitaciones y náuseas o malestar abdominal. Por este episodio acude al servicio de urgencia donde se descarta patología orgánica sin embargo el paciente evoluciona con temor permanente a la repetición del episodio ¿Cuál de los siguientes es el diagnóstico más probable?:

Perfil : Trastornos del sueño SiCl 168.-Paciente de 23 años estudiante de medicina refiere historia de mas menos 6 meses de evolución caracterizado por somnolencia diurna excesiva que se asocia a episodios de debilidad muscular que aparecen durante la vigilia producidos sobre todo por emociones, parálisis del sueño y alucinaciones al inicio del sueño, pese a que el paciente no refiere estrés alguno en el ultimo tiempo, ni tampoco cambios en los horarios de sueño. El diagnostico presuntivo mas probable:

A.-Trastornos de ansiedad generalizada. B.- Trastorno de pánico. C.-Trastorno mixto ansioso-depresivo. D.-Trastorno fóbico. E.- Trastorno de adaptación con síntomas ansiosos

A.- Trastorno por abuso de sustancias B.- Depresión C.- Ezquizofrenia D.- Trastorno de Personalidad. E.- Narcolepsia

Alternativa Correcta : B Se trata de un Trastorno de pánico, que consiste en la presencia de crisis de ansiedad recurrentes, con inicio brusco y máximo en unos diez minutos. Se presenta con dos tipos de síntomas: vegetativos (palpitaciones, disnea, desfallecimiento, náuseas, malestar abdominal,...) y psicológicos (sensación de muerte, de estar volviéndose loco, sentimientos de despersonalización y desrealización). El trastorno tiene ansiedad anticipatorio. No es un Trastorno de ansiedad generalizada, ya que no presenta un estado de ansiedad y preocupación crónico, ni múltiples síntomas somáticos que ocasionen malestar o mal funcionamiento social o laboral. El Trastorno mixto ansioso-depresivo, exigiría síntomas de las dos categorías que engloba de forma crónica, sin cumplir criterios de ninguna de ellas.

Alternativa Correcta : E La alternativa correcta es Narcolepsia, en que la somnolencia diurna excesiva se asocia a cataplejía, parálisis del sueño y alucinaciones hipnagógicas (inicio del sueño), pero la presentación más común consiste en somnolencia excesiva y cataplejía. La enfermedad aparece sobre todo en la segunda tercera décadas y dura toda la vida. La somnolencia puede ser continua u ocurrir en forma casi imprevisible, en ataques difíciles de controlar, relativamente breves. Los ataques de cataplejía son episodios de debilidad muscular que aparecen durante la vigilia producidos sobre todo por emociones. La parálisis del sueño consiste en episodios de incapacidad para moverse, al inicio del sueno o al despertar., La

187

sumisa, con temor al abandono. No se caracteriza por el retraimiento social. El Trastorno de personalidad obsesivocompulsivo, el mecanismo central el mantener el control sobre sus procesos internos y sobre las situaciones del mundo externo, lo que se manifiesta en perfeccionismo, obstinación, indecisión, excesiva devoción al trabajo y al rendimiento, dificultad para expresar emociones cálidas y tiernas. El Trastorno pasivo-agresivo, que no se clasifica en el DSM IV, se caracteriza por resistencia laboral o social expresada de forma indirecta (tozudez, demoras, olvidos, etc.) En el Trastorno límite de la personalidad, lo central es un patrón de inestabilidad en las relaciones interpersonales, la autoimagen y la afectividad y una notable impulsividad.

No es un Trastorno fóbico, ya que no presenta un temor persistente a algo, que a pesar de reconocerlo como irracional permanece fuera del control voluntario, generando conductas de evitación. El trastorno de adaptación, suele responder a un factor estresante identificable, y se expresa clínicamente como un malestar mayor de los esperable por la magnitud del estresante, con deterioro significativo de la actividad social o laboral. Bibliografía : Kaplan 7ª ed., págs 745 y 596-597

Perfil : Trastornos específicos de la personalidad SiCl

Bibliografía : Vallejo, 5ª edición, Pág. 572

172.- Una mujer de 37 años comenta estar muy limitada en sus actos porque siempre acaba abandonando los planes que de antemano le parecen interesantes por miedo a que su familia los desapruebe. No se relaciona con personas que no conoce y no ha hecho nuevos amigos desde hace años por temor al rechazo, la humillación o la vergüenza. Muestra además un importante retraimiento social a pesar de que afirma que le encantaría dar cariño a otras personas y ser correspondida. Dice que le cuesta mucho destacar en su trabajo porque sus compañeros con mucho menos esfuerzo que ella obtienen mejores resultados y son más brillantes. Sus padres comentan que siempre fue un persona con baja autoestima y que necesita que se le anime mucho para sacar adelante nuevos proyectos. ¿Qué trastorno de la personalidad tendría la paciente? A. Trastorno por dependencia. B. Trastorno de personalidad obsesivocompulsivo. C. Trastorno pasivo-agresivo. D. Trastorno por evitación. E. Trastorno límite de la personalidad.

Alternativa Correcta : D Se trataría de un Trastorno por evitación, por la hipersensibilidad al rechazo, la humillación o la vergüenza, el retraimiento social, a pesar del deseo de afecto, y la baja autoestima. En el Trastorno por dependencia, lo que prima es la necesidad, de modo exagerado, de cuidado, apoyo y protección, por lo que adopta una actitud

188

SECCION SEIS ESPECIALIDADES

REVISADO POR OFEM SR RENATO ALARCÓN.

189

190

191

192

ESPECIALIDADES - DERMATOLOGÍA

Respuesta correcta D Datos clave para reconocerlo; el lupus cutáneo subagudo suele tratarse de mujeres jóvenes, con marcada fotosensibilidad (como en este caso clínico) con lesiones en hombros, espalda y zona del escote, que consiste en lesiones eritematoedematosas con forma anular, y confluentes. Este caso se trataría de la forma anular policíclica. Cuando hay más descamación, se trataría de la forma psoriasiforme. Es muy característica la presencia de anticuerpos antiRo, así como que si esta paciente se quedara embarazada, el paso transplacentario de dichos anticuerpos podría ocasionar lupus neonatal. Las lesiones básicas del resto de opciones son: a) eritema polimorfo: lesiones en diana b) liquen plano: pápulas poligonales, pruriginosas, planas, poligonales, violáceas c) porfiria hepatocutánea: ampollas en dorso de manos d) dermatomiositis: eritema en heliotropo (párpados), pápulas sobre nudillos de las manos (Gottron).

Perfil : Sarna SiCl 31.Hombre de 32 años con intenso prurito que se exacerba por las noches. Presenta pápulas eritematosas y pruriginosas en la cara ventral de las muñecas, pliegues interdigitales y en prepucio. Su hijo de cuatro años presenta síntomas semejantes. ¿ Cuál de los siguientes tratamientos es el indicado ? A.- Crema de permetrina al 5% B.- Clotrimazol C.- Griseofulvina D.- Glucantime E.- Corticoides tópicos Respuesta correcta A Ante un caso de sarna ha de tratarse al paciente y a su familia con loción o crema de permetrina al 5%, cubriendo toda la piel del cuerpo excepto la cabeza. Así mismo ha de tratarse la ropa con agua muy caliente. La triada diagnostica de escabiosis es prurito nocturno, lesiones compatibles en tipo y localización y mas de un caso en la familia

Bibliografía: Reumatología, 5a Edición, Pág. 21.

Bibliografía: Armijo, Camacho. Tratado de dermatología. Págs. 711-714

Perfil: Líquen plano-SiCl 129.- Paciente de 40 años, hipertenso moderado en tratamiento con enalaprila maleato. Presenta desde hace tres meses lesiones muy pruriginosas, papulosas, poligonales, localizadas en cara anterior de muñecas y tobillos y lesiones reticuladas blanquecinas en la mucosa oral . ¿Cuál de los siguientes es el diagnóstico más probable?

Perfil : Manifestaciones cutáneas de enfermedades sistémicas-PrDe 56.- Mujer de 32 años, después de exponerse al sol, presenta desde hace 20 días lesiones cutáneas que le ocasionan pocas molestias. Examen físico: lesiones anulares en la zona externa de hombros y brazos y en la región escapular. Las lesiones son eritematosas, con edema en su borde y regresión central. Miden dos a tres centímetros de diámetro y algunas son confluentes. ¿ Cuál de los siguientes es el diagnóstico más probable ?

A.- Pitiriasis rosada. B.- Papulosis linfomatoide. C.- Psoriasis eruptiva. D.- Liquen plano. E.- Reacción adversa a fármaco Respuesta correcta D Es claro que, ante un paciente con lesiones papulosas POLIGONALES, color rojo-violáceo, muy pruriginosas y en cara flexora de muñecas y antebrazos, tobillos, región lumbosacra o flancos, pensamos sin dudarlo... ¡¡En un LIQUEN PLANO!! En casi un 75% de los casos, hay

A.- Eritema polimorfo. B.- Liquen plano. C.- Porfiria hepatocutánea. D.- Lupus eritematoso cutáneo E.- Dermatomiositis.

193

La psoriasis se caracteriza por el brote de pápulas y placas eritematosas bien delimitadas y cubiertas por varias capas de escamas finas y nacaradas . Tras eliminar las escamas, por raspado de las lesiones, aparece en su superficie un piqueteado hemorrágico (signo de Auspitz). El prurito es en general escaso. La aparición y la distribución de la erupción son muy variables, pero a menudo el inicio es gradual y la distribución simétrica, siendo las zonas afectadas con mayor frecuencia el cuero cabelludo, la región lumbosacra y la superficie de extensión de las extremidades, sobre todo codos y rodillas

lesiones en mucosa oral y genital, donde característicamente se presentan como lesiones reticuladas blanquecinas. A veces, esta enfermedad aparece asociada a hepatitis C o diversos fármacos (sales de oro, cloroquina, tiazidas.). La anatomía patológica muestra un infiltrado inflamatorio en banda, en la unión dermoepidérmica, acantosis con papilas en dientes de sierra, hiperqueratosis con hipergranulosis y queratinocitos apoptóticos (cuerpos de Civatte). El tratamiento consiste en corticoides tópicos y antihistamínicos para casos leves. Si no hay respuesta o es un caso grave, se usan corticoides orales (recordad que en la psoriasis, otra enfermedad eritematoescamosa descamativa muy típica , los corticoides sólo se usan de forma tópica, ya que los sistémicos pueden producir efecto rebote al ser suspendidos y taquifilaxia). Otras opciones para casos graves serían los PUVA o la ciclosporina.

Bibliografía:

Bibliografía: Dermatología, 4a Edición, Pág. 21

Perfil: Psoriasis-SiCl 138.-Paciente de 30 años con pápulas y placas eritematosas bien delimitadas y cubiertas por varias capas de escamas finas, en codos y rodillas. Al retirar la última capa de escamas aparece un piqueteado hemorrágico en la superficie de la lesión . ¿ Cuál de los siguientes es el diagnóstico más probable ? A.- Psoriasis. B.- Liquen Plano. C.- Dermatitis de contacto. D.- Pitiriasis rosada. E.- Dermatitis atópica. Respuesta correcta A

194

Farreras, 13ª Edición, página 1311

195

196

ESPECIALIDADES OFTALMOLOGÍA

Pseudomembranas (exudado adherido a la conjuntiva): víricas, gonocócicas (rara). Edema: puede aparecer en cualquier tipo. Si es severa se denomina quemosis. Flictenas: queratoconjuntivitis flictenular. Se asocia a blefaritis estafilocócica. Origen inmunitario. Petequias conjuntivales: principalmente en la conjuntiva tarsal, orientan hacia una etiología vírica.

Perfil : Conjuntivitis viral no complicada SiCl 17.-Paciente de 62 años que presenta enrojecimiento ocular bilateral con secreción serosa abundante, sensación de cuerpo extraño, lagrimeo, reacción folicular, hemorragia subconjuntival difusa y adenopatía preauricular palpable. Hace una semana presentó una infección de vías respiratorias altas ¿Cuál es la etiología más probable de esta conjuntivitis? A.- Bacteriana. B.-De inclusión. C.- Vírica. D.- Alérgica E.- Tóxica. Respuesta correcta C En el caso de la pregunta no hay opción a dudas, pues la única posibilidad que te ofrecen es la conjuntivitis. Los principales datos clínicos para orientar el diagnóstico etiológico son el tipo de secreción y la aparición de folículos o papilas.

Para ver esta película, debe disponer de QuickTime™ y de un descompresor TIFF (LZW).

*Secreción: - Serosa (parecida a la lágrima): vírica o tóxico. -Mucosa (filante): alérgica o primaveral. -Purulenta (pus): bacteriana. Bibliografía: Manual CTO 3ª Ed., Oftalmología 6.2 Pregrado oftalmología Capítulos VIII y IX. XXX

*Folículos (formación linfoide sobreelevada localizada predominantemente en la conjuntiva tarsal inferior): - Vírica - Chlamydia - Foliculosis (hipertrofia del tejido linfoide que aparece en casi tres cuartas partes de los niños sin síntomas ni relevancia alguna). - Papilas (elevaciones rojizas vasculares; si son grandes da un aspecto de empedrado, especialmente en la conjuntivitis vernal, primaveral, de predominio en la conjuntiva tarsal superior): - Alergia primaveral. - Uso de lentillas (papilas gigantes). Y otros datos: Adenopatía preauricular: típica de las víricas o por Chlamydia.

Para ver esta película, debe disponer de QuickTime™ y de un descompresor TIFF (LZW).

197

los cristales de colesterol subretinianos y el aspecto pizarroso retiniano.

Perfil : Tumores endoculares (retinoblastoma y otros) SiCl 34.-Niño de 20 meses de edad que presenta leucocoria en el ojo derecho. ¿ Cuál de los siguientes es el diagnóstico más probable?

• Fibroplasia retrolental: Es un proceso bilateral en prematuros o recién nacidos de bajo peso que hayan precisado oxigenoterapia. Hay que reconocer las alteraciones vasculares proliferativas periféricas y la ausencia de calcificaciones.

A.- Dacriocistitis del recién nacido. B.- Coloboma de iris. C.- Hemianopsia homónima derecha. D.- Astigmatismo. E.- Retinoblastoma.

Bibliografía: Manual CTO 3a Ed., Oftalmología 11.9 Pregrado de Oftalmología Pág. 232 - 233

Respuesta correcta E Se denomina leucocoria al cuadro clínico inespecífico multicausal que cursa con la aparición de un reflejo blanquecino en el área pupilar. Las principales causas de leucocorias en el niño son:

Perfil: Neuritis óptica SiCl. 120.- Paciente de 23 años de edad con pérdida progresiva de la agudeza visual y dolor con los movimientos oculares. Examen Ocular : exploración del segmento anterior y posterior normal. Escasa respuesta pupilar a la luz. ¿ Cuál de los siguientes es el diagnostico más probable ? A.- Obstrucción de la arteria central de la retina. B.- Papilitis. C.- Glaucoma crónico simple D.-Desprendimiento de retina. E.-Neuritis óptica

Catarata congénita (causa más frecuente). Retinoblastoma. Retinopatía de la prematuridad. Persistencia del vitreo primario hiperplásico. Toxocariasis ocular. Enfermedad de Coast. Veamos ahora el diagnóstico diferencial de las causas más frecuentes: Persistencia del vitreo primario: El vitreo en su desarrollo es invadido por los vasos que constituyen la arteria hialoidea y la túnica vasculosa lentis. Dicho sistema vascular regresa y desaparece mientras se constituye la vascularización propia de la retina. El paro en este proceso condiciona el mantenimiento de dicho tejido vasculoglial, constituyendo la persistencia del vitreo primario en su forma anterior o retrocristalina o en su forma posterior o retiniana. El diagnóstico diferencial con el retinoblastoma se basa en su aparición congénita, unilateral en el 90% de los casos. Se suele asociar a microftalmía.

Respuesta correcta E El paciente de la pregunta presenta una pérdida progresiva de la agudeza visual, por lo que podemos descartar la opción A, dado que la obstrucción de la arteria central de la retina cursa con una pérdida brusca e indolora de la agudeza visual. La clínica del desprendimiento de retina tampoco encaja con la clínica de nuestro paciente. El desprendimiento de retina suele comenzar con miodesopsias o moscas volantes. Cuando la retina se va desprendiendo se producen fosfenos, por estímulos mecánicos. A medida que el área desprendida va aumentando de tamaño, aparece una sombra continua en el campo visual periférico, que va progresando hacia el centro.

• Enfermedad de Coast: En general, de comienzo más tardío, unilateral, afecta fundamentalmente a varones y se observan en el fondo de ojo las típicas anomalías vasculares,

El glaucoma crónico simple es la forma más frecuente de glaucoma. En la mayoría de los casos aparece en la edad media de la vida o algo más

198

tarde. La evolución suele ser gradual asintomática, y en general, muy lenta (años).

las opacificaciones centrales, mientras la miosis no, razón por la cuál la agudeza visual mejora en la oscuridad. La fotosensibilidad es debida a que al atravesar la zona opacificada tiene lugar una distorsión del haz de luz, estimulando mayor zona de la retina. La misma explicación nos sirve para el hecho de aparición de puntos luminosos con halos a su alrededor. Para el diagnóstico se lleva a cabo una midriasis farmacológica y observamos a simple vista la leucocoria, o bien por oftalmoscopia directa a 30 cm, viéndose manchas oscuras sobre el reflejo del fondo de ojo o impidiéndose ver su reflejo cuando son maduras. La lámpara de hendidura permite ubicar la opacidad dentro del cristalino. La presbicia cursa con pérdida de la acomodación por disminución de la elasticidad del cristalino y pérdida de la fuerza del músculo ciliar. Provoca una incapacidad para enfocar objetos cercanos, sobre todo cuando la acomodación disminuye por debajo de 3 ó 4 dioptrías, con lo que no se enfoca objetos situados a 25-30 cm La degeneración macular senil se caracteriza por un síndrome macular y drusas en la mácula, no justificando este cuadro. La neuropatía isquémica anterior se caracterizaría por disminución de la agudeza visual, defecto pupilar aferente y escotoma altitu-dinal, por lo que

y

La duda razonable sería entre las opciones B (papilitis) y E (neuritis óptica retrobulbar). La clínica es similar en ambas; disminución de la agudeza visual entre moderada y severa, dolor ocular u orbitario que aumenta con los movimientos oculares, así como defecto pupilar aferente. La diferencia está en la exploración del fondo de ojo, mostrando una papila hiperémica y edematosa, en el caso de la papilitis, y un fondo de ojo normal en el caso de la neuritis óptica retrobulbar (se dice de ellas que ni el paciente ni el médico ven nada). Bibliografía: Manual CTO 3ª Ed., Oftalmología 13.3, 11.6, 11.3, 9.1 XXX

Perfil Catarata SiCl. 126.- Hombre de 64 años que consulta por disminución de la visión en los dos ojos, de varios meses de evolución. La disminución es en la visión de lejos y más acentuada en la visión próxima o de lectura y aumenta con la luz solar intensa. También se deslumbra con mayor facilidad y cree ver los colores más apagados. No refiere metamorfopsias. ¿Cuál de los siguientes diagnósticos es el más probable? A.- Presbicia. B.- Degeneración macular asociada a edad. C.- Cataratas. D.- Neuropatía óptica anterior isquémica. E.- Glaucoma. Respuesta correcta C Nos presentan un caso clínico en el que se produce pérdida progresiva de agudeza visual bilateral, siendo esto muy típico de las cataratas, y aunque la presbicia podría darlo, no explica sin embargo el resto del cuadro. En las cataratas (opción 3 correcta) la afectación aunque bilateral es asimétrica. Se produce una miopización progresiva por el aumento del poder de la convergencia de la lente, que hace que a veces debute como la mejoría de una presbicia, mejorando la visión cercana. La mejoría o disminución de la agudeza visual con las variaciones del diámetro pupilar se debe a que con la midriasis los rayos de luz pueden salvar

tampoco corresponde al cuadro que nos presentan. El glaucoma se caracteriza por la aparición progresiva de escotomas, inicialmente periféricos y perimaculares, que posteriormente evolucionan a escotomas de Bjerrum y escalón nasal de Ronne, para finalmente quedar sólo un islote central y otro temporal, acabándose en amaurosis. Por esto

199

la agudeza visual se conserva hasta el final ya que primero la zona macular esta intacta. Su diagnóstico se sospecha con el aumento de la PIÓ y se confirma con la campimetría. Bibliografía: Manual CÍO 3a Ed., Oftalmología 8. Guiones cíe Oftalmología Alio 8a Ed., Pág. 83 XXX

200

201

202

203

ESPECIALIDADES OTORRINOLARINGOLOGÍA

se aprecian como nódulos bilaterales en la unión de los tercios anterior y medio, y su presentación típica es en profesoras o cantantes con disfonía persistente sin ninguna otra clínica (diferencia con los tumores glóticos malignos, respuesta A falsa). La presentación del edema de Reinke (respuesta B falsa) suele ser en varones fumadores empedernidos con voz grave, diplofonía y estridor. Es una lesión bilateral, que en la laringoscopia se aprecia como una cuerda edematosa entre el epitelio y el ligamento vocal. La lesión del nervio laríngeo recurrente suele presentarse tras cirugías previas del cuello, tumores malignos cervicales o intubaciones bruscas. La cuerda paralizada se queda en posición paramediana y presentan disfonía con voz bitonal (respuesta C falsa).

Perfil: Disfonía SiCl. 24.- Mujer de 38 años, profesora primaria . Consulta por disfonía persistente . En la entrevista se observa una mujer motivada por su trabajo, muy activa, con un elevado nivel de estrés, Antecedente : fumadora ocasional. ¿ Cuál de los siguientes es el diagnóstico más probable ? A.- Carcinoma espinocelular de asiento glótico. B.- Edema de Reinke. C.- Parálisis del nervio recurrente. D.- Reflujo gastroesofágico. E.- Nódulos vocales.

Bibliografía: Manual CTO de Otorrinolaringología, 4a Edición, Pág. 35.

Perfil: Complicaciones de la OMA SiCl. 107.- Niña de 7 años, que presento una otitis media hace cuatro semanas. Actualmente tiene fiebre hasta 39 ºC, malestar general, otalgia que se irradia a la región temporal y al occipucio y otorrea. Exámenes : VHS : 35 mm/hora y glóbulos blancos : 12.000 mm3 . ¿Cuál de los siguientes es el diagnóstico más probable?: A.- Laberintitis purulenta. B.- Laberintitis serosa. C.- Mastoiditis D.- Meningitis otógena E.- Paralaberintitis

Respuesta correcta E

Respuesta correcta C

La presentación clínica del caso nos orienta directamente hacia los nódulos vocales (opción E correcta), que son la formación benigna glótica más frecuente en las mujeres. En la laringoscopia

204

La mastoiditis es la complicación más frecuente de la otitis media, y cursa con empeoramiento del estado general, fiebre, otalgia pulsátil que irradia a la región temporal y al occipucio, y supuración purulenta no fétida e hipoacusia. La forma más común que indica que la mastoiditis está ocurriendo es la aparición de dolor y sensibilidad en la mastoides. Como la infección progrese, aparecerá edema y eritema sobre los tejidos post - auriculares. Se acompaña de leucocitosis con desviación izquierda y gran elevación de la VHS (alternativa C correcta). Las laberintitis y paralaberintitis también pueden ser complicaciones de una otitis media, pero el cuadro clínico está dominado por los signos y síntomas cocleovestibulares con vértigo, nauseas, vómitos. Romberg periférico positivo, acúfenos y sordera rápidamente progresiva (alternativas A, B y E incorrectas). La meningitis otógena presenta un cuadro clínico dominado por la tríada fiebre, cefalea y vómitos más los signos de irritación meníngea (rigidez de nuca, Kernig y Brudzinski (alternativa C incorrecta).

El cáncer nasofaríngeo (respuesta 2 correcta) normalmente no causa síntomas precoces; sin embargo en ocasiones puede provocar otitis inedia serosa unilateral por obstrucción de la trompa de Eustaquio, obstrucción nasal uni-bilateral o epistaxis. Lo que nos orienta al diagnóstico en esta pregunta es la diplopía, ya que debido a la proximidad de la nasofaringe a la base del cráneo, un proceso invasivo local puede causar neuropatías craneales aisladas o múltiples: I, ITI, fV, V, VI, IX, X y XI.

Bibliografía: 1.- Current Diagnosis & Treatment in Otolaryngology Head & Neck Surgery, 2004. XII. External & Middle Ear, Chapter 48. Otitis Media

Bibliografía: Manual de otorrinolaringología.. Ramírez., Ia edición, Pág. 306.

Perfil : Cáncer de Nasofaringe SiCl.

Perfil: Síndrome vertiginoso . SiCL

127.- Hombre de 40 años que presenta una otitis serosa unilateral hace dos años y diplopía de comienzo insidioso. ¿Cuál de los siguientes es el diagnóstico más probable? A.- Petrositis. B.- Carcinoma de nasofaringe. C.- Ototubaritis. D.- Poliposis nasal. E.- Trombosis del seno cavernoso.

159.- Paciente de 28 años , refiere mareo al levantarse o acostarse en la cama sobre el lado derecho que le dura unos 20 segundos, a veces con sensación nauseosa, desde hace un mes. ¿Cual de los siguientes es el mejor tratamiento? A.- Sulpiride endovenoso. B.- Cinarizina oral. C.- Maniobra liberadora de Semont. D.- Rehabilitación vestibular. E.- Cirugía.

Respuesta correcta B Respuesta correcta C

205

Pregunta de dificultad media, de fácil diagnóstico

Para ver esta película, debe disponer de QuickTime™ y de un descompresor TIFF (sin comprimir).

pero que exige conocer el tratamiento de vértigo. Se trata de un caso típico de vértigo posicional paroxístico benigno porque aparece un vértigo al adoptar una determinada posición, que se repite, de corta duración y sin síntomas auditivos. El diagnóstico se realiza con la maniobra de DixHallpicke, y el tratamiento no es farmacológico sino con maniobras de reubicación canalicular (Eppley, Semont).La maniobra de Semont (también llamada maniobra "liberadora") es un procedimiento donde el paciente es movido desde una posición acostado sobre uno de sus lados rápidamente desplazados hacia el otro lado (Levrat et al 2003). Esta es una maniobra un tanto brusca por lo que no se realiza generalmente en Estados Unidos, pero tiene 90 % de eficacia luego de 4 sesiones de tratamiento. El sulpiride y la cinarizina son tratamientos farmacológicos para la fase aguda de otros vértigos. La rehabilitación vestibular acelera la compensación en los vértigos con déficit vestibular, pero no en el VPPB. La cirugía se reserva para paciente con Menière que no responden a tratamiento médico o neurinomas de VIII PC.

Bibliografía: Manual CTO de Otorrinolaringología, 5a Edición, Pág. 17.

206

SECCION SIETE

SALUD PUBLICA

REVISADO POR LUIS MARTÍNEZ OLIVA MARCIAL ORELLANA

207

208

Salud Pública.

Perfil: tamizaje.

Epidemiología

Pruebas

de

Enfermedad X, se encuentran los siguientes resultados expresados como Odds Ratio (OR) y sus Límites de Confianza (inferior y superior), respecto a cinco factores. Señale cuál de ellos muestra una mayor fuerza de asociación estadísticamente significativa.

2. Un nuevo test para medir concentración de plomo en sangre, detecta como positivos a 160 niños de 200, que por otra prueba se sabe que tienen plumbemia elevada (sobre 12 microgramos por 100 ml). El mismo test aplicado a 200 niños con plumbemia normal, capta como negativos a 190 de ellos. Con estos datos, la sensibilidad y especificidad del nuevo test en término porcentuales, son respectivamente:

a) Antecedentes familiares: OR = 1.3 (0.4 – 2.8) b) Consumo de alcohol OR = 2.3 (1.6 – 3.0) c) Consumo de tabaco: OR = 2.5 (1.0 – 4.0) d) Exposición a solventes orgánicos OR = 2.3 (0.4 – 4.2) e) Bajo consumo de frutas y verduras frescas: OR = 0.98 (0.85 – 4.98) Respuesta correcta B La respuesta correcta es consumo de alcohol, ya que, si bien el consumo de tabaco y exposición a solventes orgánicos presentan una fuerza de asociación igual o superior a la del consumo de tabaco, en ambos casos su límite de confianza inferior toca el valor 1 (no hay asociación) o está por debajo del valor 1, lo cual indica la posibilidad de ser un factor de protección para la enfermedad X. En el caso del consumo de frutas y verduras frescas, la estimación puntual de OR es bajo 1, por lo tanto se trata se una asociación protectora, pero su límite superior toca el valor 1 y alcanza una magnitud de 4.98, lo que indica la posibilidad de no tener asociación (toca el valor 1) o incluso ser un factor de riesgo; además la gran amplitud del intervalo (0,85 a 4,98) nos señala una estimación imprecisa.

a) 80% y 95% b) 95% y 80% c) 40% y 47,5% d) 20% y 5% e) 5% y 20%

Respuesta correcta A La respuesta correcta es 80% y 95%, que son los resultados de dividir 160 por 200 y 190 por 200 respectivamente y multiplicados por 100 para expresarlos en porcentaje. Se asume que la “otra prueba” es el “gold standar”, considerada como la prueba que determina validamente el nivel de plomo en sangre. Por lo tanto se considera que 200 niños efectivamente tienen un nivel elevado de plomo y 200 tienen plumbemia normal. Se debe recordar que siempre una nueva prueba o test debe ser comparado con otro (gold standar), el que se asume válido, para detectar el fenómeno de interés.

Bibliografía L. Gondis.- Epidemiología 3ª Ed. Elsevier 2005 Madrid.

Bibliografía R. Fletcher: Epidemiologia Clínica. Ediciones Consulta S.A. 1989. Barcelona.

Perfil: Epidemiología Estudios de caso - control.

Perfil: Epidemiología Estudios caso - control.

14.- El diseño de casos y controles, se recomienda para estudiar los factores de riesgo de una enfermedad, por ser un diseño:

analítica.

12. - En un estudio de casos y controles sobre factores de riesgo para desarrollar una

a)

209

Apropiado en enfermedades historia natural.

analítica.

de larga

b) indicarle los procedimientos de diagnóstico y tratamiento que está autorizado para efectuar. c) comparar en una pauta preestablecida los requisitos competencias y capacidades reales con que cumple. d) certificar la calidad de las actividades y procedimientos que realiza. e) destacar los méritos y defectos detectados en los servicios que produce.

b) Que no esta expuesto al sesgo de recuerdo o “memoria” c) Que permite establecer tasas de incidencia en casos y controles. d) De tipo prospectivo, para detectar la incidencia de la enfermedad. e) Que permite estimar directamente el riesgo relativo. Respuesta correcta A La respuesta correcta es apropiado en enfermedades de larga historia natural, ya que éste diseño se realiza seleccionando casos de la enfermedad y se compara con un grupo de control adecuado. Por lo tanto, si la enfermedad en estudio presenta una historia natural prolongada entre la exposición a factores de riesgo y la detección de la enfermedad; ese tiempo que puede ser de varios años para enfermedades crónicas prevalentes, se “ahorra” al emplear un diseño de caso control. Sin embargo, por esta forma de construir el diseño, se debe recurrir a casos y controles para que informen sobre la eventual exposición al o los factores de riesgo en el pasado reciente o remoto y de ahí que esté expuesto a sesgos de memoria. Dado que generalmente se desconoce la población (denominadores) que efectivamente originó casos y controles no es posible establecer tasas de incidencia. Es habitualmente retrospectivo, ya que se parte de sujetos que presentan el efecto (enfermedad) y retrospectivamente se establecerá su exposición o grado de exposición al o los factores de riesgo. Por la carencia de tasas de incidencia, no es posible estimar directamente el riesgo relativo y se recurre a un estimador especial denominado Razón de Disparidades (en inglés Odds Ratio).

Respuesta correcta C. La respuesta correcta es c). El proceso de acreditación consiste en definir previamente los estándares mínimos que debe cumplir una organización y luego aplicar la pauta resultante a los establecimientos u organizaciones que se acreditan. El solo definir los requisitos mínimos no es acreditación sino creación de estándares. La autorización de funcionamiento procede pero después de la acreditación y la calidad del producto no es parte de la acreditación. Bibliografía G. Malagón: Auditoría en Salud. Ed. Médica Panamericana 2003, Bogotá.

Perfil: tamizaje.

Pruebas

de

47. La probabilidad de que una persona con un resultado negativo en una prueba de tamizaje, verdaderamente no tenga la enfermedad buscada es: a) El valor predictivo positivo de la prueba. b) La exactitud de la prueba. c) La especificidad de la prueba. d) El valor predictivo negativo de la prueba. e) La sensibilidad de la prueba.

Bibliografía L. Gondis: Epidemiología 3ª Ed. Elsevier 2005, Madrid.

Respuesta correcta D La respuesta correcta es el valor predictivo negativo, es decir, la probabilidad de que una persona con resultado negativo efectivamente no tenga la enfermedad. El valor predictivo positivo es la probabilidad de que una persona con un resultado positivo realmente tenga la enfermedad. Se debe recordar para entender estos concepto que cualquiera prueba o examen aplicado a una población presuntamente sana, presentará una cierta proporción de resultados falsos positivo y falsos negativos, de ahí que, no todos los que presentan un resultado negativo no

Perfil: Bases conceptuales de la administración y gestión. Evaluación y control. 36. Acreditar consiste en:

Epidemiología

un establecimiento de salud

a) definirle los requisitos mínimos con los cuales debe cumplir para que se pueda autorizar su funcionamiento.

210

b) incorporar eficientemente al personal nuevo. c) enseñar métodos productivos modernos. d) capacitar a los menos hábiles e) identificar al personal poco productivo

estén enfermos y por el contrario no todos los que presentan un resultado positivo están realmente enfermos. La exactitud de la prueba es la suma de resultados verdaderos positivos más verdaderos negativos divididos por el total de exámenes. La sensibilidad expresada en términos probabilísticos es la probabilidad de ser captado como positivo, dado que se tiene la enfermedad. La especificidad, por su parte, es la probabilidad de dar un resultado negativo dado que no se tiene la enfermedad.

Respuesta correcta B. El proceso de inducción es parte del “ciclo del personal”, proceso necesario en toda organización, especialmente con muchos empleados. La inducción permite al empleado orientarse y resolver problemas que son básicos para su supervivencia dentro organización.

Bibliografía R. Fletcher: Epidemiologia Clínica. Ediciones Consulta S.A. 1989. Barcelona.

Bibliografía E. Medina y A.M. Kaempfer: Elementos de Salud Pública. Ed. Andrés Bello 1978, Santiago.

Perfil: Bases conceptuales de la administración y gestión. Conceptos básicos. Perfil: Epidemiología Estudios de prevalencia.

61. La delegación de autoridad: a)

está prohibida dentro de la administración pública b) procede automáticamente ante cualquier falta de la autoridad superior c) siempre retiene la responsabilidad final d) está restringida solo al nivel directivo e) es imprescindible ante los imprevistos

106. En un estudio epidemiológico de prevalencia es posible determinar a partir de los datos recolectados, las siguientes mediciones: a)

La incidencia acumulada del fenómeno estudiado. b) La densidad de incidencia del fenómeno estudiado. c) La proporción de expuestos al factor de riesgo. d) El riesgo relativo de los expuestos al factor de riesgo. e) El número necesario a tratar para evitar un caso de la enfermedad

Respuesta correcta C El principio de delegación es universal e indica que se pueden delegar las funciones y la responsabilidad directa de la ejecución pero siempre la responsabilidad final no es delegable, por lo tanto depende de la supervisión. La delegación es siempre formal.

Respuesta correcta C La respuesta correcta es la proporción de expuestos al factor de riesgo, ya que se trata de un estudio transversal en un momento determinado (prevalencia) de una población o en una muestra de la misma donde simultáneamente se determina la presencia de la enfermedad o característica de interés y el o los factores de riesgo a los que se esta expuesto. Dado que no hay seguimiento de poblaciones, es imposible obtener datos sobre incidencia acumulada, densidad de incidencia y por esta dificultad se carece de la información necesaria para estimar el riesgo relativo, el que se obtiene mediante la razón entre incidencias acumuladas o densidades de incidencia en expuesto y no expuestos a un determinado factor de riesgo. El

Bibliografía E. Medina y A.M. Kaempfer: Elementos de Salud Pública. Ed. Andrés Bello 1978 Santiago.

Perfil: Bases conceptuales de la administración y gestión. Conceptos básicos. 82. El proceso de inducción del personal en una organización de salud tiene como objetivo: a)

analítica.

impulsar a sus integrantes a desarrollar un trabajo productivo.

211

nos informaría de los casos nuevos de cáncer, pero no del tamaño de la población efectivamente a cubrir mediante el examen de Papanicolau.

Número Necesario a Tratar (NNT) es una medición utilizada en ensayos clínicos, que requiere tener información sobre incidencias acumuladadas y/o tasas de ocurrencia de un fenómeno de interés en el ensayo clínico (Por ejemplo muertes, remisión de la enfermedad, frecuencia de complicaciones etc.)

Bibliografía E. Medina y A.M. Kaempfer.- Elementos de Salud Pública. Ed. Andrés Bello 1978 Santiago.

Bibliografía L. Gondis: Epidemiología 3ª Edición Elsevier 2005, Madrid.

Perfil: Proceso salud Diagnóstico situación salud.

enfermedad.

161. ¿Cuál de los siguientes indicadores, es más apropiado para solicitar los recursos necesarios para aumentar la cobertura de exámenes de Papanicolau en la población de mujeres entre 25 y 34 años de edad? a)

Tasa de mortalidad por cáncer cérvico uterino en mujeres entre 25 a 34 años. b) Número de hospitalizaciones por cáncer cérvico uterino en mujeres entre 25 a 34 años. c) Prevalencia estimada de cáncer cérvico uterino en mujeres entre 25 a 34 años. d) Número absoluto de mujeres entre 25 a 34 años. e) Incidencia estimada de cáncer cérvico uterino en mujeres entre 25 a 34 años. Respuesta correcta D La respuesta correcta es número absoluto de mujeres entre 25 a 34 años, ya que informa claramente sobre el número real de mujeres en ese grupo etareo a las que se debe realizar un Papanicolau con una mayor cobertura que la actualmente lograda, por lo tanto, permite establecer los recursos necesarios con mayor precisión. La tasa de mortalidad, el número de hospitalizaciones y la prevalencia estimada de cáncer cervico uterino en ese grupo de edad, informan de casos seleccionados (los más graves y que fallecieron), los que están presumiblemente en etapas más avanzada (hospitalizadas) o prevalentes clínicamente, por lo tanto son de poca utilidad para el objetivo que se plantea en la pregunta (aumentar cobertura del examen). La incidencia estimada requeriría seguir a una población presuntamente sana durante un tiempo relativamente prolongado y

212